Sunteți pe pagina 1din 146

THE UNIVERSITY OF MELBOURNE

Department of Mathematics and


Statistics

620-311
Metric Spaces
Lecture Notes
Semester 1, 2004
Notes by:

Kris Wysocki

This compilation has been made in accordance with the provisions of Part
VB of the copyright act for the teaching purposes of the University. For
the use of students of the University of Melbourne enrolled in the subject
620-311.

Metric and Topological Spaces

Contents
1 Introduction

Metric Spaces

3 Continuity

18

4 Complete Spaces

22

5 Compact Metric Spaces

34

6 Topological Spaces

40

7 Compact Topological Spaces

44

8 Connected Spaces

46

9 Product Spaces

50

10 Uryshons and Thietzes Theorems

56

11 Appendix

59

12 Problem Sheets
12.1 Problem Sheet
12.2 Problem Sheet
12.3 Problem Sheet
12.4 Problem Sheet
12.5 Problem Sheet
12.6 Problem Sheet
12.7 Problem Sheet

69
69
71
73
74
76
77
79

1
2
3
4
5
6
7

.
.
.
.
.
.
.

.
.
.
.
.
.
.

.
.
.
.
.
.
.

.
.
.
.
.
.
.

.
.
.
.
.
.
.

.
.
.
.
.
.
.

.
.
.
.
.
.
.

.
.
.
.
.
.
.

.
.
.
.
.
.
.

.
.
.
.
.
.
.

.
.
.
.
.
.
.

.
.
.
.
.
.
.

.
.
.
.
.
.
.

.
.
.
.
.
.
.

.
.
.
.
.
.
.

.
.
.
.
.
.
.

.
.
.
.
.
.
.

.
.
.
.
.
.
.

.
.
.
.
.
.
.

.
.
.
.
.
.
.

.
.
.
.
.
.
.

.
.
.
.
.
.
.

.
.
.
.
.
.
.

.
.
.
.
.
.
.

.
.
.
.
.
.
.

12.8 Problem
12.9 Problem
12.10Problem
12.11Problem

Sheet
Sheet
Sheet
Sheet

8
9
10
11

.
.
.
.

.
.
.
.

.
.
.
.

.
.
.
.

.
.
.
.

.
.
.
.

.
.
.
.

.
.
.
.

.
.
.
.

.
.
.
.

.
.
.
.

.
.
.
.

.
.
.
.

.
.
.
.

.
.
.
.

.
.
.
.

.
.
.
.

.
.
.
.

.
.
.
.

.
.
.
.

.
.
.
.

.
.
.
.

.
.
.
.

.
.
.
.

.
.
.
.

80
81
83
84

Introduction

The ideas of limit and continuity which we encounter in Euclidean spaces


occur in various other contexts e.g in function spaces. Set topology is the
study of limits and continuity in a general setting. The notion of limit is
based on the idea of nearness. These concepts are easier to perceive when
the notion of nearness is given by distance. The corresponding spaces are
called metric spaces. These are introduced in Chapter 2 and applications to
function spaces are discussed early. The desirability of finding limits leads
to the notion of completeness and compactness. As we go on, we find that
many of the arguments do not really need the notion of distance. This leads
to the concept of topological spaces which are discussed from Chapter 6
onward. The idea of compactness is discussed in general setting in Chapter
7 and the notion of connectedness (which is related to the Intermediate Value
Theorem) is discussed in Chapter 8. Under mild assumptions we can study
abstract toplogical spaces by constructing continuous functions to the real
line; the results known as Uryshon and Thitzes theorem are discussed in
Chapter 10. The concepts of completeness and compactness come again in
the guise of the important Ascoli-Arzela theorem are discussed in Chapter
9. The necessary preliminary material is collected in Chapter 11. The are
11 problem sheets which do not exactly correspond to the chapters of the
notes. This notes is only a brief introduction to the subject and we refer
to Munkres Topology [Mu] for comprehensive treatment. More elementary
introductions are the books by Mendelson [M] and Croom [C].

Metric Spaces

Basic Concepts
By a metric space we mean a set X together with a function d : X X
[0, ) which satisfies the following axioms:
M1 d(x, y) = 0 if and only if x = y;
M2 d(x, y) = d(y, x) for every x, y X;
M3 d(x, z) 6 d(x, y) + d(y, z) for every x, y and z X.
Elements of X are called points, a function d is called a metric on X,
and the value d(x, y) is called a distance between x and y. The axiom M2
says that a metric is symmetric, and the axiom M3 is called the triangle
inequality since it reflects the geometrical fact that the length of one side
of a triangle is less or equal to the sum of the lengths of the other two sides.

Examples
Example 2.1. The most important example of a metric space is the set of
all real numbers R with the metric d(x, y) = |x y|. In the following we will
call this metric the usual metric in R.

Example 2.2. Let X be any set and let


(
0 x = y,
d(x, y) =
1 x=
6 y.
Then d is a metric on X called the discrete metric.

Example 2.3. Any subset Y of a metric space (X, d) becomes a metric


space with the metric
for all x, y Y .

dY (x, y) = d(x, y)

The pair (Y, dY ) is called a metric subspace of (X, d). We will refer to Y
as a subspace of X, rather than (Y, dY ) as a subspace of (X, d).

Example 2.4. [Cartesian product of finite number of metric spaces].


Consider a finite collection
Qn of metric spaces (X i , di ) , 1 6 i 6 n, and let X be
the
cartesian
product
i=1 Xi . For x = (x1 , . . . , xn ) and y = (y1 , . . . , yn )
Qn
i=1 Xi , set
n
X
d(x, y) =
di (xi , yi ).
i=1

Then d is a metric on X. Clearly, axioms M1-M2 are satisfied. To see that d


satisfies M3 take x = (x1 , . . . , xn ), y = (y1 , . . . , yn ) and z = (z1 , . . . , zn ) X.
Then
d(x, z) =

n
X

di (xi , zi ) 6

i=1

n
X

i=1


di (xi , yi ) + di (yi , zi ) = d(x, y) + d(y, z)

as required. The pair (X, d) defined above is called a metric product (or
just a product) of (Xi , di ), 1 6 i 6 n, andQthe metric d is called a product
metric. (Other metrics are also used on ni=1 Xi ).

Norms and normed vector spaces


We next define the class of metric spaces which are the most interesting in
analysis. Let X be a vector space over R (or C).

Definition 2.5. A norm is a function : X R having the following
properties


N1 x 0 and x = 0 if and only if x = 0.


N2 x = || x for all x X and R.


N3 x + y 6 x + y for all x, y X.

The pair (X, ) is called a normed vector space.
Proposition 2.6. Let X be a normed space. Then


d(x, y) = x y
is a metric on X.

Proof. The axioms M1 and M2 are clear. If x, y and z X, then, in view


of N3,



d(x, z) = x z = (x y) + (y z)



6 x y + y z = d(x, y) + d(y, z),
and so the triangle inequality follows.

Examples of normed spaces


Example 2.7. [Euclidean Space] Consider R n and let
#1/2
" n
X

x =
x2i
i=1

for x = (x1 , . . . , xn ) Rn . Clearly, N1 and N2 are satisfied. To see that N3


holds we need the Cauchy inequality.
Lemma 2.8. For x, y Rn ,
X
X
1/2
1/2 X
n
n

n
2
2

.
|yi |

|xi |
xi yi 6

i=1

i=1

i=1

Proof.

06
=
=
=

n
X

(xi yj xj yi )2 =

i,j=1
n X
n
X

n
X

(x2i yj2 2xi xj yi yj + x2j yi2 )

i,j=1
n
n
n X
n
XX
X
x2i yj2 +
x2j yi2 2
xi xj yi yj
i=1 j=1
i=1 j=1
i=1 j=1
X
2
n
n
n
X
X
2 2
2 2
y xi +
x yj 2
xi yi
i=1
j=1
i=1
X
2
n
2 2
2 x y 2
xi yi
i=1

As a corollary we have
Corollary 2.9.


x + y 6 x + y
7

for all x, y Rn .

Proof. In view of the Cauchy inequality we have


n
n
n
n
X
X
X
X


x + y 2 =
|xi + yi |2 =
x2i + 2
xi yi +
yi2
i=1

i=1

i=1

i=1

n
X
2 2
2
2


xi yi + y 6 x + 2 x y + y
= x +2

i=1

= ( x + y )2 .

By taking square roots of both sides we the desired inequality follows.

Consequently,
#1/2
" n
X


(xi yi )2
d(x, y) = x y =
i=1

Rn .

defines a metric on
the standard metric.

We shall call this metric the Euclidean metric or

Example 2.10. [Space of bounded functions. ] Let X be a non-empty


set. Call a function f : X R bounded if there exists a constant M such
that |f (x)| 6 M for all x X. Denote by B(X) = B(X, R) the set of all
bounded functions from X to R, and define
kf k = sup{|f (x)| | x X}.
Then k k is a norm on B(X), and, in view of Proposition 2.6, this norm
defines a metric on B(X) by


d(f, g) = f g = sup{|f (x) g(x)| | x X},

for f, g B(X).

Example 2.11. Let X be the set of all continuous functions f : [0, 1] R.


For any f X we set
Z 1

f =
|f (x)|dx.
0

Then k k defines a norm on X which induces a metric on X by


Z 1
d(f, g) =
|f (x) g(x)|dx, f, g X
0

Balls and diameter


Let x0 X and r > 0. The set
B(x0 , r) = {x X | d(x0 , x) < r}
is called an open ball with centre at x 0 and radius r > 0, and the set
B(x0 , r) = {x X | d(x0 , x) 6 r}
is called a closed ball with centre at x 0 and radius r > 0.
If A is a non-empty subset of X, then we define the distance between x
and A by
d(x, A) = inf{d(x, y) | y A}
and more generally if B is another non-empty subset of X, then the distance
between A and B is defined as
d(A, B) = inf{d(x, y) | x A, y B}.
For a non-empty subset A of X we define its diameter by setting
diam A = sup{d(x, y) | x, y A}.
Clearly, if A B, then diam A 6 diam B. A subset A X bounded if its
diameter is finite, that is, diam A < .

Sequences and Convergence


Convergence of a sequence in a metric space is defined as in calculus.
Definition 2.12. Let {xn } be a sequence of points in (X, d) and x X.
The sequence {xn } is said to converge to x, if for every > 0 there exists
a positive integer k such that
for all n k.

d(xn , x) <

A sequence {xn } is said to converge if there is x X to which it converges.


If there is no such x, then {xn } is said to diverge. If {xn } converges to x
we write limn xn = x0 or xn x. The point x is called the limit of {x n }.
The definition can be expressed in terms of the convergence of sequences of
real numbers. Namely, a sequence {x n } converges to x X if and only if
d(xn , x) 0 as n . We are justified in referring to the limit because of
the following proposition.
9

Proposition 2.13. Let {xn } be a sequence in a metric space (X, d). Then
there is at most one point x X such that {x n } converges to x.

Proof. Arguing by contradiction we assume that x n x and xn y with


x 6= y. Then d(x, y) > 0 and we can apply the above definition of convergence with = d(x, y)/2. We find a positive integer k such that
d(xn , x) <

and d(xn , y) < ,

for n k.

By the triangle law


d(x, y) 6 d(x, xn ) + d(xn , y) < + = d(x, y)
which gives a contradiction. Hence we conclude that it is impossible for a
sequence {xn } to converge to two different points.

Given a sequence {xn } of points in X, consider a sequence {n k } such that
n1 < n2 < n3 < . Then {xnk } is called a subsequence of {xn }.
Q
Proposition 2.14. If X = ni=1 Xi is the product of metric spaces (Xi , di ),
m
m
m
1 6 i 6 n, and xm = (xm
1 , x2 , . . . , xn ) X, then x x = (x1 , . . . , xn )
m
X if and only if xi xi in Xi for i = 1, . . . , n.
Proof. Recall that we consider X with the metric
d(x, y) =

n
X

di (xi , yi )

i=1

for x = (x1 , . . . , xn ), y = (y1 , . . . , xn ) X. Observe that


di (xi , yi ) 6 d(x, y) 6 n max{di (xi , yi ) | 1 6 i 6 n},

x, y X.

(1)

Let xm x, where x = (x1 , . . . , xn ). Then given > 0 there exists k N


such that
d(xm m, x) < for m k.
In view of the left hand side inequality (1)
dj (xm
j , xj ) <

for m k and j = 1, . . . , n.

m
So xm
j xj as required. Conversely, assume that x j xj for j = 1, . . . n.
Hence for a given > 0, there exists k(j) N such that

dj (xm
j , xj ) < /n

for m k(j).

In view of the right hand side inequality in (1) we get


d(xm , x) 6 n max{dj (xm
j , xj ) | j = 1, . . . , n} <
for all m > k := max{k(j) | j = 1, . . . , n}. Hence x n x as required.
10

Definition 2.15. Two metrics d and d0 in X are called equivalent if


d(xn , x0 ) 0

if and only if

d0 (xn , x0 ) 0.

Example 2.16. Let d be a metric on X. Define


d0 (x, y) =

d(x, y)
,
1 + d(x, y)

x, y X.

(2)

Then d0 is a metric on X (show this!)

which is equivalent to d. Ind(xn , x0 )


deed, if d(xn , x0 ) 0, then d0 (xn , x0 ) =
0. Conversely,
1 + d(xn , x0 )
d0 (x, y)
. So if d0 (xn , x0 ) 0, then d(xn , x0 ) 0. Note
d(x, y) =
1 d0 (x, y)
that with respect to this equivalent metric, the space X is bounded since
d0 (x, y) < 1 for all x, y X.
Example 2.17. Consider the product (X, d) of metric spaces (X i , di ). Recall that
d(x, y) =

n
X

di (xi , yi ),

i=1

Set

x = (x1 , . . . , xn ), y = (y1 , . . . , yn ) X.

(x, y) = max{di (xi , yi )| 1 6 i 6 n}


1/2
X
n
(x, y) =
.
di (xi , yi )2
i=1

Then d is equivalent to and .

Open and closed sets


Definition 2.18. Let A X. A point x A is called an interior point
of A, if B(x, r) A for some r > 0. The collection of all interior points of
a set A is called the interior of A, and is denoted by A . A set A is called
open if A = A .
Obviously, the interior of any set is an open set. Hence open sets A are
characterized by equality A = A.

11

Example 2.19. The empty set and the whole space X are open in any
metric space X. If X is equipped with the discrete metric d, then any subset
of X is open.

Example 2.20. The set Q is not open in R with the usual metric but it
is open in (R, d), where d is the discrete metric in R. Indeed, if x Q and
r > 0, then for large n N, we have

2
x<x+
<x+r
n

so that x + 2/n B(x, r) but x + 2/n 6 Q. In the case of the discrete


metric, for every x Q, B(x, 1/2) = {x} Q, so Q is open in (R, d).
Example 2.21. Let B(x, R) be an open ball in a metric space X. Then
B(x, R) is an open set. Indeed, let y B(x, R). We have to show that y is
an interior point of B(x, R), that is, B(y, r) B(x, r) for some r > 0. Set
r = R d(x, y). Then for any z B(y, r),
d(x, z) 6 d(x, y) + d(y, z) < d(x, y) + r = d(x, y) + [R d(x, y)] = R.
Thus B(y, r) B(x, r) as required.

y
r
x
PSfrag replacements
R

Figure 1: An open ball is an open set

12

More terminology: if x X, then a set A X is called a neighbourhood


of x, if x A .
Definition 2.22. A point x X is adherent to A provided that B(x, r)
A 6= for all r > 0. The set of all the adherent points of A is called the
closure of A and is denoted by A. If A = A, then A is called closed.
Proposition 2.23. A point x is adherent to A if and only if there exists a
sequence in A converging to x.
Proof. Suppose that x A. In view of the definition, for each positive
integer n, there exists a point xn B(x, 1/n) A. Obviously, {xn } is the
sequence of points of A converging to x. Conversely, suppose that {x n } A
and xn x. Let r > 0. Then d(xn , x) < r for n greater than some k. Hence
xn B(x, r) A and x is adherent to A.

Example 2.24. Let B(x, r) be a closed ball in X. Then it is a closed set
in X. To see this we have to show that all adherent points of B(x, r) are
contained in B(x, r). If y is adherent B(x, r), then y n y for some sequence
{yn } B(x, r). Since
d(y, x) 6 d(y, yn ) + d(yn , x) 6 d(y, yn ) + r r,
it follows that y B(x, r) as required.
Example 2.25. The closure of an open ball B(x, r) does not have to coincide with a closed ball B(x, r). Indeed, consider X = R \ (0, 1) with the
usual metric, d(x, y) = |x y|. Then
B(0, 1) = [1, 0]

but B(0, 1) = [1, 0] {1}.

Example 2.26. A subset of metric space may be neither open nor closed.
For instance, [0, 1) is neither open nor closed in R. The same is true for Q.
On the other hand, a subset may be open and at the same time closed. In a
metric space equipped with the discrete metric any subset is both open and
closed.
The relation between interior and adherent points is given in the next proposition.
13

Proposition 2.27. A point x X is an adherent point of A if and only if


x is not an interior point of Ac .
Proof. Assume that x is adherent to A. Then for every open ball at x,
B(x, r) A 6= . Hence there is no open ball B(x, r) contained in A c which
means that x 6 (Ac ) . Conversely, assume that x 6 (Ac ) . Hence there is
no open ball B(x, r) contained in Ac . Hence B(x, r) A 6= , for all r > 0,
which means that x is adherent to A.

As a corollary we obtain.
Corollary 2.28. If A X, then
X \ A = (X \ A)

and

X \ A = X \ A.

A set A is closed if and only if X \ A is open, and A is open if and only if


X \ A is closed.
Theorem 2.29 (Properties of Interiors and Closures).
(a) A A

(a0 ) A A

(c) A B = A B

(c0 ) A B = A B

(b) (A ) = A

(b0 ) A = A

(d) (A B) = A B
!
[
[
(e)
Ai
Ai
iI

(f )

iI

Ai

(d0 ) A B = A B
(e0 )

iI

iI

Ai

(f 0 )

iI

Ai

iI

Ai

iI

Ai

Ai

iI

Proof. (a) follows immediately from the definition of the interior point. To
see (b) note that A is an open. (c): If A B and x A , then B(x, r)
A B. So x B . (d): Note that (A B) A and (A B) B
so (A B) A B . On the other hand A B is open and contained
in A B,so by (b), A B (A B) . Proofs of (e) and (f) are left as
exercises. Proofs of (a)-(f) follows from the corresponding statements for
interiors by taking complements.


14

Theorem 2.30 (Properties of open and closed sets).


(a) and X are open
[
(b) {Ai }iI open
Ai open

(a0 ) and X are closed


\
(b0 ) {Ai }iI closed
Ai closed

(c) {Ai }m
i=1 open

(c0 ){Ai }m
i=1 closed

iI
m
\

Ai open

i=1

(d) A =the largest open set

iI

m
[

Ai is closed

i=1

(d0 ) A =the smallest closed set

contained in A

containing A

Proof. The parts


S (a) and (a) are obvious.
(b) Let x iI Ai . S
Choose an index j I so thatSx Ai . Since Aj is
open, B(x, r) S
Aj iI Ai . Hence any point x iI Ai is an interior
point, and so iI Ai is open.
T
(c) Assume Ai X, i = 1, . . . , m are open subsets of X, and let x m
i=1 Ai .
Then x Ai for i = 1, . . . , m. Since the sets Ai are open, T
B(x, ri ) Ai for
someTri > 0. Take r = min{r1 , . . . , rm }. Then B(x, r) m
i=1 Ai , and the
m
sets i=1 Ai is open.
(d) is left as an exercise. (a)-(d) are obtained from corresponding statements for open sets by taking complements and applying Corollary 2.28 
Theorem 2.31. Let Y be a subspace of X.
(a) B Y is open in Y if and only if B = Y A for some open set A in
X.
(b) B Y is closed in Y if and only if B = Y F , where F is closed in
X.
Proof.
(a) Assume first that B = Y A for some open set A in X. Take x B.
Then there exists an open ball B(x, r) in X such that B(x, r) A. But
then Y B(x, r) Y A = B. Since the open ball in the subspace Y with
centre x X and radius r > 0 is the intersection Y B(x, r), the set B is
open in Y . Conversely, suppose that B is an open subset of the subspace Y .
Then for every x B there exists rx such that the
S open ball B(x, rx ) Y in
Y is contained in B. Then the open subset A = xB B(x, rx ) of X satisfies
Y A B. Since any x B also belongs to A, Y A = B as required.
(b) A set B is closed in Y if and only if Y \ B is open in Y . Hence if and
only if Y \ B = Y A for some open subset A of X. Let F = X \ A. Then
15

F is closed in X and B = Y \ [Y A] = Y \ A = Y [X \ A] = X F as
required.

Theorem 2.32. Let X be the product of metric spaces (X i , di ), 1 6 i 6 m.
Q
(a) If Ai is open in Xi ,Q
1 6 i 6 m, then the product A = ni=1 Ai is an
open subset of X = ni=1 Xi .
Q
(b) If Fi is closedQin Xi , 1 6 i 6 m, then F = m
i=1 Fi is closed in the
m
product X = i=1 Xi .

Proof.
(a) We prove the result for the product of two metric spaces X 1 and X2 .
Let a = (a1 , a2 ) A X. Since Ai is open in Xi , there exists ri such that
an open ball B(ai , ri ) in Xi is contained in Ai . Let r = min{r1 , r2 }. We
claim that B(a, r) A. Indeed, if x = (x 1 , x2 ) B(a, r), then d(a, x) < r
where x = (x1 , x2 ),and since di (ai , xi ) < d(a, x) < r 6 ri we conclude that
xi B(ai , ri ). Hence xi Ai , i = 1, 2, so that x A.
(b) The proof follows from Proposition 2.14


Definition 2.33. The boundary of A in X, denoted by A, is the set


A X \ A.
Hence x A if for any r > 0 an open ball B(x, r) intersects A and X \ A as
well. Clearly, the boundary is a closed set as an intersection of closed sets.
Example 2.34. Consider R with the usual metric. Then
([0, 1]) = ((0, 1)) = {0, 1}
(Q) = (R \ Q) = R.

We shall show the last equality. Fix x R. If x Q, then

2
1
Qc .
x 6= x + Q and x 6= x +
n
n
Since

x = lim(x + 1/n) = lim(x + 2/n),


n

it follows that x
So Q
= Q. If x Qc , then x+1/n Qc
and there exists a sequence of rational numbers x n such that
QQc.

QQc

x = lim(x + 1/n) = lim xn .


n

Hence x Q Qc and (Q) = (Qc ) = Q Qc = R.

16

Definition 2.35. A point x X is called isolated if {x} is open. A space


X is called discrete if all of its points are isolated.
If x is an isolated point, then for some > 0, an open ball B(x, ) {x},
that is, B(x, ) = {x} and if y 6= x, then d(x, y) . Conversely, if
inf{d(x, y) | y 6= x} > 0, then {x} is open. Note also that {x} is always closed. For example, consider N as subspace of R. Then it is discrete.
Also the space J = {1/n | n N} is discrete. In a discrete space any set
is open, since it is a union of one-point sets which are open. Also any set
is closed being a complement of an open set. Finally, a space is discrete
if and only if the only convergent sequences are those which are eventually
constant (Prove this!).
Definition 2.36. A subset A of a metric space is dense if A = X.
Example 2.37. The sets Q and Qc are dense in R with the usual metric.
Proposition 2.38. Let X be a metric space and A X. Then A is dense if
and only if for every non-empty open set U of X, the intersection U A 6= .
Definition 2.39. A subset A of X is called nowhere dense if (A) = .

Example 2.40. The sets of all natural numbers N or all integers Z are
nowhere dense in R with the usual metric. The set of real numbers R is
nowhere dense in R2 with the standard metric.
Example 2.41. [Cantor set] The Cantor set is subset of [0, 1] constructed
as follows:
Consider the interval C0 = [0, 1]. At the first step divide C0 into three equal
intervals [0, 1/3], [1/3, 2/3] and [2/3, 1] and remove the middle open interval
(1/3, 2/3). Denote the remaining intervals by C 1 = [0, 1/3] [2/3, 1]. The
length of intervals which constitute C 1 is equal to 2/3. In the second step we
perform the same operations as in the first step on each of the intervals of C 1 .
We remove intervals (1/9, 2/9) and (7/9, 8/9). Denote the four remaining
intervals by C2 . Having finished the step (n 1), we perform the nth step
and obtain the set Cn consisting of 2n intervals.
Each of the sets Cn is closed and bounded, and Cn+1 Cn . The Cantor set
is defined as

\
C=
Cn
n=1

17

It is non-empty and since for every n, C n is closed, C is closed. The set


C does not contain any interval (show this!), and so, C has empty interior.
Hence C is nowhere dense.

Continuity

The definition of continuity is the definition of calculus.


Definition 3.1. Let (X, d) and (Y, ) be metric spaces and let f : X Y be
a function. The function f is said to be continuous at the point x 0 X
if the following holds: for every > 0, there exists > 0 such that for all
x X if d(x, x0 ) < , then (f (x), f (x0 )) < . The function f is said to be
continuous if it is continuous at each point of X.
The following proposition rephrases the definition in terms of open balls.
Proposition 3.2. Let f : X Y be a function from a metric space X to
another metric space Y and let x0 X. Then f is continuous at x0 if and
only if for every > 0 there exists > 0 such that
f (B(x0 , )) B(f (x0 ), ).
Theorem 3.3. Let f : X Y be a function from a metric space (X, d)
to another metric space (Y, ) and let x 0 X. Then f is continuous at x0
if and only if for every sequence {xn } such that xn x0 , f (xn ) f (x0 ).
And f is continuous if and only if for every convergent sequence {x n } in X,
lim f (xn ) = f (lim xn ).
n

Proof. Suppose that f is continuous at x 0 and let xn x0 . We will prove


that f (xn ) f (x0 ). Let > 0 be given. By the definition of continuity at
x0 , there exists > 0 such that for all x X,
if d(x, x0 ) < , then (f (x), f (x0 )) < .

(3)

Since xn x0 , there exists an integer k such that for all n k,


d(xn , x0 ) < .

(4)

Combining (3) and (4), we get


(f (xn ), f (x0 )) <
18

for all n k.

(5)

Hence f (xn ) f (x0 ) as required. Conversely, arguing by contradiction


assume that f is not continuous at x0 . To obtain a contradiction we will
construct a sequence {xn } such that xn x0 but the sequence {f (xn )}
does not converge to f (x0 ). Since f is not continuous at x0 , there is positive
> 0 such that for all > 0 there exists x satisfying d(x, x 0 ) < but
(f (x), f (x0 )) . For each n, take = 1/n and then choose x n so that
d(xn , x0 ) < 1/n but (f (xn ), f (x0 )) . Hence xn x0 but the sequence
{f (xn )} does not converge to f (x0 ). The second part of the theorem is an
immediate consequence of the first.

Global continuity has a simple formulation in terms of open and closed sets.
Theorem 3.4. Let f be a function from a metric space (X, d) to (Y, ).
Then f is continuous if and only if for every open set U Y , f 1 (U ) is
open in X.
Proof. Suppose first that f is continuous and U is open in Y . If x f 1 (U ),
then f (x) U . Since U is open in Y and f (x) U , there exists a positive
number such that B(f (x), ) U . In view of Proposition 3.2, there exists
> 0 such that f (B(x, )) B(f (x), ). Hence B(x, ) f 1 (f (B(x, )))
f 1 (U ), so f 1 (U ) is open in X. Conversely, suppose that f 1 (U ) is open
in X for every open set U in Y . Let x X and let > 0 be given.
Since B(f (x), ) is open in Y , the set f 1 (B(f (x), )) is open in X. Since
x f 1 (B(f (x), )), there exists > 0 such that B(x, ) f 1 (B(f (x), )).
This implies that f (B(x, )) B(f (x), ), and in view of Proposition 3.2, f
is continuous.

Theorem 3.5. Let f be a function from (X, d) to (Y, ). Then f is continuous if and only if for every closed set F Y , f 1 (F ) is closed in X.
The proof is left as an exercise.
Theorem 3.6. Let X, Y and Z be three metric spaces.
(a) If f : X Y and g : Y Z are continuous, then the composition
g f is continuous.
(b) If f : X Y is continuous, and A is a subspace of X, then the
restriction of f to A, f|A : A Y , is continuous.
Proof. (a) Let xn x0 . Since f is continuous at x0 , f (xn ) f (x0 ). Since
g is continuous at f (x0 , g(f (xn )) g(f (x0 )). Hence g f (xn ) g f (x0 ).
The second statement follows from the first. Here is another proof of the
19

second statement. Let U be an open subset of Z. Since g is continuous,


g 1 (U ) is open in Y , and since f is continuous, f 1 (g 1 (U )) is open in X.
But f 1 (g 1 (U )) = (g f )1 (U ) and so, (g f )1 (U ) is open in X. Hence
g f is continuous.
(b) Note that f|A = f j, where j : A X is the inclusion, i.e., defined by
j(x) = x for x X. Since for any open set U in X, j 1 (U ) = U A which
is open in A, it follows that j is continuous. So (b) follows from (a).

Theorem 3.7. Let (X, d), (Y1 , 1 ) and (Y2 , 2 ) be metric spaces. Let f be a
function from X to Y1 and g a function from X to Y2 . Define the function
h from X to the product Y1 Y2 by
h(x) = (f (x), g(x)),

for x X.

Then h is continuous at x0 if and only if f and g are continuous at x 0 . And


h is continuous if and only if both functions f and g are continuous.
The similar statement about functions from the direct product does not hold
in general. Suppose that f is a function from X Y to Z. It may happen
that f discontinuous, though the maps x 7 f (x, y) for every y Y and
y 7 f (x, y) for every x X are all continuous. For example, consider a
function from R R to R defined by
xy

for (x, y) 6= (0, 0);


f (x, y) = x2 + y 2
0
for (x, y) = (0, 0).

The function f is discontinuous at (0, 0) but all the functions x 7 f (x, y)


and y 7 f (x, y) are continuous.

Theorem 3.8 (The pasting lemma). Let X = A B, where A and B


are closed subspaces of X. Let f : A Y and g : B Y be continuous. If
f (x) = g(x) for all x A B, then the function h : X Y defined by
(
f (x) if x A;
h(x) =
g(x) if x B
is continuous.
Proof. Let C be a closed subset of Y . Then h 1 (C) = f 1 (C) g 1 (C).
Since f is continuous, f 1 (C) is closed in A. But since A is closed f 1 (C)
is closed in X. Similarly, g 1 (C) is closed in X. So h1 (C) is closed in X
and the proof is finished.

20

Uniform Continuity and Uniform Convergence


Definition 3.9. A mapping f from a metric space (X, d) to a metric space
(Y, ) is said to be uniformly continuous, if for every > 0, there exists
> 0 such that (f (x), f (y)) < for all x, y X satisfying d(x, y) < .
Obviously, a uniformly continuous function is continuous.
Example 3.10. The function f (x) = x/(1 + x 2 ) from R to R is uniformly
continuous. To see this observe that for any x < y, in view of the mean
value theorem of calculus, there exists t (0, 1) such that


1 t2
0
|x y| 6 |x y|.

|f (x) f (y)| = |f (t)| |x y| =
(1 + t2 )2

since |f 0 (t)| 6 1. Hence for given , choose = . Then for any x, y sych
that d(x, y) = |x y| < , we have
d(f (x), f (y)) = |f (x) f (y)| 6 |x y| = d(x, y) < = .
So f is uniformly continuous.

Example 3.11. The function f (x) = x2 for x R is not is uniformly


continuous. Indeed, for a given > 0 we can set
x = 1/ + /2

and

y = 1/,

then |x y| = /2 < but |x2 y 2 | > 1. However, if we consider the


same function on some bounded interval, say [a, a], then the function is
uniformly continuous since if < /2a and x, y [a, a] with |x y| < ,
then |x2 y 2 | = |x y| |x + y| < 2a|x y| < .
Let (X, d) and (Y, ) be metric space. Consider a sequence {f n } of functions
fn : X Y and let f : X Y .
Definition 3.12. The sequence {fn } is said to converge pointwise to f
if for every x X and for every > 0, there exists an index N such that
(fn (x), f (x)) <

for all n N .

The sequence {fn } is said to converge uniformly to f if for every > 0,


there exists an index N such that
(fn (x), f (x)) <

for all n N and all x X.


21

Equivalently, {fn } converges uniformly to f on X if


sup{(fn (x), f (x)) | x X} 0.
The notion of uniform convergence of a sequence of functions is, in general,
more useful than that of pointwise convergence.
Theorem 3.13. Let {fn } be a sequence of continuous functions from a
metric space (X, d) to a metric space (Y, ). Suppose that {f n } converges
uniformly to f from X to Y . Then f is continuous.
In words, the uniform limit of continuous functions is continuous.
Proof. Let x0 X and let > 0 be given. Since {fn } converges uniformly
to f , there exists an index N such that for all n N and all x X,
(fn (x), f (x)) < /3.

(6)

Since fN is continuous at x0 , we can choose > 0 so that


(fN (x), fN (x0 )) < /3

(7)

for all d(x, x0 ) < . Now if d(y, x0 ) < , then


(f (y), f (x0 )) 6 (f (y), fN (y)) + (fN (y), fN (x0 )) + (fN (x0 ), f (x0 )).
Each term of the right-hand side is less than /3, the first and the third in
view of (6) and the second in view of (7). Thus
(f (y), f (x0 )) <
for all d(y, x0 ) < . This proves that f is continuous.

Complete Spaces

Definition 4.1. Let (X, d) be a given metric space and let {x n } be a sequence of points of X. We say that {xn } is Cauchy (or satisfies the
Cauchy condition) if for every > 0 there exists k N such that
d(xn , xm ) <

for all n, m k.

Properties of Cauchy sequences are summarized in the following propositions.


22

Proposition 4.2. If {xn } is a Cauchy sequence, then {xn } is bounded.


Proof. Take = 1. Since {xn } is Cauchy, there exists an index k such that
d(xn , xk ) < 1 for all n k. Let R > 1 be such that than d(x i , xk ) < R for
1 6 i 6 k 1. Then xn B(xk , R) for all n, so {xn } is bounded.

Proposition 4.3. If {xn } is convergent, then {xn } is a Cauchy sequence.
Proof. Assume that xn x. Then for a given > 0 there exists k N such
that d(xn , x) < /2 for all n k. Hence taking any n, m k,
d(xn , xm ) 6 d(xn , x) + d(x, xm ) < /2 + /2 = .
So {xn } is Cauchy.

Proposition 4.4. If {xn } is Cauchy and it contains a convergent subsequence, then {xn } converges.
Proof. Assume that {xn } is Cauchy and xkn x. We will show that xn
x. Let > 0. Since {xn } is Cauchy, there exists k 0 such that d(xn , xkn ) < /2
for all n k 0 . Also since xkn x, there exists k 00 such that d(xkn , x) < /2
for all n k 00 . Set k = max{k 0 , k 00 }. Then for n k,
d(xn , x) 6 d(xn , xkn ) + d(xkn , x) < /2 + /2 =
showing that xn x.

A Cauchy sequence need not converge. For example, consider {1/n} in the
metric space ((0, 1), | |). Clearly, the sequence is Cauchy in (0, 1) but does
not converge to any point of the interval.
Definition 4.5. A metric space (X, d) is called complete if every Cauchy
sequence {xn } in X converges to some point of X. A subset A of X is called
complete if A as a metric subspace of (X, d) is complete, that is, if every
Cauchy sequence {xn } in A converges to a point in A.
By the above example, not every metric space is complete; (0, 1) with the
usual metric is not complete.
Theorem 4.6. The space R with the usual metric is complete.
Proof. Let {xn } be a Cauchy sequence in R. Then it is bounded, say |x n | 6
M . Set yn = inf{xk | k n}. Then {yn } is increasing and yn 6 M for all
n. Hence {yn } converges, say to x (see Proposition 11.11 in Appendix). We
23

claim that also xn x. To see this choose N so that |xn xm | < /2 for
n, m N . In particular,
xN /2 < xk < xN + /2

for all k N .

xN /2 6 yn 6 xN + /2

for all n N .

Hence
Let n . Then

xN /2 6 x 6 xN + /2,

or equivalently, |xN x| 6 /2. Hence for n N ,


|xn x| 6 |xn xN | + |xN x| < /2 + /2 = .
Thus {xn } converges to x.

A subspace of a complete metric space may not be complete. However, the


following holds true.
Theorem 4.7. If (X, d) is a complete metric space and Y is a closed subspace of X, then (Y, d) is complete.
Proof. Let {xn } be a Cauchy sequence of points in Y . Then {x n } also
satisfies the Cauchy condition in X, and since (X, d) is complete, there
exists x X such that xn x. But Y is also closed, so x Y showing that
Y is complete.

Theorem 4.8. If (X, d) is a metric space, Y X and (Y, d) is complete,
then Y is closed.
Proof. Let {xn } be a sequence of points in Y such that x n x. We have
to show that x Y . Since {xn } converges in X, it satisfies the Cauchy
condition in X and so, it also satisfies the Cauchy condition in Y . Since
(Y, d) is complete, it converges to some point in Y , say to y Y . Since any
sequence can have at most one limit, x = y. So x Y and Y is closed. 
Theorem 4.9. If (Xi , di ) are complete metric spaces for i = 1, . . . , m, then
the product (X, d) is a complete metric space.
Proof. Let xn = (x1n , . . . , xm
n ) and {xn } be a Cauchy sequence in (X, d).
Then for a given > 0 there exists k such that d(x n , xm ) < for all n, m k.
Since
dj (xjn , xjm ) 6 d(xn , xm ) < ,
24

it follows that {xjn } is Cauchy in (Xj , dj ) for j = 1, . . . m. Since (Xj , dj ) is


complete, for j = 1, . . . , m there exists x j Xj such that xjn xj . Then,
in view of Proposition 2.14, xn x, where x = (x1 , . . . , xm ).

Let (X, d) and (Y, d0 ) be metric spaces and let C(X, Y ) be the space of
continuous and bounded functions f : X Y . If Y = R, we abbreviate
C(X, R) by C(X). Consider
(f, g) := sup{d0 (f (x), g(x)) | x X}
for f, g C(X, Y ).
Theorem 4.10. The space (C(X, Y ), ) is a complete metric space if (Y, d 0 )
is complete.
Proof. The verification that is a metric is left as an exercise. Suppose that
Y is complete, and suppose that {fn } is a Cauchy sequence in C(X, Y ).
Then for every x X,
d0 (fn (x), fm (x)) 6 (fn , fm )
so that {fn (x)} is a Cauchy sequence on Y . Hence there exists a point,
denoted by f (x) Y , such that d0 (fn (x), f (x)) 0. In this way we obtain
a function f : X Y which associate with a point x X a point which is
the limit of {fn (x)}. We must check that f is continuous and bounded, and
that (fn , f ) 0. Let x X, and > 0. Then there exists N such that
d0 (f (x), fN (x)) < /3, and an open ball B(x, ) such that d 0 (fN (x), fN (y)) <
/3 for every y B(x, ). It follows that for every y B(x, ),
d0 (f (x), f (y)) 6 d0 (f (x), fN (x)) + d0 (fN (x), fN (y)) + d0 (fN (y), f (y)) < .
Hence f is continuous. Now given > 0, chose n 0 such that (fn , fm ) <
for all n, m n0 . Then for every x X,
d0 (fn (x), f (x)) = lim d0 (fn (x), fm (x)) 6
m

for every n n0 . This says that (fn , f ) 6 for n n0 . It remains to show


that f is bounded. Take x, y X and let N N be such that
d0 (f (x), fN (x)) < 1/2

and d0 (f (y), fN (y)) < 1/2

Note that we can find such N since (fn , f ) 0. Then


d0 (f (x), f (y)) 6 d0 (f (x), fN (x)) + d0 (fN (x), fN (y)) + d0 (fN (y), f (y))
< 1 + d0 (fN (x), fN (y)) 6 1 + diamfN (X).
25

Since x, y X were arbitrary, diamf (X) 6 1 + diamf N (X). Hence f is


bounded. The proof is completed.

Corollary 4.11. The space (C(X), ) is complete.

Structure of complete metric spaces-Baires theorem


Let (X, d) be a metric space. If U and V are open and dense, then U V
is also open and dense. To see that U V is dense, we have to show that
O U V is non-empty for any open set O. Since U is dense, there is
u O U , and since O U is open, B(u, r) O U for some r > 0. Since
V is dense, B(u, r) V 6= so that, 6= B(u, r) V O U V . If U and
V are assumed to be dense but not necessarily open, then the intersection
U V does not have to be dense. For example, let U be the set of rational
numbers and V the set of irrational numbers Q c . Then both sets are dense
in R with the usual metric, however, U V = . Consider,
now a sequence of
T
dense and open sets Un . In general, the intersection n1 Un may be empty.
For example, consider (Q, d) with the usual metric d. Let {q n |n N} be
enumeration of rational numbers, and let U n = Q \ {qn }. Then each Un is
open since itTis a complement
set {q n }, and is dense . However,
 of a closed
S
T
Q
\
{q
}
=
Q
\
{q
U
=
n
n1 n } = . The Baire theorem says
n1 n
n1
T
that if (X, d) is complete, then n1 Un is dense.

Theorem 4.12. Let (X, d) be a complete metricTspace, and let {U n} be a


sequence of open and dense subsets of X. Then n1 Un is dense.
T
Proof. It suffices to show that B(x, r) contains a point belonging to n1 Un
for any open ball B(x, r). Since U1 is open and dense, B(x, r) U1 is nonempty and open. So, there exists an open ball B(x 1 , R) with R < 1 such
that B(x1 , R) B(x, r) and B(x1 , R) U1 . Taking r1 < R, we get that
B(x1 , r1 ) B(x, r) and B(x1 , r1 ) U1 Similarly, since U2 is open and
dense, there exists x2 and r2 < 1/2 such that B(x2 , r2 ) B(x1 , r1 ) U2 .
Continuing in this way we find a sequence of balls B(x n , rn ) with rn < 1/n
and B(xn+1 , rn+1 ) B(xn , rn ) Un . We claim that {xn } is Cauchy. By
construction, Bn (xn , rn ) B(xk , rk ) for all n k. Given > 0 choose
k N so that 1/k < /2. Then, if n, m k,
d(xn , xm ) 6 d(xn , xk ) + d(xk , xm ) < 1/k + 1/k < .
Because (X, d) is complete, {xn } converges, say to y. The point y lies in
all balls B(xk , rk ) since xn B(xk , rk ) for all n k and B(xk , rk ) is closed
for all k, so that after taking a limit as n , y B(x k , rk ) for all k. In
26

particular, y B(x1 , r1 ) T
B(x, r) and y B(xn+1 , rn+1 ) Un for all n.
Consequently, y B(x, r) n1 Un , and the proof is finished.


As a consequence we obtain the following theorem.

Theorem 4.13. If (X, d) is a complete


S metric space and {F n } is a sequence
of nowhere dense subsets of X, then Fn has empty interior.
S
Proof. Arguing
by
contradiction
assume
that
Fn has non-empty interior.
S
So B(x, r) Fn for some x and r > 0. Define Un = X \ Fn . Clearly, Un
is open and we claim that it is dense. Indeed, if for some open set V , we
have V Un = , then V X \ Un = Fn contradicting
T that Fn has empty
interior. Consequently, in view of the above theorem, n1 Un is dense. So
T
S
S
B(x, r) n1 Un 6= . On the other hand, B(x, r) Fn Fn so that



S
T 
T
= B(x, r) X \ n1 Fn = B(x, r) n1 X \ Fn = B(x, r) n1 Un ,
contradiction.

Example 4.14. The metric space R with the standard metric space cannot
be written as a countable union of nowhere sets since it is complate. By
contrast, Q with the standard metric can be written as the union of one
point sets {qn }, where {qn |n N} is an enumeration of Q. Every one point
set {qn } is closed in Q and its interior is empty, so nowhere dense. This
does not contradict Baires theorem since Q with the standard metric is not
complete.

Applications
Theorem 4.15. Let (X, d) be a complete metric space, and let {f n } be a
sequence of continuous functions f n : X R. Assume that the sequence
{fn (x)} is bounded for every x X. Then there exists a non-empty open set
U X on which the sequence {fn } is bounded, that is, there is a constant
M such that |fn (x)| 6 M for all x U and all n N.
Proof. Since the function fn is continuous, the set fn1 ([m, m]) = {x X |
|fn (x)| 6 m} is closed for any pair of positive integers n and m. Thus,
\
Em = {x X | |fn (x)| 6 m for all n N} =
f 1 ([m, m])
n

is closed for every m N. If x is any point in X, then |f n (x)|


S 6 k for some
k N and all n because {fn (x)} is bounded. Hence X = m Em . In view
27

of the Baire theorem, one of the sets E m has non-empty interior, say Em .
the conclusion follows.
Setting U = Em

Theorem 4.16. There exists continuous function f : [0, 1] R which is
not differentiable at every point point x [0, 1).
Proof. Recall that f has a right-hand derivative at x, if
lim [(f (x + h) f (x))/h] exists.

h0+

We denote this limit by f+0 (x). In particular, if f is differentiable at x [0, 1)


then f+0 (x) exists and is equal to f 0 (x). Consider the complete metric space
C([0, 1], R) with a metric d given by
d(f, g) = sup{|f (x) g(x)||x [0, 1]}.
Let
M = {f C([0, 1], R) | exists x [0, 1) such that f +0 (x) exists}
and let Mm , for m 2, be the set of all f C([0, 1], R) for which exists
some x [0, 1 1/m] such that
|f (x + h) f (x )| 6 m h for all h [0, 1/m].
S
Claim 1: M n2 Mm . Let f M . Then there exists x [0, 1) such
that f+0 (x) exists. We will show that |f (x + h) f (x)| 6 m h for some
m N and all 0 6 0 6 1/m. Since
lim

h0+

we have

f (x + h) f (x)
= f+0 (x),
h



f (x + h) f (x)

= |f+0 (x)|.
lim

h
h0+

(1)

Take an integer k 2 such that |f+0 (x)| 6 k and x [0, 1 1/k]. In view of
(1), there exists 0 < < 1/k such that
|f (x + h) f (x)| 6 k h for all 0 6 h 6 .
Since f is continuous on a closed and bounded interval, there is C > 0
such that |f (x)| 6 C for all x [0, 1] (this is proved in the section on
28

compactness). Let k 0 be any integer so that 2C/ < k 0 . Then, for 6 h 6 1


such that x + h 6 1,
|f (x + h) f (x)| 6 |f (x + h)| + |f (x)| 6 2C =

2C
2C
6
h 6 k 0 h.

Taking m = max{k, k 0 }, we have x [0, 11/m] and |f (x+h)f (x)| 6 mh


for all h [0, 1/m], so that f M. .
Claim 2: Mm is closed for all m 2. To see this, take f M m . We
will show that f Mm , that is, |f (x + h) f (x)| 6 m h for some x
[0, 11/m] and all h [0, 1/m]. There exists (f k ) Mm such that d(fk , f ) =
sup|fk (x) f (x)|| x [0, 1]} 0 as k . Since f k Mm , there exists
xk [0, 1 1/m] such that
|fk (xk + h) fk (xk )| 6 m h

(2)

for all h [0, 1/m]. Since {xk } [0, 1 1/m], there exists a subsequence
which converges to some point x [0, 1 1/m] . Without loss of generality
we may assume that xk x [0, 11/m]. Hence, by the triangle inequality
and by (2),
|f (x + h) f (x)| 6 |f (x + h) f (xk + h)| + |f (xk + h) fk (xk + h)|

+ |fk (xk + h) fk (xk )| + |fk (xk ) fk (x)| + |fk (x) f (x)|

6 |f (x + h) f (xk + h)| + d(fk , f ) + m h


+ |fk (xk ) fk (x)| + d(fk , f )

for all 0 6 h 6 1/m. Since d(fk , f ) 0, and |f (x + h) f (xk + h)| 0,


and |f (x) f (xk )| 0, as k , we get that
|f (x + h) f (x)| 6 m h
for all 0 6 h 6 1/m. Consequently, f M m and Mm is closed.
= . Let f M , and let > 0. Then there exists a piecewise
Claim 3: Mm
m
linear function g : [0, 1] R such that d(f, g) = sup{|f (x) g(x)| | 0 6
0 (x)| > m for all x [0, 1]. That is, g B(f, ) and
x 6 1} < and |g+
g 6 Mm . (Here B(f, ) is a ball in C([0, 1], R) with centre at f and radius
= .
). So Mm
S
In
S view of the Baires theorem, C([0, 1], R) 6= m2 Mm since otherwise
has non-empty interior. SHence there exists f C([0, 1], R) so
m2 Mm S
that f 6 m2 Mm . Since M m2 Mm , f 6 M . Since M contains
all functions which are differentiable at least one point in [0, 1), f is not
differentiable at any x [0, 1)

29

graphs
of functions
f and g

Figure 2: The black curve is the graph of f and the grey curve is the graph
of g.

Contraction mapping principle-Banach fixed point theorem


Let (X, d) be a metric space and let f : X X. A point x X is a fixed
point of f if f (x) = x. The solution of many classes of equations can be
regarded as fixed points of appropriate functions. In this section we give
conditions that guarantee the existence of fixed points of certain functions.
A function f : X X is called a contraction if there exists (0, 1) such
that
d(f (x), f (y)) 6 d(x, y)
(8)
for all x, y X.
Theorem 4.17 (Banach Fixed Point Theorem). Let f : X X be a
contraction of a complete metric space. Then f has a unique fixed point p.
For any x X, define x0 = x and xn+1 = f (xn ) for n 0. Then xn p,
and
d(x, f (x))
.
(9)
d(x, p) 6
1
Proof. We start with the uniqueness of the fixed point of f . Assume that
p 6= q and that f (p) = p and f (q) = q. Then
d(p, q) = d(f (p), f (q)) 6 d(p, q)
so that d(p, q) = 0 since a (0, 1). So p = q, contradicting our assumption.
Hence f has at most one fixed point. Fix any point x X, and let x 0 = x
30

and xn+1 = f (xn ) for n 0. Then for any n,


d(xn+1 , xn ) = d(f (xn ), f (xn1 )) 6 d(xn , xn1 )
and,
d(xn+1 , xn ) 6 d(xn , xn1 ) 6 2 d(xn1 , xn2 ) 6 6 n d(x1 , x0 ).
For m > n,
d(xm , xn ) 6 d(xn , xn+1 ) + d(xn+1 , xn+2 ) + + d(xm1 , xm )

X


i d(x1 , x0 )
6 n + n+1 + + m1 d(x1 , x0 ) 6
i=n

= n

X
i=0


i

d(x1 , x0 ) =

n d(x

1 , x0 )
.
1

Since n 0 as n (recall (0, 1)), the sequence {x n } is Cauchy in


X. Since (X, d) is complete, there exists p X such that x n p. Taking
a limit m in the last inequality we find that
d(p, xn ) 6

n d(x1 , x0 )
.
1

(10)

Thus,
d(f (p), p) 6 d(f (p), xn+1 ) + d(xn+1 , p) = d(f (p), f (xn )) + d(xn+1 , p)
6 d(p, xn ) + d(xn+1 , p) 6
= n

n d(x1 , x0 ) n+1 d(x1 , x0 )


+
1
1

(1 + )d(x1 , x0 )
0,
1

and therefore p = f (p). The inequality (9) follows from (10) by taking
n = 0.

Here is an application of Banach fixed point theorem to the local existence
of solutions of ordinary differential equations.
Theorem 4.18 (Picards Theorem). Let U be an open subset of R 2 and
let f : U R be a continuous function which satisfies the Lipschitz condition
with respect to the second variable, that is,
|f (x, y1 ) f (x, y2 )| 6 |y1 y2 |
31

for all (x, y1 ), (x, y2 ) U , and some > 0. Then for a given (x 0 , y0 ) U
there is > 0 so that the differential equation
y 0 (x) = f (x, y(x))
has a unique solution y : [x0 , x0 + ] R such that y(x0 ) = y0 .
Proof. Note that it is enough to show that there are > 0 and a unique
function y : [x0 , x0 + ] R such that
Z x
y(x) = y0 +
f (t, y(t))dt.
x0

Fix (x0 , y0 ) U , then there exists > 0 and b > 0 such that if I =
[x0 .x0 +] and J = [y0 b, y0 +b], then IJ U . Since f is continuous and
I J is closed and bounded, f is bounded on I J. That is, |f (x, y)| 6 M
for some M and all (x, y) U . Take smaller so that < 1 and M < b.
Denote by X the set of all continuous functions g : I J. The set X with
the metric (g, h) = sup{|g(x) h(x)|, x I} is a complete metric space.
For g X, let
Z
x

(T g)(x) = y0 +

f (t, g(t))dt.

x0

Then T g : I R is continuous since if x 1 , x2 I and x2 > x1 , then


Z x2
Z x2


|f (t, g(t))|dt 6 M |x2 x1 |.
|(T g)(x2 ) (T g)(x1 )| =
f (t, g(t))dt 6
x1

x1

For x0 6 x 6 x0 + ,
Z x
Z



|(T g)(x) y0 | =
f (t, g(t))dt 6
x0

x0

|f (t, g(t))|dt 6 M |x x0 | < M < b

The same inequality holds for x0 6 x 6 x0 , and so T g X for any


g X. Since f is Lipschitz with respect to the second variable, we obtain
for g, h X and x [x0 , x0 + ],
Z x




|(T g)(x) (T h)(x)| =
[f (t, g(t)) f (t, h(t))] dt
Z xx0
|f (t, g(t)) f (t, h(t))|dt
6
x0

6 |x x0 |d(g, h) < d(g, h).


32

Similarly, |(T g)(x) (T h)(x)| 6 |x x 0 |d(g, h) < d(g, h) for x [x0


, x0 ]. Since < 1, T is a contraction and in view of Banachs fixed point
theorem there exists a unique continuous function y : I J such that
Z x
y(x) = (T y)(x) = y0 +
f (t, y(t))dt.
x0

Completions
The space (0, 1) with the usual metric is not complete but is a subspace
of the complete metric space [0, 1] with the usual metric. This example
illustrates the general situation: every metric space X may be regarded as
e
e in such a way that X = X.
a subspace of a complete metric space X
We will need the following concept.

Definition 4.19. A bijective map f from (X, d) onto (Y, ) is called an


isometry if
(f (x), f (y)) = d(x, y)

for all x, y X.

If f : X Y is an isometry, then f 1 : Y X is also an isometry, and


the spaces (X, d) and (Y, ) are called isometric. Two isometric spaces can
be regarded as indistinguishable for all practical purposes that involve only
distance.
Definition 4.20. A completion of a metric space (X, d) is a pair consiste and an isometry : X (X) such
e d)
ing of a complete metric space (X,
e
that (X) is dense in X.

Theorem 4.21. Let (X, d) be a metric space. Then (X, d) has a completion. The completion is unique in the following sense: If ((X 1 , d1 ), 1 ) and
((X2 , d2 ), 2 ) are completions of (X, d), then (X1 , d1 ) and (X2 , d2 ) are isometric. That is, there exists an isometry : X 1 X2 such that 1 = 2 .
Proof.
Existence: Let B(X) be the space of bounded functions defined on X
equipped with the uniform norm (f, g) = supyX |f (y) g(y)|. Fix a point
a X. With every x X we associate a function f x : X R defined by
fx (y) = d(y, x) d(y, a),
33

y X.

We have
|fx (y)| = |d(y, x) d(y, a)| 6 d(x, a)
so that fx is bounded. Since
|fx1 (y) fx2 (y)| 6 d(x1 , x2 )

for all y X,

(fx1 , fx2 ) = supyX {|fx1 (y) fx2 (y)|} 6 d(x1 , x2 ). On the other hand,
(fx1 , fx2 ) |fx1 (x2 ) fx2 (x2 )| = d(x1 , x2 ).
Hence
(fx1 , fx2 ) = d(x1 , x2 ),
and the map : X C(X, R) defined by (x) = f x is an isometry onto
(X),
((x1 ), (x2 )) = d(x1 , x2 ).
Denote by X 0 the closure of (X) in B(X) and let d0 be the metric on X 0
induced by . Since (B(X), ) is complete and X 0 is closed in B(X), the
space (X 0 d0 ) is complete.
Uniqueness:
The isometry 1 : X 1 (X) has an inverse 1
1 : 1 (X) X. Then
is
an
isometry
from

(X)
onto
X
.
Since 1 (X) is dense in
2 1
1
2
1
1
(X1 , d1 ), 2 1 extends to the map : X1 X2 satisfying
d2 ((x), (y)) = d1 (x, y),

x, y X1 .

Since X1 is complete, in view of the above equation, (X 1 ) is closed in


X2 . Since 1 = 2 , 2 (X) (X1 ). This implies that X2 = 2 (X)
(X1 ) = (X1 ) since (X1 ) is closed in X2 . Consequently, (X1 ) = X2 ,
i.e., is surjective and the proof is completed.


Compact Metric Spaces

We start with the classical theorem of Bolzano-Weierstrass.


Theorem 5.1 (Bolzano-Weierstrass). Let I be a closed and bounded interval of R, and let {xn } be a sequence in I. Then there exists a subsequence
{xnk } which convereges to a point in I.

34

Proof. Without loss of generality we may assume that I = [0, 1]. Bisect the
interval [0, 1] and consider the two intervals [0, 1/2] and [1/2, 0]. One of these
subintervals must contain xn for infinitely many n. Call this subinterval I 1 .
Now bisect I1 . Again, one the two subintervals contains x n for infinitely
many n. Denote this subinterval I2 the interval containing xn for infinitely
many n. Proceeding in this way we find a sequence of closed intervals I n ,
each one contained in the preceding one, each one half of the length of the
preceding one, and each containing x n for infinitely many n. Choose an
integer n1 so that xn1 I1 . Then choose n2 > n1 such that xn2 I2 .
Then choose n3 > n2 such that xn3 I3 , and so on. Continuing this way
we choose we find a sequence {xnk } such that xnk Ik . If i, j k, then
xni , xnj Ik and so
|xni xnj | 6 1/2k .
Hence {xnk } is Cauchy and since [0, 1] is complete, {x nk } converges to a
point in [0, 1]


Definition 5.2. A metric space (X, d) is called compact if every sequence


in X has a convergent subsequence. A subset Y of X is compact if every
sequence in Y has a subsequence converging to a point in Y .
Proposition 5.3. Let (X, d) be compact and Y a closed subset of X. Then
Y is compact.
Proof. Let {xn } be a sequence in Y . Since X is compact, the sequence {x n }

has a converging subsequence, say x nk x. Since Y is closed, x Y .
Proposition 5.4. Let X be a metric space and Y a compact subset of X.
Then Y is closed and bounded.
Proof. Take any x Y . There exists a sequence {x n } in Y converging to
x. Since Y is compact, the sequence {x n } has a converging subsequence,
say xnk y with y Y . In view of the uniqueness of the limit, y = x.
Hence Y is closed. To see that Y is bounded, we argue by contradiction and
construct a sequence {xn } which does not have a converging subsequence.
Fix any point y X. For every n N, there exists a point x n Y so that
d(xn , y) n since otherwise Y B(y, n) for some n. The sequence {x n }
contains converging subsequence since Y is compact. Say x nk x Y .
Let = d(x, y), Then d(xnk , x) 6 1 for all k N . Hence by the triangle
inequality,
d(x, y) d(y, xnk ) d(x, xnk ) nk 1 k 1
for all k N , contradiction. Consequently, Y is bounded.
35

Combining Proposition 5.4 with Theorem 5.1 we get


Theorem 5.5. A subset Y of R is compact if and only if Y is bounded and
closed.
The result is also valid in Rn with the standard metric. A subset of R n is
compact if and only if it is bounded and closed. This follows from the fact
that if Ai is a compact subset of (Xi , di ) for 1 6 i 6 n, then A1 A2 An
is compact in the product space X1 X2 Xn . In particular, using
Theorem 5.1, [a, a]n is compact in Rn . So if A is bounded and closed in
Rn , then A is a subset of a compact set [a, a] n , and then Proposition 5.3
implies that A is compact.
Theorem 5.5 does not hold true for general metric spaces.
Example 5.6. Consider the metric space ((C([0, 1], R), d) consisting of
all continuous functions on the interval [0, 1] with the supremum metric
d(f, g) = sup{|f (x)g(x)||x [0, 1]}. Let A = {f 1 , f2 , . . .}, where fi (x) = xi
for x [0, 1]. The set A is bounded since B(0, 2). For k > i, we have
|fk (x) fi (x)| = xi |xki 1|.
Let i be fixed. Then for x close to 1, xi > 1/2 and for k large xki < 1/2.
Hence
|fk (x) fi (x)| = xi |xki 1| > 1/4,
So d(fi , fk ) 1/4 for k large. Let f A. We claim that f A. Indeed,
there exists a sequence {gk } A such that d(gk , f ) 0. Hence {gk } is
Cauchy and there is N such that d(gN , gk ) < 1/4 for all k N . Since gk A,
gk = fnk . Hence d(fnN , fnk ) < 1/4 for all k N . From d(fi , fk ) 1/4 for
all k large, it follows that the set {n k } is bounded, that is, nk 6 m for some
m N and all k N . Hence for all k, gk {f1 , f2 , f3 , . . . , fm } so that
the sequence {gk } has a constant subsequence, say gnl = fi for some i 6 m
and all l. Since a subsequence of a convergent sequence converges to the
same limit, the sequence {gk } converges to fi , that is, f = fi . Hence A is
closed. To see that A is not compact, consider a sequence {f n }. If A were
compact, then a subsequence of {fn } converges to some fi A. But then
d(fi , fnk ) < 1/4 for large k contradicting d(f i , fk ) 1/4 for large k.
Theorem 5.7. Let (X, d) and (Y, d0 ) be metric spaces and let f : X Y be
continuous. If a subset K X is compact, then f (K) is compact in (Y, d 0 ).
In particular, if (X, d) is compact, then f (X) is compact in Y .
36

Proof. Let {yn } be any sequence in f (K), and let {xn } be a sequence in K
of points such that f (xn ) = yn . Since K is compact, {xn } has a converging
subsequence to a point in K, say xnk x with x K. Since f is continuous,
f (xnk ) f (x). That is, ynk f (x) and since f (x) f (K), f (K) is
compact.

As a corollary we get
Corollary 5.8. Let f : X R be a continuous function on a compact
metric space. Then f attains a maximum and a minimum value, that is,
there exist a and b X such that f (a) = inf{f (x)| x X} and f (b) =
sup{f (x)| x X}.
Proof. By Theorem 5.7, f (X) is compact and so, it is bounded and the
sup{f (x)| x X} is finite. Set C = sup{f (x)| x X}. By definition of
supremum, for every n N, there exists x n such that C 1/n 6 f (xn ) 6
C. The sequence {xn } has a converging subsequence, xnk b because
X is compact. In view of the continuity of f , f (x nk ) f (x), and since
C 1/n 6 f (xn ) 6 C, f (x) = C. Similarly, f (a) = inf{f (x)| x X}.

Theorem 5.9. Suppose f : (X, d) (Y, d 0 ) is a continuous mapping defined
on a compact metric space X. Then f is uniformly continuous.
Proof. Suppose not. Then there is some > 0 such that for all > 0 there
exist points x, y with d(x, y) < but d 0 (f (x), f (y)) . Take = 1/n
and let xn , yn be points such that d(xn , yn ) < 1/n but d0 (f (xn ), f (yn )) .
Compactness of X implies that there is a subsequence {x nk } converging to
some point x X. Since d(xnk , ynk ) < 1/nk 0 as k , the sequence
{ynk } converges to the same point x. Continuity of f implies that the
sequences {f (xnk )}, {f (ynk )} converge to f (x). Then d0 (f (xnk ), f (x)) < /2
and d0 (f (ynk ), f (x)) < /2 for k large, and so,
d0 (f (xnk , f (ynk )) 6 d0 (f (xnk ), f (x)) + d0 (f (x), f (ynk )) <
for k large, contradiction that d0 (f (xn ), f (yn )) for all n.

Characterization of Compactness for Metric Spaces


Definition 5.10. Let (X, d) be a metric space
and let A X. If {U i }iI
S
is a family of subsets of X such that A iI Ui , then it is called a cover
of A, and A is said to be covered by the U i s. If
S each Ui is open, then
{Ui }iI is an open cover. If J I and still A iJ Ui , then {Ui }iJ is
a subcover.
37

Definition 5.11. Let (X, d) be a metric space and let A X. Then A has
the Heine-Borel property if S
for every open cover {U i }iI of A, there is a
finite set F I such that A iS Ui .

Example 5.12. Consider a set X with a discrete metric. Then every onepoint set is open and the collection of all one-point sets is an open cover of
X. Clearly, this cover does not have any proper subcover. Hence, a discrete
metric space X has the Heine-Borel property if and only if X consists of a
finite number of points.

Definition 5.13. Let (X, d) be a metric space and


S A X. Let > 0. A
finite subset S is called an -net for A if A xS B(x, ). A set A is
called totally bounded if, for every > 0, there is anS-net for A. That
is, for every > 0, there is a finite set S such that A xS B(x, ).
S
Every totally bounded set is bounded, for if x, y ni=1 B(xi , ), say x
B(x1 , ), y B(x2 , ), then
d(x, y) 6 d(x, x1 ) + d(x1 , x2 ) + d(x2 , y) 6 2 + max{d(xi , xj )| 1 6 i, j 6 n}.
The converse is in general false.
Example 5.14. Consider (R, d) with d(x, y) = min{|x y|, 1}. Then (R, d)
is bounded since d(x, y) 6 1 for all x, y R. But (R, d) is not totally
bounded since it cannot be covered by a finite number of balls of radius 1/2.
Indeed, let S be any finite subset of R, and let x be the largest number in
S. If y S, then d(x + 1, y) = min{|x + 1 y|, 1} = 1 and so there is no
1/2-net for R.

Theorem 5.15. Let A be a subset of a metric space (X, d). Then the
following conditions are equivalent:
(a) A is compact.
(b) A is complete and totally bounded.
(c) A has the Heine-Borel property.
Proof. We will show that (a) implies (b), (b) implies (c), (c) implies (a).
(a) implies (b):
Let {xn } be a Cauchy sequence in A. We have to show that it converges to
38

a point in A. By compactness of A, some subsequence, {x nk }, converges to


x A. Then xn x. Indeed, let > 0. Choose n0 such that d(xn , xm ) <
/2 for all n, m n0 . Also choose k0 such that d(xnk , x) < /2 for all
k k0 . Then if k k0 , is such that nk n0 , then for m n0 we have,
d(xm , x) 6 d(xm , xnk ) + d(xnk , x) < /2 + /2 = .
Hence we proved that A is complete.
Suppose that A is not totally bounded. Then there exists r so that A
cannot be covered by finitely many balls of radius r. We construct a sequence {xn } in A which does not have a converging subsequence. Take any
x1 A. Since B(x1 , r) does not cover A, there is a point in A \ B(x 1 , r).
Call this point x2 . Having
chosen points x1 , . . . ,xn , we choose xn+1 so
S
that it belongs to X \ ni=1 B(xi , r). This is possible since A is not covered
by B(x1 , r), . . . , B(xn , r). Continuing in this way we get a sequence {x n }
such that d(xn , xm ) r for all n and m. Such a sequence cannot have
a convergent subsequence since if {x nk } converges, then it is Cauchy and
d(xnk , xnm ) < r for large k and m. Hence A is no compact, contradiction.
(b) implies (c):
Let U = {Ui }iI be a collection of open sets covering A. Arguing by contradiction we assume that U does not contain a finite subcover. Total boundedness of A implies that there is a finite set of closed balls B 1 , . . . ,Bn of
radius 1 which cover A. If each of the sets A B i can be covered by a finite
number of sets from U, then A can also be covered by a finite subcollection of sets from U. Therefore some A B i , , denoted by B 1 , cannot be
covered by a finite number of sets from U. Since B 1 is a subset of A and
1 be a finite
A is totally bounded, B 1 is totally bounded. So let B11 , . . . , Bm
1
i
set of closed balls of radius 1/2 which cover B . If each Bi B 1 can be
finitely covered by sets from U, the same is true for B 1 . Therefore, some
Bj1 B 1 , denoted by B 2 , cannot be covered by a finite number of sets from
U. Continuing in this way we obtain a sequence of closed sets B n such that
B n B n1 B 1 , none of which can be finitely covered and
diam B n 6 1/n. From each B n choose a point xn . The sequence {xn } is
Cauchy since for n, m k, xn , xn Bk and
d(xn , xm ) 6 diam B k 6 1/k.
By completness of A, the sequence {x n } converges, say xn x. In fact,
x B k for all k since xn B k for all n k and since B k is closed. In
particular, x A . Since U covers A, the point x belongs to some U i , and
39

therefore, B(x, ) Ui for some . If y B n , then


d(x, y) 6 d(x, xn ) + d(xn , y) 6 d(x, xn ) + diam B n 6 d(x, xn ) + 1/n.
For large n, the right side is less than . So for large n, B n B(x, ). Hence
B n Ui which shows that B n can be finitely covered by sets from U. This
contradiction show that A has the Heine-Borel property.
(c) implies (a):
Suppose that A is not compact. Then there exists a sequence {x n } in A
with no convergent subsequence in A. Then for every x A, there exists
a ball B(x, x ) which contains xn for at most finitely many n. Otherwise,
there exists x such that for every r > 0, B(x, r) contains x n for infinitely
many n. Then, in particular, for every k, B(x, 1/k) contains x n for infinitely
many n. Choose n1 so that xn1 B(x, 1). Since B(x, 1/2) contains x n for
infinitely many n, there is n2 > n1 such that xn2 B(x, 1/2). In this
way we construct a subsequence {xnk } such that xnk B(x, 1/k). This
implies xnk x contradicting our assumption on {x n }. Now the family
{B(x, x )}xA is an open cover of A from which it is impossible to choose
a finite number of balls which will cover A since any finite cover by these
balls contains xn for finitely many n and since A contains x n for all positive
integers. Consequently, A is compact.


Topological Spaces

Our next aim is to push the process of abstraction a little further and define
spaces without distances in which continuous functions still make sense.
The motivation behind the definition is the criterion of continuity in terms
of open sets. This criterion tells us that a function between metric spaces
is continuous provided that the preimage of an open set is open. We make
the following definition.
Definition 6.1. Let X be a non-empty set. A topology on a set X is a
collection T of subsets of X satisfying the following properties:
O1 and X T ;
O2 if {Ui }iI T , then

Ui T ;
T
O3 if U1 , U2 , . . . , Un T , then ni=1 T ;
iI

The pair (X, T ) is called a topological space. If X is a topological space


with topology T , we say that a subset U of X is an open set in X if U T .
40

Here are some examples of topological spaces.


Example 6.2. Let (X, d) be a metric space. Then the family of open subsets of X with respect to the metric d is a topology on X.

Example 6.3. Let X be any non-empty set. The collection of all subsets of
X, P(X), is a topology on X. This topology is called the discrete topology. Every subset U of X is an open set. On the other extreme, consider
X and the collection {, X}. It is also a topology on X, and is called the
indiscrete topology or the trivial topology.

Example 6.4. Let X = R and let Tu be a collection of subsets of X consisting of , R, and the unbounded open intervals (, a) for all a R. Then
Tu is a topology on R. Similarly, we can define a topology T l consisting of
, R and all unbounded intervals (a, ), a R.
Example 6.5. Let (X, T ) be a topological space and Y X. Then
TY = {U Y | U T } is a topology on Y . It is called the subspace
topology or relative topology induced by T .
Definition 6.6. Suppose that T and T 0 are two topologies on X. If T T 0
we say that T 0 is finer or larger than T . In this case we also say T is
coarser or smaller than T 0 . Topologies T and T 0 are comparable if
T 0 T and T T 0
Along with a concept of open sets there is the companion concept of closed
set. If X is a topological space, then a set F X is closed if F c = X \ F
is open. By de Morgans laws, the family of closed sets is closed under
arbitrary intersection of closed sets and finite unions. More precisely, the
class of closed sets has the following properties:
C1 X and are closed;

T
C2 If Fi is a closed set for every i I, then iI Fi is closed;
S
C3 If F1 , . . . Fn are closed, then ni=1 Fi is closed.
41

Given a subset A of a topological space X, its closure is the intersection


of all closed subsets of X containing A. The closure of A is denoted by A.
The interior of A, denoted by A , is the union of all open subsets of A.
If x X, then a set A X is called a neighbourhood of x, if x A .
Basis
If X is a topological space with topology T , then a basis for T is a collection
B T such that every member of T , i.e., every open set, is a union of
elements of B.
Example 6.7. The collection of all open balls forms a basis for the topology
of metric space.

Theorem 6.8. Let X be a set. Then a collection B of subsets of X is a


basis for a topology of X is and only if B has the following two properties:
(1) For every x X, there exists B B such that x B.
(2) If B1 , B2 B and x B1 B2 , then there exists B3 B such that
x B 3 B1 B2 .

B1

x
B3
B2

PSfrag replacements

Proof. Any basis satisfies (1) since the whole space X is open, and (2) since
the intersection of two open sets B1 B2 is open. Conversely, assume that
B is a collection of subsets of X with properties (1) and (2). Define T to be
42

the collection of all subsets of X that are unions of sets in B. We shall show
that T is topology. The condition (1) guarantees that X T . Clearly, an
arbitrary union of sets in T belongs to T in view of definition of T . Assume
that U, V T . We have to show that U V is the union of sets in B. Take
any x U V . Since U and V are unions of sets in B, there exist B 1 , B2 B
such that x B1 U and x B2 V . So x B1 B2 , and, in view of
(2), there exists Bx B such that x B3 B1 B2 . Hence Bx U V ,
and consequently,
[
U V =
Bx .
xU V

This shows that U V T .

Hausdorff and normal spaces


Definition 6.9. A topological space X is called a Hausdorff space if for
every two points x, y X such that x 6= y, there exist disjoint open sets U
and V satisfying x U and y V . A space X is normal if for each pair
A, B of disjoint closed subsets of X, there exist disjoint open sets U and V
such that A U and V V .
PSfrag replacements
U
V
U
V
x

Continuity
Continuous functions in metric spaces were characterized in terms of open
and closed sets (see Theorem 3.4 and Theorem 3.5).This suggest the definition of continuity in topological spaces.
Definition 6.10. Let X and Y be topological spaces and let f : X Y .
The map f is continuous at a point x0 if for every neighbourhood U of
f (x0 ) in Y there exists a neighbourhood V of x 0 in X such that f (V ) U .
Global continuity of f is defined in terms of open sets: f is continuous if
f 1 (U ) is open in X for every open set U in Y .

43

If f : X Y is bijective and f and f 1 are both continuous, f is called a


homeomorphism and X and Y are said to be homeomorphic. We call
a property topological if it is invariant under homeomorphism.

Elementary properties of continuous functions


(1) If f : X Y and g : Y Z are continuous maps between topological
spaces, then the composition g f : X Z is continuous.
(2) If f : X R and g : X R are continuous, then h : X R 2 given
by h(x) = (f (x), g(x)) is continuous.
(3) If A is a subspace of X, then the inclusion map i : A X is continuous
(this follows from the definition of the topology on the subspace A). If
f : X Y is continuous, where Y is another topological space, then
the restriction map h : A Y defined by h(x) = f (x) for x A, is
continuous. This follows from (1) using the fact that h = f i.

Compact Topological Spaces

Theorem 4.15 gives three equivalent characterizations of compactness for


metric spaces: the Bolzano-Weierstrass property, completeness together with
total boundedness and the Heine-Borel property. In the case of general
topological spaces the most useful is the Heine-Borel property. A subset
Y of a topological space (X, T ) is called compact
it if for every collection
S
U = {Ui }iISof open sets such that A = iI Ui , there is a finite J I for
which Y iJ Ui . DeMorgans laws lead to the following characterization
of compactness in terms of closed sets.
Definition 7.1. A family {Fi }iI T
of closed subsets of X is said to have the
finite intersection property if iJ Fi 6= for all finite J I.
Theorem 7.2. A topological space X has is compact if and only if for every
family
{Fi }iI of closed subsets of X having the finite intersection property,
T
F
iI i 6= .

Proof. Assume that X is compact. Let {F i }iI be a collection of closed sets


having
T the finite intersection property. Arguing by contradiction
S
Sassume
= . Denoting by Ui = X \ Fi we have iI Ui = iI [X \
that iFi T
Fi ] = X \ iI Fi = X. So {Ui }iI is an open cover of X. Hence there
are Ui1 , . . . , Uik such that X = Ui1 Uik . But then = X \ X =
T
S
X \ kl=1 Uil = nl=1 Fil , contradicting the assumption that {F i } has the
44

finite intersection property. Conversely, suppose that for


T every collection
{Fi }iI having the finite intersection property we have iI Fi 6= . Take
any open
define Fi = X \ Ui . Then Fi s are closed
T covaerT{Ui }iI of X, and S
and iI Fi = iI [X \ Ui ] = X \ iITUi = . So {Fi } does nor have the
finite intersection
T property (otherwise iI F
Si 6= ). So there
S is a finite set
J I such that iJ Fi = . But then X = iJ [X \Fi ] = iJ Ui showing
that X is compact.

Theorem 7.3. A closed subspace of a compact topological space is compact
Proof. Let K be a closed subset of a topological space X, and let {U iJ } be
an open cover of X. Then the collection {U } iJ {K c } is a family of open
subsets of X that covers X. Since X is compact, there is a finite subfamily
of this family that covers X. The corresponding subfamily of {U iJ } covers
Y.

Theorem 7.4. If X is a Hausdorff space, then every compact subset of X
is closed.
Proof. Let K be a compact subset of X. Since X is Hausdorff, for every
x K c and every y K, there are disjoint open sets U xy and Vxy such that
x Uxy and y Vxy . Then for every x K c , {Vxy }yK is anSopen cover of
n
K. Since
Tn K is compact, there exist y1 , . . . yn K such K i=1 Vxyi . Setc
U = i=1 Uxyi . Then U is open, U K = , and x U . Thus x U K
showing that K c is open, and consequently, that K is closed.

Theorem 7.5. A compact Hausdorff space is normal
Proof. Let A and B be disjoint closed subsets of a compact Hausdorff space.
In view of Theorem 7.3, the sets A and B are compact. Proceeding like in the
proof of the previous theorem, we find for every x B disjoint open sets V x
and Ux such that x Vx and A Ux . Then the open sets {Vx }xB cover B.
Consequently, there exist x1 , . . . , xn B such that B Vx1 Vxn := V .

Then U := Ux1 Uxn is open, U V = , and A U, B V .
Theorem 7.6. Suppose that f : X Y is a continuous map between
topological spaces X and Y . If K X is a compact set, then f (K) is a
compact subset of Y . In particular, if X is compact, then f (X) is compact.
Proof. Let U be an open cover of f (K). That is, U consists of open subsets
of Y such that their union contains f (K). The continuity of f implies that
for any set U U, f 1 (U ) is an open subset of X. Moreover, the family
45

{f 1 (U ) | U U} is an open cover of K. Indeed, if x K, then f (x)


f (K), and so f (x) SU for some U U. This implies that x f 1 (U ).
S Since
K is compact, K ni=1 f 1 (Ui ) for some n. It follows that f (K) ni=1 Ui
which proves that f (K) is a compact subset of Y . This completes the proof
of the theorem.

Theorem 7.7. Let f be a continuous bijective function from a compact
topological space X to a Hausdorff topological space Y . Then the inverse
function f 1 : Y X is continuous.
Proof. Denote by g = f 1 : Y X. We have to show that g 1 (K) is closed
in Y for any closed set K in X. Since f is a bijection, g 1 (A) = f (A) for
any subset of Y . So g 1 (K) = f (K). Since K is closed and X compact, K
is also compact. By the previous result, f (K) is compact in Y and since Y
is Hausdorff, f (K) is closed. So g 1 (K) is closed in Y , as required.

Example 7.8. Let S 1 be the unit circle in R2 of radius 1 and centre (0, 0).
We consider S 1 as a subspace of R2 . Let f : [0, 2) S 1 be given by
f (x) = (cos x, sin x) for x [0, 2). Show that f is a continuous bijection
but the inverse map f 1 : S 1 [0, 2) is not continuous. Why doesnt this
contradict Theorem 6.7?

Connected Spaces

A pair of non-empty and open sets U , V of a topological space X is called


a separation of X if U V = and X = U V . A topological space X is
called disconnected if there is a separation of X, and otherwise is called
connected. A subset Y of X is said to be connected if it is connected
as a subspace of X, that is, Y is not the union of two non-empty sets
U, V |mathcalT Y such that U V = .
Example 8.1. The set X containing at least two points and considered
with the discrete topology is disconnected, however, X with the indiscrete
topolgy is connected.

Example 8.2. The subspace R \ {0} of R is disconnected since R \ {0} =


A B, where A = {r R | r < 0} and B = {r R | r > 0}. If X = Q is
considered as subspace of R, then X is disconnected since X = A B with
A = Q (, r) and B = Q (r, ), where r is irrational.
46

A 2-valued function is a function from X to {0, 1}, where {0, 1} is considered with discrete topology.
Theorem 8.3. A space X is connected if and only if every 2-valued continuous function on X is constant. Equivalently, X is disconnected if and only
if there exists a 2-valued continuous function from X onto {0, 1}

Proof. Suppose that X is connected and f : X {0, 1} is continuous. Let


A = f 1 ({0}) and B = f 1 ({1}). The sets A, B are open, disjoint and
X = A B. So one of A, B has to be empty. Conversely, assume that every
continuous 2-valued function is constant. Assume that X = A B, A and
B are open, and A B = . Define
(
0
if x A,
f (x) =
1
if x B.
Clearly, the function f is continuous. So f is constant, say f (x) = 0 for all
x X. But then A = X and B = . Hence X is connected as claimed. 
Theorem 8.4. Let f : X Y be a continuous function between spaces X
and Y . If X is connected, then the image f (X) is connected.
Proof. Let g : f (X) {0, 1} be continuous. Then the composition g f :
X {0, 1} is continuous, hence constant since X is connected. Hence g is
constant on f (X) and the result follows in view of Theorem 8.3.

Theorem 8.5. If A is a connected subset of a space X, then A is also
connected.
Proof. Let f : A {0, 1} be continuous. Then f |A is continuous, and so,
f is constant on A. Say f = 0 on A. We claim that f = 0 on A. Suppose
f (x) = 1 for some x A. The set {1} is open in {0, 1} and since f is
continuous f 1 ({1}) is an open subset of A. Thus say f 1 ({1}) = U A
for some open set U in X. This mean that f = 1 on U A. Since x A,
U A 6= , say y U A. Then f (y) = 1 since y U A U A, but
one the other hand f (y) = 0 since f = 0 on A. Therefore, A is connected
as claimed.

Example 8.6. The union of connected subspaces does not have to be connected. Consider R with the usual topology. Then the sets (, 0) and
(0, ) are connected subspaces of R, but the union (, 0)(0, ) = R\{0}
is disconnected.

47

Theorem 8.7. If {Ai }S


iI is a family of connected subsets of X such that
T
A
=
6
,
then
A
=
iI i
iI Ai is connected.

Proof. Let f : A {0, 1} T


be continuous. Then f |Ai is continuous for every
i, so it is constant. Since iI Ai 6= , we must have the same constant on
every Ai . Hence f is constant and A is connected.

As an application of this theorem we have the following
Theorem 8.8. If for any two points in a space X there exists a connected
subspace of X containing these two points, then X is connected.
Proof. Fix a point a X. For b X S
denote by C(b) a connected
subspace
T
of X containing a and b. Then X = bX C(b). Since a bX C(b), the
result follows from the previous theorem.

Let x X and let Cx be the union of all the connected subsets of X
containing x. Each Cx is called a component (or connected component)
of X.
Proposition 8.9. Let Cx be the connected component of X containing x.
Then
(a) for each x X, Cx is connected and closed; and
(b) for any two x, y X, either Cx = Cy or Cx Cy = .
Proof. The set Cx is connected in view of Theorem 8.7, and by Theorem
8.5, Cx is connected. Hence by the definition of C x , Cx Cx , so Cx = Cx
and Cx is closed. If Cx Cy 6= , then Cx Cy is connected by Theorem 8.7.
So again by the definition of Cx Cx Cy Cx . Hence Cy Cx , Similarly,
Cx Cy , so Cx = Cy as required.

Example 8.10. If X is equipped with the discrete topology, then every
subset of X is open and closed. Hence the connected components of X are
sets consisting of one point.
Next we shall determine the connected subsets of R. By an interval I R
we mean a subset of R having the following property: if x, y I and x 6
z 6 y, then z I.
Theorem 8.11. A subset of R is connected if and only if it is an interval.

48

Proof. Suppose that J R is not an interval. Then there are x, y J and


z 6 J with x < z < y. Then define A = (, z) J and B = (z, ) J.
Clearly, A, B are disjoint, non-empty, relatively open, and A B = J. So
J is not connected. Conversely, suppose that J is an interval. We will show
that J is connected. Let f : J {0, 1} be continuous, and suppose that
f is not constant. Then there are x1 and y1 J such that f (x1 ) = 0 and
f (y1 ) = 1. For simplicity assume that x1 < y1 . Let a be the midpoint
of [x1 , y1 ]. If f (a) = 0, then set x2 = 0 and y2 = y1 , and otherwise,
x2 = x1 and y2 = a. So x1 6 x2 6 y2 < y1 , |x2 y2 | 6 21 |x1 y1 |,
and f (xi ) 6= f (yi ). Iterating this procedure we find a sequences {x n } and
{yn } with the following properties: x1 6 x2 6 6 xn < yn 6 6 y1 ,
|xn yn | 6 21 |xn1 yn1 | 6 2n1 |x1 y1 |, and f (xn ) = 0, f (yn ) = 1.
Since R is complete, {xn } converges to some z, and since |xn yn | 0,
yn z. Clearly, z J. Hence 0 = limn f (xn ) = f (z) = limn f (yn ) = 1, a
contradiction. So f is constant, and this implies that J is connected.

We can apply the last theorem to analyze the structure of open subsets of R.
We claim that any open set U R is a countable union of pairwise disjoint
open intervals. Indeed, let x U and let I x be the connected component of
U containing x. Thus, Ix is an interval. If y Ix , then there is > 0 such
that (y , y + ) U since U is open. Hence I x (y , y + ) is connected
and since Ix is a connected component, (y , y + ) I x . So Ix is an open
interval, and U is a union of open intervals (its components). Since each
must contain a different rational number, U is at most countable union of
disjoint open intervals.
Here is an important application of Theorem 8.11.
Theorem 8.12 (Intermediate Value Theorem). Let f be a continuous
function defined on a connected space X. Then for any x, y X and any
r R such that f (x) 6 r 6 f (y) there exists c X such that f (c) = r.
Proof. The set f (X) is a connected subset of R. Hence f (X) is an interval,
and since f (x), f (y) f (X), it has to contain r.

Definition 8.13. A space X is called path connected if for any two points
p and q X, there exists a continuous function f : [0, 1] X such that
f (0) = p and f (1) = q. The function f is called a path from f (0) to f (1).
If X is path connected, then X is connected but the converse is false in
general as the following example shows.

49

Example 8.14. Denote by X = {(t, sin(/t)) | t [0, 2]} R 2 . Let


: R2 R be the projection onto the first coordinate, that is, (x, y) = x.
Then : X (0, 2] is a homeomorphism and since (0, 2] is connected so
is X. Therefore, X = ({0} [1, 1]) X = J X is connected, where we
abbreviated J = {0} [1, 1]. We shall show that X is not path connected.
Arguing by contradiction assume that f : [0, 1] X is a continuous path
in X such that f (0) J and f (1) X. Consider f 1 (J). It is closed in
[0, 1] and contains 0. Let a = sup{t [0, 1], t f 1 (J)}. Since f (1) X,
a < 1. Since f is continuous, there exists > 0 such that f (a + ) X.
Write f (t) = (x(t), y(t)). Then x(a) = 0 and x(t) > 0, y(t) = sin(/x(t))
for t (0, a + ]. For every large n find r n such that 0 < rn < x(a + 1/n)
and sin(/rn ) = (1)n . Since the function x is continuous by the Intermediate Value Theorem there is tn (a, a + ] such that x(tn ) = rn and
y(tn ) = (1)n . So tn a but y(tn ) does not converge contradicting the
fact that f is continuous. Hence X is not path connected.

Product Spaces

We define a topology on a finite product of topological spaces. Consider a


finite collection X1 , . . . , Xn of topological spaces. The product topology
on the product X = X1 Xn is the topology for which a basis of open
sets is given by rectangles
{U1 Un | Uj is open in Xj for 1 6 j 6 n}.

(11)

Observe that the intersection of two such sets is again a set of this form.
Indeed,
(U1 Un ) (V1 Vn ) = (U1 V1 ) (Un Vn )
Consequently, the family (11) forms a basis. Let j : X Xj be the
projection of X onto the jth factor, defined by
j (x1 , . . . , xn ) = xj ,

(x1 , . . . , xn ) X.

For an open set Uj Xj , we have


j1 (Uj ) = X1 Xj1 Uj Xj+1 Xn
which is a basic open set. Hence each projection j is continuous.
50

(U1 V1 ) (U2 V2 )

X2
PSfrag replacements

U2
V2
U1
V1
X1

Theorem 9.1. Let X be the product of the topological spaces X 1 , . . . , Xn ,


and let j be the projection of X onto Xj . The product topology for X is
the smallest topology for which each of the projections j is continuous.
Proof. Let T be another topology on X such that the projections j are
T -continuous. Take open sets Uj Xj , 1 6 j 6 n. Then each j1 (Uj )
belongs to T since j is T -continuous. Since
11 (U1 ) n1 (Un ) = U1 Un
the basic set U1 Un belongs to T and T includes the product topology.

Call a function f from one topological space to another open if it maps
open sets onto open sets.
Theorem 9.2. Let X be the product of the topological spaces X 1 , . . . , Xn .
Then each projection j of X onto Xj is open.
Proof. Let U = U1 Un be a basic open set in X. Then j (U ) = Uj ,
and since the maps preserve unions, the image of any open set is open.

Theorem 9.3. Let Y be a topological space and let f be a continuous map
from Y to the product X = X1 Xn . Then f is continuous if and only
if j f is continuous for all 1 6 j 6 n.
Proof. If f is continuous, the j f is continuous as a composition of continuous maps. Conversely, suppose that j f is continuous for all 1 6 j 6 n.
Take a basic open set U = U1 Un in X. Then
f 1 (U ) = (1 f )1 (U1 )) (n f )1 (Un ))
51

is a finite intersection of open sets and hence is open. Since the inverses of
functions preserve unions, the inverse image of any open set is open, and
consequently, f is continuous.

We next study which properties of topological spaces are valid for the product X = X1 Xm whenever they hold for X1 , X, . . . , Xm .
Theorem 9.4. Let X be the product of Hausdorff spaces X 1 , . . . , Xn . Then
X is Hausdorff.
Proof. Take two different points x = (x 1 , . . . , xn ), y = (y1 , . . . , yn ), and
choose an index i so that xi 6= yi . Since Xi is Hausdorff, there exist open
sets Ui and Vi in Xi such that Ui Vi = . Then i1 (Ui ) and i1 (Vi ) are
open and disjoint sets containing x and y, respectively. Consequently, X is
Hausdorff as required.

Theorem 9.5. Let X be the product of path-connected spaces X 1 , . . . , Xn .
Then X is path-connected.
Proof. Take two points x = (x1 , . . . , xn ), y = (y1 , . . . , yn ) in X. Since each
Xj is path-connected, for each 1 6 j 6 n there exists a path j : [0, 1] Xj
from xj to yj . Define : [0, 1] X by setting
(t) = (i (t), . . . , n (t)),

t [0, 1].

Then is a path connecting x with y. So X is path-connected.

To study connectedness of the product of connected spaces we will need the


following fact. Fix points x2 X2 , . . . xn Xn and define a map h : X1 X
by setting h(x1 ) = (x1 , . . . , xn ). Then h is a homeomorphsim of X1 onto
the slice X1 {x2 } {xn } of X. Indeed, if U = U1 Un is
a basic open set in X, then h1 (U ) = U1 is open so that h is continuous.
Since the inverse of h is equal to 1 |X1 {x2 }{xn } , h1 is continuous and
h is a homeomorphism. Similarly, for each j fixed and fixed points x i Xi ,
i 6= j, the map Xj {x1 } {xj1 } Xj {xj+1 } {xn } is a
homeomorphism.
Theorem 9.6. Let X be the product of connected spaces X 1 , . . . , Xn . Then
X is connected.

52

Proof. We prove the theorem for the product of two connected spaces X 1 and
X2 . We apply Theorem 8.8. Take two points a = (a 1 , a2 ), b = (b1 , b2 ) X
and consider sets C1 = {(x, b2 ) X | x X1 } and C2 = {(a1 , y) X | y
X2 }. By the above remark, the sets C1 , C2 are connected. Then, in view of
Theorem 8.7, C = C1 C2 is connected since C1 C2 = {(a1 , b2 )}. Applying
Theorem 8.8, the space X is connected since a, b C.

To study compactness of the product of compact spaces we need the following lemma.
Lemma 9.7. Let Y be a topological space and let B be a basis for the topology
of Y . If every open cover of Y by sets in B has a finite subcover, then Y is
compact.
Proof. Let {Ui }iI be an open cover of Y . For each y Y , choose V y B
and an index j so that y Vy Uj . The family {Vy }yY forms an open
cover of Y by sets belonging to B. In view of the assumption, there exists a
finite number of the Vy s that cover Y . Since each of these V y s is contained
in at least one of the Uj s, we obtain a finite number of Uj s that cover Y .
Hence Y is compact.

Theorem 9.8 (Tichonoff s Theorem for the finite product). Let X
be the product of compact spaces X1 , . . . , Xn . Then X is compact.
Proof. We consider only the product of two compact spaces X 1 and X2 . let
R be a cover of X1 X2 by basic open sets of the form U V , U open in X 1
and V open in X2 . In view of Lemma 9.7, it is enough to show that R has a
finite subcover. Fix z X2 . The slice X1 {z} is compact. Hence there are
finitely many sets U1 V1 , . . . , Un Vn in R covering the slice X {z}. We
may assume that z Vj for all 1 6 j 6 n. The set V (z) = V1 Vn is
an open set containing z, and the set 21 (V (z)) is covered by sets Uj Vj ,
1 6 j 6 n. The collection {V (z)}zX2 is an open cover of X2 , and since
X2 is compact, X2 = V (z1 ) V (zl ) for some finite number of points
zj X2 . Then X = 21 (V (z1 )) 21 (V (zl )). Each 21 (V (zj )) is
covered by finitely many sets in R. Consequently, X can be covered by
finitely many sets in R, and, in view of Lemma 9.7, X is compact.


53

Compactness in function spaces: Ascoli-Arzela theorem


Next we study compact subsets of the space of continuous functions. Let
X be a compact topological space and (M, ) a complete metric space. By
C(X, M ) we denote the set of all continuous functions from X to M . We
consider C(X, M ) with the metric
d(f, g) = sup{(f (x), g(x)) | x X}
Definition 9.9. Let X be a topological space and (M, ) a compact metric
space, and let F be a family of functions from X to M . The family F is called
equicontinuous at x X if for every > 0 there exists a neighbourhood
U of x such that
(f (y), f (x)) < for all y U and all f F.
The family F is called equicontinuous if it is equicontinuous at each x
X.
Example 9.10. Consider two metric spaces (X, ) and (M, ). Given M >
0 let F be a set of all functions f : X Y such that
(f (x), f (y)) 6 M (x, y)
for all x, y X. Then F is an equicontinuous family of functions. For if
> 0, take U = B(x, /M ). Then if y U and f F, we have
(f (x), f (y)) 6 M (x, y) < M /M = .

Theorem 9.11 (Ascoli-Arzela Theorem). Let X be a compact space and


let (M, ) be a complete metric space. Let F C(X, Y ). Then the closure
F is compact in C(X, M ) if and only if two of the following conditions hold:
(1) F is equicontinuous.
(2) for each x X. the set F(x) = {f (x) | f F} has a compact closure
in M .

54

Proof. Since C(X, M ) is a complete metric space, F is compact if and only


if F is totally bounded. Assume first that the conditions (1) and (2) are
satisfied. In view of the above remark we have to show that F is totally
bounded. Given > 0, for each x X there exists an open neighbourhood
V (x) such that if y V (x), then (f (x), f (y)) < for all f F. Since
{V (x)}xX is an open cover of X and X is compact by assumption, there
exist a finite number of points x1 , . . . , xn such that V (x1 ), . . . , V (xn ) cover
X. The sets F(xj ) are totally bounded in M , hence so is the union S =
F(x1 ) F(xn ). Let {a1 , . . . , am } be an -net for S. For every map
: {1, . . . , n} {1, . . . , m} denote by
B = {f F | (f (xj ), a(j) ) < for all j = 1, . . . , n}.
Observe that there is only a finite number of sets B and every f F
belongs to one of such sets. Moreover, if f, g F, then
(f (y), g(y)) 6 (f (y), f (xk )) + (f (xk ), a(k) )
+ (a(k) , g(xk )) + (g(xk ), g(y))
< + + + = 4
for all y V (xk ). So if f, g B , then d(f, g) < 4. Consequently, the
diameter of B is < 4, and since there are finitely many such B and they
cover F, the set F is totally bounded.
Conversely, assume that F is totally bounded. Note that the mapping :
F M given by (f ) = f (x) is distance decreasing, i.e.,
((f ), (g)) = (f (x), g(x)) 6 d(f, g).
It follows that for every x X, the set F(x) M is totally bounded
and (2) holds. To see that (1) holds, let > 0 and let f 1 , . . . , fn be an
-net of F. Given x X we find open neighbourhood V (x) of x such that
(fj (x), fj (y)) < for all y V (x) and all j = 1, . . . , n. Then if f F
choose an index j so that d(f, fk ) < . It follows that if y V (x), then
(f (x), f (y)) 6 (f (x), fj (x)) + (fj (x), fj (y)) + (fj (y), f (y))
< + + = 3.
Therefore, the family F is equicontinuous at x, and since x was an arbitrary
point of X, F is equicontinuous as required.

Corollary 9.12. Let X be a compact topological space and Y a compact
metric space. Let F be an equicontinuous a of C(X, Y ). Then every sequence
in F has a uniformly convergent subsequence.
55

Definition 9.13. A family F of maps f : X Y , where Y is a metric


space is called pointwise bounded if {f (x) | f F} is bounded in Y for
every x X.
Lemma 9.14. Assume that X is a compact metric space and let F be an
equicontinuous and pointwise bounded family in C(X). Then there is a
constant M such that f (X) [M, M ] for all f F.
Proof. For each x X, there is Mx such that |f (x)| 6 Mx for all f F.
Since F is equicontinuous, for each x there is an open set U x such that
|f (x) f (y)| 6 1 for all f F and y Ux . Then
|f (y)| 6 |f (y) f (x)| + |f (x)| 6 1 + Mx = Kx
for all y Ux . The sets Ux form an open covering of X and since X
is compact, there exists a finite subcovering U x1 , . . . , Uxn . Set now M =
max{Kx1 , . . . , Kxn . Then |f (x)| 6 M for all x M .

Corollary 9.15 (Arzel
a - Ascoli Theorem, classical version). Let X
be a compact topological space. Assume that F is a pointwise bounded and
equicontinuous subset of C(X). Then every sequence in F has a uniformly
convergent subsequence.
Proof. In view of the above exercise the set F is uniformly bounded, that
is, |f (x)| 6 M for all f F and x X. Set Y = [M, M ]. Then Y is
compact in R, and F is a subset of C(X, Y ). So the corollary follows from
Corollary 9.12.


10

Uryshons and Thietzes Theorems

We show the existence of continuous functions on normal topological spaces.


We start with the following characterisation of normal spaces.
Lemma 10.1. A topological space X is normal if and only if for every closed
subset A X and every open subset B X containing A , there exists an
open set U such that A U U B.
Proof. Assume first that X is normal and A and B are as above. Then the
sets A and X \ B are closed and disjoint. So, in view of normality of X,
there exist open disjoint sets U and V such that A U and X \ B V .
Then U X \ V B, so that U has the required properties.
Conversely, let A and B be closed disjoint subsets of X. Then V = X \ B
56

is open and A V . By assumption there exists an open set U such that


A U U V . Then U and X \ U are disjoint open sets satisfying A U
and B X \ U . So X is normal and the proof is completed.

Theorem 10.2 (Urysohns Lemma). Let A and B be closed subspace of
a normal space X. Then we can find a continuous function f : X [0, 1]
such that f (a) = 0 for all a A and f (b) = 1 for all b B.
Proof. For the proof recall that a dyadic rational number is a number which
m
can be written in the form p = n . with n, m being integers. Set V = X \B,
2
an open set which contains A. By Lemma 10.1, there exists an open set U 1/2
such that
A U1/2 U 1/2 V.
Applying Lemma 10.1 again to the open set U 1/2 containing A and to the
open set V containing U 1/2 , we obtain open sets U1/4 and U3/4 such that
A U1/4 U 1/4 U1/2 U 1/2 U3/4 U 3/4 V.
Continuing in this way, we associate to each such number p D an open
subset Up X having the following properties
Up U q ,
A Uq ,
Up V,

0 < p < q < 1,

(12)

0 < p < 1,

(13)

0 < p < 1.

(14)

Next we shall construct the function f which is continuous and such that
the sets Up are level sets of f on which f assumes the value p. Define
f (x) = 0 if x Up for all p > 0 and f (x) = sup{p| x 6 Up } otherwise.
Clearly, 0 6 f 6 1, f (x) = 0 for all x A and f (x) = 1 for all x B. It
remains to show that f is continuous. Take x X. We only consider the
case that 0 < f (x) < 1. (The remaining cases f (x) = 0 and f (x) = 1 are
left as an exercise). Let > 0 and choose dyadic rationals p and q such that
0 < p, q < 1 and
f (x) < p < f (x) < q < f (x) + .
Then x 6 Ur for dyadic rationals r between p and f (x) so that, in view
of (12), x 6 U p . On the other hand x Uq . So W = Uq \ U p is an open
neighbourhood of x. Then p 6 f (y) 6 q for any y W which shows that
|f (x) f (y)| < for all y W . Hence f is continuous and the proof is
completed.

57

Theorem 10.3 (Thietzes extension theorem). Let A be a closed subset


of a normal space X and let f be a bounded continuous real valued function
on A. Then there exists a bounded continuous function h : X R such that
f = h on A.
Proof. Set a0 = sup{|f (a)|| a A}. (Note that a < since f is bounded).
Define sets
B0 = {a A| f (a) 6 a0 /3}

C0 = {a A| f (a) a0 /3}.

Since f is continuous on A and A is closed, the sets B and C are closed and
disjoint subsets of X. Taking a linear combination of constant function and
the function from Uryshons lemma, we find a continuous function g 0 : X
R such that a0 /3 6 g0 6 a0 /3, g0 = a0 /3 on B0 and g0 = a0 /3 on C0 .
In particular,
|g0 | 6 a0 /3

|f g0 | 6 2a0 /3.
Iterating this process we construct the sequence of functions {g n } satisfying
|gn | 6 2n a0 /3n+1

|f g0 g1 gn | 6 2

n+1

a0 /3

(15)
n+1

on A.

(16)

Indeed, suppose that the functions g 0 , . . . , gn1 have been constructed. To


construct gn+1 , set
an1 = sup{|f (a) g0 (a) g1 (a) gn1 (a)|| a A},
and repeat the above argument with a n1 replacing a0 and f g0 g1
gn1 replacing f . This gives the function g n such that
|gn | 6 an1 /3

|f g0 g1 gn | 6 2an1 /3

on A.

Since an1 6 2n a0 /3n , the function gn satisfies (15)-(16). Set


hn = g 0 + + g n ,

n 1.

If n > m, then
 n 
 m+1
a0
2
2

+ +
|hn hm | = |gm+1 + + gn | 6
3
3
3
 m+1
2
6
a0 .
3
58

Consequently, {hn } is Cauchy in C(X, R). Hence there exists a continuous


function h : X R such that hn h. In addition,
|h| = |lim hn | = lim|hn | 6 lim

n
X
k=0

 
a0 X 2 n
|gk | 6
= a0 ,
3
3
k=1

so that h is a bounded. Finally, in view of (16), |f h| = lim|f h n | 6


lim 2n a0 /3n+1 0 on A, so that f = h on A. The proof is completed.

Both theorems are valid in metric spaces as the following theorem shows.
Theorem 10.4. Every metric space is normal.
Proof. Let A and B be disjoint closed subsets of X. Define
f (x) = inf{d(x, a)| a A}
for x X. Observe that f (x) = 0 if and only if x A since A is closed.
The function f is continuous. Similarly, let g(x) = {d(x, b)| b B}. Then g
is continuous and g(x) = 0 if and only if x A. Since A and B are disjoint,
f (x) + g(x) > 0 for all x X. Set
h(x) =

f (x)
,
f (x) + g(x)

xX

Then h is continuous, h(x) = 0 if and only if x A and h(x) = 1 if and only


if x B. Take now U = {x| h(x) > 3/4} and V = {x| h(x) < 1/4}. Clearly,
U V = , U, V are open, and A U , B V , so that X is normal.


11

Appendix

Sets
A set is considered to be a collection of objects. The objects of a set A are
called elements (or members) of A. If x is an element of a set A we write
x A, and if x is not an element of A we write x 6 A. Two sets A and B
are called equal, A = B, if A and B have the same elements. A set A is a
subset of a set B, written A B, if every element of A is also an element
of B. The empty set has no elements; it has the property that it is a
subset of any set, that is, A for any set A. Given two sets A and B we
define:
59

(a) the union A B of A and B as the set


A B = {x | x A or x B};
(b) the intersection A B of A and B as the set
A B = {x | x A and x B};
(c) the set difference A \ B of A and B as the set
A \ B = {x | x A and x 6 B}.
Sets A and B are called disjoint if A B = . The concept of union
and intersection of two sets extends to unions and intersections of arbitrary
families of sets. By a family of sets we mean a nonempty set F whose
elements are sets themselves. If F is a family of sets, then
[
A = {x | x A for some A F}
AF

AF

A = {x | x A for all A F }.

When it is understood that all sets under considerations are subsets of a


fixed set X, then the complement Ac of a set A X is defined by
Ac = X \ A = {x X | x 6 A}.
In this situation we have deMorgans laws:
!c
!c
[
\
\
[
Aci .
=
Aci ,
Ai =
Ai
iI

iI

iI

iI

The set of all subsets of a given set X is called the power set and is denoted
by P(X).
If X and Y are sets, their cartesian product X Y is the set consisting
of ordered pairs (x, y) with x X and y Y .
Given two sets X and Y , a relation from X to Y is subset R of X Y . We
say that R is a relation on X if X X, that is, R X X. Quite often we
write xRy instead of (x, y) R.
The most important example of a relation is a function. A relation f from
X to Y is called a function if for each x X there exists exactly one y Y
60

such that xf y. If xf y, we write y = f (x); y is called the value of f at x.


We also will write f : X Y to mean that f is a function from X to Y .
Here X is called the domain of f , and the set {f (x) | x X} is called the
range of f . If f : X Y is a function, A X and B Y , then the image
of A and the preimage of B under f are sets defined by
f 1 (B) = {x | f (x) B}.

f (A) = {f (x) | x A},

We say that f is injective, or one-one, if f (x) = f (y) only when x = y,


and we say that f is surjective, or onto, if f (X) = Y , that is, if the image
of f is the whole of Y . A function which is both injective and surjective
is called bijective. Sometimes we will use words a map or a mapping
instead of a function. Unions and Intersections behave nicely under inverse
image:
!
[
[
f 1 Ai .
Ai =
f 1
iI

f 1
f

iI

(A ) = f

iI
c

Ai

f 1 Ai .

iI

(A)

c

Given two functions f : X Y and g : Y Z, we define the composition


g f of f and g as the function g f : X Z defined by the equation
g f (x) = g(f (x)). If f : X Y is one-one, then f has the inverse f 1 .
The inverse f 1 is defined on the range f (X) and takes values in X; it is
given by the formula f 1 (y) = x if and only if f (x) = y.

Countable and Uncountable Sets


A set A is called finite if for some n N, there is a bijection f from
{1, . . . , n} to A. The number n is uniquely determined and is called the
cardinality of A. We denote this fact by ]A = n or card(A) = n. If
A is not finite, then it is called infinite. If A is infinite, then there is an
injective function f from the set of natural numbers N into A. If there exists
a bijection between N and A, then we say that X is countably infinite
(or just countable). So A is countably infinite if and only if its elements can
be listed in an infinite sequence X = {x 1 , x2 , . . .}. If there is no bijection
between N and A, then A is called uncountable.
Example 11.1. The set Z of all integers is countably infinite. To see this

61

consider the function f : N Z defined by


(
n/2
if n is even;
f (n) =
(n 1)/2
if n is odd.
Check that the function f is a bijection from N to Z so that Z is countably
infinite.

Example 11.2. Consider the interval I = [0, 1]. Then I is uncountable.


Seeking a contradiction, suppose that I is countable. Hence all elements of
I can be listed as an infinite sequence {x 1 , x2 , . . .}:
x1 = 0.a11 a12 a13

x2 = 0.a21 a22 a23

x3 = 0.a31 a32 a33


..
..
.
.

Define
bn =

if ann 6= 1
if ann = 1

1
2

and x = 0.b1 b2 b3 . Then x [0, 1] but it is not a member of {x n }, contradiction.

Proposition 11.3. Let A be a non-empty set. Then the following are equivalent:
(a) A is countable.
(b) There exists a surjection f : N A.
(c) There exists an injection g : A N.
Proof. Assume that A is countable. If A is countably infinite, then there
exists a bijection f : N A. If A is finite, then there is bijection h :
{1, . . . , n} A for some n. Define f : N A by
(
h(i)
if 1 6 i 6 n,
f (i) =
h(n)
if i > n.
62

Check that f is a surjection. So the implication (a) = (b) is proved. To


prove the implication (b) = (c). Let f : N A be a surjection. Define
g : A N by the equation g(a) = smallest number in f 1 (a). Since f is
a surjection, f 1 (a) is non-empty for any a A, so that g is well-defined.
Next check that if a 6= a0 , then f 1 (a) and f 1 (a0 ) are disjoint, so they have
different smallest elements. The injectivity of g follows. Now the implication
(c) (a). Assume that g : A N is injective. We want to show that A is
countable. Note that g from A to g(A) is a bijection. So it suffices to show
that any subset B of N is countable. This is obvious when B is finite. Hence
assume that B is an infinite subset of N. We define a bijection h : N B.
Let h(1) be the smallest element of B. Since B is infinite, it is non-empty
and so h(1) is well-defined. Having already defined h(n 1), let h(n) be
the smallest element of the set {k B | k > h(n 1)}. Again this set
is non-empty, so h(n) is well-defined. Now check that the function h is a
bijection from N to B.

Corollary 11.4. The set N N is countable.
Proof. In view of the previous proposition, it is enough to construct an
injective function f : NN N. For example, let f (n, m) = 2 n 3m . Suppose
that 2n 3m = 2k 3l . If n < k, then 3m = 2kn 3l . The left side of this equality
is an odd number whereas the right is an even number. So n = k, and
3m = 3l . But then also m = l. Hence f is injective as required.

Proposition 11.5. If A and B are countable, then A B is countable.
Proof. Since A and B are countable, there exist surjective functions f : N
A and g : N B. Define h : NN AB by h(n, m) = (f (n), g(m)). The
function h is surjective and N N is countable, so A B is countable. 
Corollary 11.6. The set Q of all rational numbers is countable.
Proposition
S 11.7. If I is a countable set and A i is countable for every
i I, then iI Ai is countable.

Proof. For each i I, there exists a surjection f i : N Ai . Moreover, since


I is countable, there exists a surjection g : N I. Now define h : S
NN
S
A
by
h(n,
m)
=
f
(m).
Check
that
h
is
surjective
so
that
i
g(n)
iI
ii Ai is
countable.


63

Real numbers, Sequences


The set of all real numbers, R, has the following properties:
(a) the arithmetic properties,
(b) the ordering properties, and
(c) the completeness property.
The arithmetic properties start with the fact that any two real numbers a, b
can be added to produce a real number a + b, the sum of a and b. The rules
for addition are a + b = b + a, (a + b) + c = a + (b + c). There is a real number
0, called zero, such that a + 0 = 0 + a = a for all real numbers a. Each
real number a has a negative a such that a + (a) = 0. Besides addition,
we have multiplication; two real numbers a, b can be multiplied to produce
the product of a and b, a b. The rules for multiplication are ab = ba and
(ab)c = a(bc). There is a real number 1, called one, such that a1 = 1a = a,
and for each a 6= 0, there is a reciprocal 1/a such that a(1/a) = 1.
The ordering properties start with the fact that there is a subset R + of R, the
set of positive real numbers. The set R + is characterized by the property:
if a, b R+ , then a + b and ab R+ . The fact that a R+ is denoted by
0 < a. The set of negative real numbers R = R+ is the set of negatives
of elements in R+ . For every a R, we have a R+ or a = 0 or a R .
The notation a < b (or b > a) means that b a R + . We also write a 6 b
to mean a < b or a = b. The order properties of real numbers are as follows:
(a) a < b and b < c, then a < c.
(b) a < b and c > 0, then ac < bc.
(c) a < b and c R, then a + c < b + c.
(d) a < b and a, b > 0, then 1/b < 1/a.
If A R, a number M is called an upper bound for A if a 6 M for all
a A. Similarly, m is a lower bound for A if m 6 a for all a A. A
subset A of R is called bounded above if it has an upper bound, and is
called bounded below if it has a lower bound. If A has an upper and lower
bound, then is called bounded. A given subset of R may have several upper
bounds. If A has an upper bound M such that M 6 b for any upper bound
b of A, then we call M a least upper bound of A or supremum of A,
and denote it by M = sup A. Similarly, a real number m is called greatest
lower bound of A or infimum of A if m is a lower bound of A and b 6 m
64

for all lower bounds b of A. If m is the greatest lower bound of A, we write


m = inf A.
The completeness property of R asserts that every non-empty subset
A R that is bounded above has a least upper bound, and that every nonempty subset S R which is bounded below has a greatest lower bound.
Useful characterisations of a least upper bound and a greatest lower bound
are contained in the following propositions:
Proposition 11.8. Let A R be bounded above. Then a = sup A if and
only if x 6 a for any x A, and for any > 0 there exists x A such that
a < x + .
Proof. Assume first that a = sup A. Clearly, x 6 a for any x A. Take
> 0. If for all x A, x + 6 a, then x 6 a for all x. Hence
a is an upper bound of A contradicting the definition of a as the least
upper bound of A. Conversely, from x 6 a for any x A follows that a
is an upper bound of A. Assume that there is an upper bound b such that
b < a. Then we get a contradiction with the fact that for any > 0 there
exists x A such that a < x + . Let := (a b)/2 and x A. Then
x + 6 b + = (a + b)/2 < a.

There is also a similar characterisation of inf A provided that A is bounded
from below.
Proposition 11.9. Let A R be bounded from below. Then a = inf A if
and only if a 6 x for any x A, and for any > 0 there exists x A such
that x < a.
The proof of the proposition follows from the previous one by observing
two facts: if A is bounded from below than the set A = {x | x A} is
bounded from above and that sup(A) = inf A.
It is useful to introduce the extended real number system, R = R
{, } by adjoining symbols and subject to the ordering rule
< a < for all a R. If A is not bounded above,
R then we write
sup A = , and if A is not bounded below we write A = . For
example, we have inf R = and sup R = . We also have sup =
and inf = , and for all non-empty sets A, inf A 6 sup A. With this
terminology, the completeness property asserts that every subset of R has a
least upper bound and a greatest lower bound. The arithmetic operations

65

on R can be partially extended to R. In particular we have:


+ r = r + =

() + () = ,

for r R

and () + () = .

Subtraction is defined in a similar way with the exception that


() + ()

and

() + ()

are not defined. We also define multiplication by


(
, if r > 0,
r() = ()r =
, if r < 0,
and
()() = ,

()() = .

The multiplication 0 () is not defined.


If a is an upper bound of A and a A, then a is a maximum of A, and
we write a = max A. Similarly, if a A is a lower bound of A, then a is a
minimum of A and this fact is denoted by a = min A. If A and B R,
then A + B = {a + b | a A, b B}, A + a = {x + a | x A}, and
aA = {ax | x A}. Here are some properties of supremum and infimum:
(a) monotonicity property: A B, then sup A 6 sup B and inf B 6
inf A.
(b) reflection property: sup(A) = inf A and inf(A) = sup A.
(c) translation property: sup(A + a) = sup A + a and inf(A + a) =
inf A + a.
(d) dilation property: sup(aA) = a sup A and inf(aA) = a inf A provided that a > 0
(e) addition property: sup(A + B) = sup A + sup B and inf(A + B) =
inf A + inf B.
A sequence of real numbers is a function f : N R. We often write the
sequence as {f (n)} or {fn }. A sequence {an } of real numbers is said to
converge to a real number a if for every > 0 there is an integer n 0 such
that if n n0 , then |an a| < . In this situation we call a the limit of
{an }; a convergent sequence has a unique limit. We also write a n a or
limn an = a. A sequence {an } which does not converge to any limit in
66

R is said to diverge. We say that an , if for every M > 0, there is


n0 such that an > M for all n n0 . Similarly, an , if for every
M < 0 there exists n0 such that an < M for all n n0 . A sequence {an }
is bounded if |an | < M for some number M and all n N. A convergent
sequence is always bounded. Here are some elementary properties of limits
of sequences:
Proposition 11.10. Let {an } and {bn } be convergent sequences to a and
b, respectively. Let c R. Then we have:
(a) {can } converges to ca.
(b) the sequence {an + bn } converges to a + b
(c) the sequence {an bn } converges to a b
(d) if bn 6= 0 for all n and b 6= 0, then the sequence {a n /bn } converges to
a/b
A sequence {an } is called monotone increasing if a n 6 an+1 for all n N.
It is monotone decreasing if an an+1 for all n N.
Proposition 11.11. If {an } is a monotone increasing sequence that is
bounded above, an 6 M for all n, then {an } is convergent. If {an } is
monotone increasing and it is unbounded from above, then a n . If {an }
is monotone decreasing and it is bounded below, M 6 a n for all n, then {an }
is convergent, and if {an } is unbounded from below, then an .
Proof. If {an } is unbounded from above, then for every M there is k such
that ak > M . Since the sequence is increasing, a n ak M for all
n k. Thus an . Next assume that {an } is bounded above. Then
a := sup{an | n N} < . Let > 0. By the definition of supremum,
an 6 a for all n and there is an integer n0 such that a < an0 +. Since {an } is
monotone increasing, an 6 a < an + for all n n0 , that is, |an a| < for
all n n0 . Thus the sequence converges to a. The proof for monotonically
decreasing sequences is similar and is left as an exercise.

Let {an } be a sequence. If 0 < n1 < n2 < . . . are positive integers, then
{ank } is called a subsequence of {an }.
Proposition 11.12. If {an } is a convergent sequence with the limit a, then
every subsequence of {an } converges to a. Conversely, if a sequence {a n }
has the property that each of its subsequences is convergent, then {a n } itself
converges.
67

Proof. Let {ank } be a subsequence of {an }. For a given > 0 choose n0


such that |an a| < for all n > n0 . Note that if k > n0 , then nk > n0
and so |ank a| < for all k > n0 . Therefore, {ank } converges to a. The
converse follows from the fact that the sequence {a n } is a subsequence of
itself.

Let {an } be a bounded sequence. For each n N, let b n = supmn am =
sup{an , an+1 , . . .}. Then {bn } is monotone decreasing, and it is bounded
since {an } is bounded. In view of Proposition 11.11, {b n } converges. The
limit is called upper limit of {an }. Similarly, let cn = inf mn am =
inf{an , an+1 , . . .}. Then {cn } is monotone increasing, and it is bounded
since {an } is bounded. The limit of {cn } is called lower limit of {an }. If
{an } is not bounded above, then its upper limit is equal to , and if {a n }
is not bounded below, then its lower limit is equal to . Summarizing
lim sup an = lim an = inf sup ak = lim sup ak
nm kn

n kn

lim inf an = lim an = sup inf ak = lim inf ak .


nm kn

n kn

A useful characterisation of the upper limit is the following proposition.


Proposition 11.13. Let {an } be a sequence in R. Then the following are
equivalent:
(a) lim an = a;
(b) for every b > a, an < b for all but finitely many n and for every c < a,
an > c for infinitely many n.
Proof. Assume lim an = a. Then for any b > a, there exists m such that
supnm an < b. In particular, an < b for all n m. Since the sequence
{supnm an } is decreasing and convergent to a, it follows that a 6 sup nm an
for all m. Hence if c < a, then for every m there exists n m such that
c < an . This shows the implication (a) = (b). Conversely, assume that
(b) holds. Then for every b > a, there exists m such that a n < b for all
n m. Hence supnm an 6 b. This implies that lim sup an 6 b for every
b > a so that lim sup an 6 a. If for every c < a and for every m there
exists n m such that an > c, then for every m, supnm an c. This gives
lim sup an > c and since this holds for every c < a, we have lim sup a n a.
Thus lim sup an = a and the implication (b) = (a) is proved.


68

As an exercise formulate and prove the corresponding statement for the


lower limit. The basic properties of the upper and the lower limits are listed
in the following proposition:
Proposition 11.14. If {an } and {bn } are sequences of real numbers, then:
(a) lim sup(an ) = lim inf an and lim inf(an ) = lim sup an .
(c) lim sup(can ) = c lim sup an and lim inf(can ) = c lim inf an for any c >
0.
(d) lim sup(an + bn ) 6 lim sup an + lim sup bn and lim inf an + lim inf bn 6
lim inf(an + bn ).
(e) lim inf an 6 lim sup an , with equality if and only if {an } converges. In
this case lim sup an = lim an .
(f) If {ank } is a subsequence of {an }, then lim inf an 6 lim inf ank 6
lim sup ank 6 lim sup an .
The proof is left as an exercise.
Theorem 11.15 (Bolzano-Weierstrass Theorem).
Let {an } be a bounded sequence in R. Then there is a subsequence that
converges.
Proof. Set a = lim sup an . We will construct inductively a subsequence
{ank } of {an } which converges to a. In view of Proposition 11.13, there
exists n1 such that an1 > a 1. Having obtained n1 < n2 < < nk such
that anj > a 1/j for 1 6 j 6 k, we find, again by applying Proposition
11.13, nk+1 > nk such that ank+1 > a 1/(k + 1). Hence a 6 lim inf ank 6

lim sup ank 6 lim sup an = a. So lim ank = a and the proof is finished.

12
12.1

Problem Sheets
Problem Sheet 1

1. Check if the following functions are metrics on X.


(a) d(x, y) = |x2 y 2 | for x, y X = R
(b) d(x, y) = |x2 y 2 | for x, y X = (, 0]
(c) d(x, y) = |arctan x arctan y| for x, y X = R
69

2. Let X = R2 and let d be the usual metric. Denote by 0 = (0, 0) and


define
(
0
if x = y;
d0 (x, y) =
d(x, 0) + d(0, y)
if x 6= y.
Verify that d0 is a metric X. (The metric d0 is called the post office metric).
3. Let X = R2 . For x = (x1 , x2 ) and y = (y1 , y2 ) define

if x1 = y1 and x2 6= y2 or if x1 6= y1 and x2 = y2 ;
1/2
d(x, y) = 1
if x1 6= y1 and x2 6= y2 ;

0
otherwise.
Verify that d is a metric and that the rectangles in the figure hav different
area if d is used to measure the length of sides.

4.
(a) Show that if 0 6 a 6 b, then

a
b
6
.
1+a
1+b

(b) If a, b, c 0 and a 6 b + c, show that

b
c
a
6
+
.
1+a
1+b 1+c

(c) Use (b) to show that if d is a metric on X, then

is a metric on X.

e y) =
d(x,

d(x, y)
1 + d(x, y)

for x, y X

5. Let (Xi , di ) be a metric space for 1 6 i 6 n and let X =


1/2
X
n
2
di (xi , yi )
,
d2 (x, y) =
i=1

d (x, y) = max{di (xi , yi ) | 1 6 i 6 n},


70

Qn

i=1 Xi .

Define

where x = (x1 , . . . , xn ) and y = (y1 , . . . , yn ) X. Verify that d2 and d are


metrics on X.
6. Fix a positive integer n. Denote by P n the set of all polynomials p(x) =
ak xk + ak1 xk1 + + a1 x + a0 with real coefficients ai and k 6 n. For
p(x) = ak xk + ak1 xk1 + + a1 x + a0 Pn set

p = max{|a0 |, |a1 |, . . . , |ak |}.

Verify that is a norm on Pn .
7. Sketch the open ball B(0, 1) in the metric space (R 3 , di ), where di is
defined by
d1 (x, y) = |x1 y1 | + |x2 y2 | + |x3 y3 |
p
d2 (x, y) = (x1 y1 )2 + (x2 y2 )2 + (x3 y3 )2

d (x, y) = max{|x1 y1 |, |x2 y2 |, |x3 y3 |}.

for x = (x1 , x2 , x3 ) and y = (y1 , y2 , y3 ) R3 .


8. Show that diam B(x0 , r) 6 2r. Give an example showing that the strict
inequality is possible.

12.2

Problem Sheet 2

1. Calculate d(A, B) for the following pairs of subsets in R 2 equipped with


the standard metric:
(a) A = {(x, 0)| x R} and B = {(x, 1)| x R}.
(b) A is the set of points on the x-axis whose x-coordinate satisfies 2n <
x < 2n + 1 for some n Z and B is the set of points on the line line
y = 1 for which 2m 1 < x < 2m for some m Z.
(c) A = B(x0 , r0 ) and B = B(x1 , r1 ) where x0 , x1 R2 .
2. Let d and d0 be two metrics on X such that
d(x, y) 6 d0 (x, y) 6 d(x, y)
for all x, y X and positive constants and . Show that d and d 0 are
equivalent.Give an example of X and two equivalent metrics in X for which
71

the above inequality does not hold. Use the above fact to show that
d1 (x, y) =

n
X
i=1

d2 (x, y) =

"

|xi yi |

n
X
i=1

|xi yi |2

#1/2

d (x, y) = max{|xi yi | | i = 1, . . . , n}.

are equivalent on X = Rn .
3 Consider the set X = [1, 1] as a subspace of R a metric subspace of
R with the standard metric. Which of the following sets are open in X?
Which are open in R? Which are closed in X and which are closed in R?
(a) A = {x X | 1/2 < |x| < 2}
(b) B = {x X | 1/2 < |x| 6 2}
(c) C = {x R | 1/2 6 |x| < 1}
(d) D = {x R | 1/2 6 |x| 6 1}
(e) E = {x R | 0 < |x| 6 1 and 1/x 6 Z}
4. Sketch (where possible) the following sets, and decide whether it is an
open subset, or a closed subset, or neither of R 2 with the standard metric:
(a) A = {(x, y)| 1 < x 6 1 and 1 < y < 1}
(b) B = {(x, y)| xy = 0}
(c) C = {(x, y)| x Q, y R}
(d) D = {(x, y)| 1 < x < 1 and y = 0}
S
(e) E =
n=1 {(x, y)|x = 1/n and |y| 6 n}
5. Find the interior, the closure and the boundary of each of the following
subsets of R2 with the standard metric:

72

(a) A = {(x, y)) | x > 0 and y 6= 0}


(b) B = {(x, y) | x N, y R}
(c) C = A B
(d) D = {(x, y) | x is rational}
(e) F = {(x, y) | x 6= 0 and y 6 1/x}
6. Let A be a subset of a metric space X. Is the interior of A equal to the
interior of the closure of A. Is the closure of the interior of A equal to the
closure of A itself?
7 Consider a collection Ai of subsets of a metric space X. Show that
!
[
[
\
\
Ai
Ai
Ai
Ai
iI

iI

Ai

iI

iI

Ai

iI

iI

iI

Ai

Ai

iI

8. Let U be open in X and let A be closed in X. Show that U \ A is open


in X and A \ U is closed in X.
9. Let X and Y be metric spaces and A, B are non-empty subsets of X
and Y , respectively.
(a) Prove that if A B is open in X Y , then A and B are open in X
and Y , respectively.
(b) Prove that if A B is closed in X Y , then A and B are closed in X
and Y , respectively.

12.3

Problem Sheet 3

1. Show that A and A are disjoint, and A = A A. Conclude A is


open if and only if A = A \ A.

2. Show that A is closed if and only if A A.

3. Let A, B X. Show that (A B) A B and give an example in


R in which these sets are different. Show that if A B = , then (A B) =
A B.
73

4. Let X and Y be metric spaces and A, B are dense subsets of X and Y ,


respectively. Show that A B is dense in X Y .

5. Let (X, d) be a metric space and let a be a fixed point of X. Show that
|d(x, a) d(y, a)| 6 d(x, y)

(17)

for all x, y X.

6 Use (17) to show the following result. Let A be a dense subset of (X, d).
Show that a sequence {xn } of points on X converges to x if and only if
d(xn , a) d(x, a)
for all a A.

7. Use (17) to show that the function f : X R defined by f (x) = d(x, a),
x X, is continuous.
8. Define f : R R and g : R R by
(
1 if x 0
and
f (x) =
1 if x < 0

g(x) =

x
1

if x 0
if x < 0.

Show that f and g are not continuous at x = 0.


9. Let (X, dX ) and (Y, dY ) be metric spaces. Show that f : X Y is
continuous if and only if
f (A) f (A)
for all subsets A of X.
10. Let (X, dX ) and (Y, dY ) be metric spaces. Show that f : X Y is
continuous if and only if
f 1 (B) f 1 (B).
for all subsets B of Y .

12.4

Problem Sheet 4

1. Let Y be a subset of a metric space (X, d). Show that


Y = {x X|d(x, Y ) = 0}.
Show that the function
f (x) = d(x, Y )
is continuous on X. Conclude that
74

(a) B(Y, ) = {x X| d(x, Y ) < } is open in X, and


(b) B[Y, ] = {x X| d(x, Y ) 6 } is closed in X.
2. Let A and B be disjoint non-empty closed subsets of a metric space X.
Define
d(x, A)
f (x) =
for x X.
d(x, A) + d(x, B)
Show that f is a continuous function on X whose image is in [0, 1], and that
f (x) = 0 if and only if x A and f (x) = 1 if and only if x B. Next define
sets U = f 1 ([0, 1/2)) and V = f 1 ((1/2, 1]). Show that U and V are open
and disjoint, and that A U , B V .
3. Which of the following functions are uniformly continuous?
(a) f (x) = sin x on [0, )
1
on (0, 1)
1x

(c) h(x) = x on [0, )

(b) g(x) =

(d) k(x) = sin(1/x), on (0, 1)


4. Which of the following sequences converge uniformly on [0, 1].
(a) fn (x) =

x
1 + nx

(b) gn (x) =

xn
1 + xn

5. Let (X, d) and (Y, ) be metric spaces and let f : X Y . Show that f
is uniformly continuous if and only if for any two sequences {x n } and {yn }
such that d(xn , yn ) 0 it follows that (f (xn ), f (yn )) 0.
6. Suppose that {xn } is a sequence in a metric space (X, d) such that
d(xn , xn+1 ) 6 2n for all n N. Prove that {xn } is a Cauchy sequence.

7. Suppose that {xn } and {yn } are Cauchy sequences in a metric space
(X, d). Prove that the sequence of real numbers {d(x n , yn )} converges.

8. Decide if the following metric spaces are complete:

75

(a) ((0, ), d), where d(x, y) = |x2 y 2 | for x, y (0, ).


(b) ((/2, /2), d), where d(x, y) = |tan xtan y| for x, y (/2, /2).

12.5

Problem Sheet 5

1. Let X = (0, 1]be equipped with the usual


metric
d(x, y) = |x y|. Show
1

1
y) = for x, y X. Show that
that (X, d) is not complete. Let d(x,
x y
is complete.
d is a metric on X that is equivalent to d, and that (X, d)
2. Consider the space X consisting of all continuous functions f : [a, b] R.
For f, g X, define
Z b
|f (x) g(x)| dx.
d(f, g) =
a

Show that d is a metric on X. Is (X, d) complete?

3. Cantors Intersection Theorem Let (X, d) be a complete metric space


and let {Fn } be a sequence of non-empty closed subsets of X such
T that
Fn+1 Fn for all n and diam(Fn ) 0 as n 0. Prove that nN Fn
consists of exactly one point.
Show that, if any of the conditions,
(i) (X, d) is complete,
(ii) Fn is closed,
T
is omitted, then nN Fn may be empty.

(iii) diam(Fn ) 0

e are metric spaces and that f : X Y is


4. Suppose that (X, d) and (Y, d)
a bijection such that both f and f 1 are uniformly continuous. Show that
e is complete.
(X, d) is complete if and only if (Y, d)

5. Let {fn } be a sequence of continuous functions f n : R R with the


property that {fn (x)} is unbounded for all x Q. Using Baires theorem
show that there is at least one x Qc such that {fn (x)} is unbounded.
e be metric spaces such that (X, d) is complete. Let
6. Let (X, d) and (Y, d)
{fn } be a sequence of continuous functions from X to Y such that {f n (x)}
converges for every x X. Using Baires theorem show that for every
> 0 there exist k N and a non-empty open subset U of X such that
e n (x), fm (x)) < for all x U and all n, m k.
d(f
76

12.6

Problem Sheet 6

1.
(a) Let f (x) = x2 for x (0, a], and let X = (0, a] with the usual metric.
Find for what values of a is f a contraction. Show that f : X X
does not have a fixed point.
1
for x 1, and let X = [1, ) with the usual metric
x
d. Show that f : X X, that d(f (x), f (y)) < d(x, y) for all x 6= y
but f does not have a fixed point.

(b) Let f (x) = x +

Reconcile (a) and (b) with Banach fixed point theorem.


2. Consider f : R2 R2 given by f (x) = Ax, where


0.7
0.8
A=
.
0.2 0.05
Is f a contraction if R2 is equipped with the metric d1 , d2 , d ?
3. Consider the system of nonlinear equations
x1 = b1 + sin(a11 x1 ) + sin(a12 x2 ) + + sin(a1n xn ),

x2 = b2 + sin(a21 x1 ) + sin(a22 x2 ) + + sin(a2n xn ),


..
..
.
.

xn = bn + sin(an1 x1 ) + sin(an2 x2 ) + + sin(ann xn ),


where aik , 1 6 i, k 6 n, and bk , 1 6 k 6 n, are given real
P numbers.2 Show
that the system has a unique solution x = (x 1 , . . . , xn ) if 16i,k6n aik < 1.
4. For f C([0, 1], R), define

(T f )(x) = ex +

1
f (x)

,
2 1 + f (x)2

x [0, 1].

Show that (T f ) C([0, 1], R) and that T : C([0, 1], R) C([0, 1], R) is a
contraction. Use this fact to show that there exists exactly one function
f C([0, 1], R) such that
1
f (x)3 ex f (x)2 + f (x) = ex
2
for all x [0, 1].
77

The next problems provide a different construction of the completion of


(X, d).
An equivalence relation on a set X is a relation having the following
three properties:
(a) (Reflexivity) x x for every x X
(b) (Symmetry) If x y, then y x
(c) (Transitivity) If x y and y z, then x z.
The equivalence class determined by x, denoted by [x], is defined by
[x] = {y X|y x}. We have [x] = [y] if and only if x y.

5. Let (X, d) be a metric space and let X be the set of Cauchy sequences
x = {xn } in (X, d). Define a relation in X be declaring x = {xn } y =
{yn } to mean d(xn , yn ) 0.
(a) Show that is an equivalence relation.
e denote the
Denote by [x] the equivalence class of x X , and let X
set of these equivalence classes.

(b) Show that if x = {xn } and y = {yn } X , then limn d(xn , yn )


exists. Show that if x0 = {x0n } [x] and y0 = {yn0 } [y], then
lim d(xn , yn ) = lim d(x0n , yn0 ).

e define
For [x], [y] X,

D([x], [y]) = lim d(xn , yn ).


n

Note that the definition of D is unambiguous in view of the above


equality.
e D) is a complete metric space.
(c) Show that (X,
e D). Then xn = {xn , xn , xn , . . .} is
Hint: Let [xn ] is Cauchy in (X,
1
2
3
Cauchy in (X, d). So for every n N, there exists k n N such that
d(xnm , xnkn ) < 1/n

for all m kn .

Set x = {x1k1 , x2k2 , x3k3 , . . .}. Then show that x is Cauchy in (X, d) and
D([xn ], [x]) 0.
78

(d) If x X, let (x) be the equivalence class of the constant sequence


x = (x, x, x, . . .). That is, (x) = [x] = [{x, x, x, . . .}]. Show that
: X (X) is an isometry.
e D).
(e) Show that (X) is dense in (X,
e with x = {x1 , x2 , x3 , . . .}. Denote by xn the conHint: Let [x] X
stant sequence {xn , xn , xn , . . .} and show that D([xn ], [x]) 0.

12.7

Problem Sheet 7

1. Let (X, d) be a complete metric space and f : X X be a function


such that
d(f (x), f (y)) 6 d(x, y)
for all x, y B(x0 , r0 ), where 0 < < 1 and d(x0 , f (x0 ))/(1 ) = r0 .
Show that f has a unique fixed point p B(x 0 , r0 ).
2. (a) Let I = [x0 a, x0 + a] and let r > 0. If f, g : I R are continuous,
define
d(f, g) = sup{er(xx0 ) |f (x) g(x)||x I}.

Show that d defines a metric on C(I, R) which is equivalent to the supremum


metric (f, g) = sup{|f (x) g(x)|| x I}.
(b) Let K = I J where I = [x0 a, x0 + a], J = [y0 b, y0 + b], and
let f : K R be a continuous function satisfying |f (t, y 1 ) f (t, y2 )| 6
|y1 y2 | for all (t, y1 ), (t, y2 ) K. Let C = sup{|f (t, y)|| (t, y) K}, and
let = min{a, b/C}. For a continuous function y : [x 0 , x0 + ] J, set
Z x
T y(x) = y0 +
f (t, y(t))dt, x [x0 , x0 + ].
x0

Show that T : C([x0 , x0 + ], J) C([x0 , x0 + ], J) is a contraction


with respect to the metric d(y1 , y2 ) = sup{e2(xx0 ) |f (x) g(x)||x I}.
Remark: The above device simplifies the last step in the prove of Picards
theorem given in lectures. Recall that in the last step we had to take > 0
such that C < 1 in order to guarantee that T is a contraction with respect
to . Using d we dont have to adjust .
3. Let I = [x0 a, x0 + a] and let U = I R. Let f : U R be continuous.
(a) Let y0 , y1 R. Show that a continuous function y : I R is a solution
of
y 00 (x) = f (x, y(x)), x I
y(x0 ) = y0

y 0 (x0 ) = y1
79

if and only if
y(x) = y0 + (x x0 )y1 +

x0

(x s)f (s, y(s))ds,

x I.

(b) Assume, in addition, that f satisfies the Lipschitz condition with respect
to the second variable, |f (x, y1 )f (x, y2 )| 6 |y1 y2 | for all (x, y1 ), (x, y2 )
U , and some > 0. Prove that for given y 0 , y1 R there exists > 0 such
that the equation y 00 (x) = f (x, y(x)) has a unique solution y : [x 0 , x0 +
] R satisfying y(x0 ) = y0 and y 0 (x0 ) = y1 .
4. Let (X, d) be a metric space with the property that if Y is any nonempty closed subset of X and f : Y Y is any contraction, then f has a
fixed point. Show that (X, d) is complete.

Hint: Arguing by contradiction assume that there exists a Cauchy sequence


which does not converge in X. We may assume that x n 6= xm for all n 6= m.
For x X, let F (x) = inf{d(x, xn )|n N}. Show that F (x) > 0 for all
x. Choose (0, 1) and define sequence of integers {n k } as follows. Set
n1 = 1, let n2 be an integer satisfying n2 > n1 and d(xi , xj ) 6 F (xn1 ) for
all integers i, j n2 . If n1 , n2 , . . . , nk1 are chosen, then nk > nk1 is an
integer such that d(xi , xj ) 6 F (xnk1 ) for all integers i, j nk . Now let
Y = {xn1 , xn2 , xn3 , . . .} and let f : Y Y be given by f (xnk ) = xnk+1 for
all k 1. Show that Y is closed, f is a contraction but does not have a
fixed point.
5. Let A be a dense subset of a metric space (X, d), and let (Y, ) be complete. Consider a uniformly continuous function f : A Y . Show that
there exists a unique uniformly continuous function F : X Y such that
F (x) = f (x) for all x A.

12.8

Problem Sheet 8

1.
Show that if (X, dX ) and (Y, dY ) are compact metric spaces, then the product metric space (X Y, d) is compact. (Here d is the product metric).

2.
Show that if A1 , . . . ,Ak are compact subsets of a metric space (X, d), then
S
k
i=1 Ai is compact.

3.
Which of the following subsets of R and R 2 are compact? (R and R2 are
80

considered with the usual metrics).


(a) A = Q [0, 1]
(b) B = {(x, y) R2 |x2 + y 2 = 1}
(c) C = {(x, y) R2 |x2 + y 2 < 1}}
(d) D = {x||x| + |y| 6 1}
(e) E = {x|x 1 and 0 6 y 6 1/x}

4.
Consider (Q, d) where d is the usual metric. Give an example of a set in this
metric space that is closed and bounded but is not compact.
5.
Let A be a non-empty compact subset of a metric space (X, d).
(a) Let x X. Show that d(x, A) = d(x, a) for some a A.
(b) Let U X be open and A U .
Show that there exists > 0 such that S = {x X|d(x, A) < } U . Does
this hold if A is only closed but not compact?
6.
Show that if A is a totally bounded subset of a metric space (X, d), then
for every
Sn > 0 there exists a finite subset {a 1 , . . . , an } of A such that
A i=1 B(ai , ).

7.
Show that a metric space (X, d) is totally bounded if and only if every sequence {xn } X contains a Cauchy subsequence.

8.
Let X be a compact metric space and let U be an open cover of X. Show
that there exists a number r > 0 with the property: For every x X, there
exists U U such that B(x, r) U . The number r is called a Lebesgues
number of the cover U.

12.9

Problem Sheet 9

1.
Show that (X, T ) is a topological space.
(a) Let X be infinite set. Let
T = {A X| A = or A = X or X \ A is finite}.
This is called co-finite topology or finite complement topology.
81

(b) Let X be uncountable set. Define T by


T = {A X| A = or A = X or X \ A is countable}.
This is called co-countable topology or countable complement topology.
(c) Let X = R and let
T = {A R| A = or A = R or A = (a, ) with a R}.
2.
Let B = {[a, b)| a, b R}. Show that B is a basis for a topology on R. This
topology, denoted Tl , is called the lower-limit topology on R. Show that
the lower-limit topology is larger than the usual topology on R . Find the
closures of [a, b), (a, b), (a, b] and [a, b] in (R, T l ).

3.
Let T = {A R| 0 6 A or A = R}. Show that T is a topology on R. What
are the closed sets in (R, T )? What is {1}? Is this topology Hausdorff?

4.
Let A, B be a subsets of a topological space (X.T ). Show that A B AB
and A B = A B.

5.
Let A X where (X, T ) is a topological space. Show that X \ A = X \ A
and (X \ A) = X \ A.

6.
Prove the following statements about continuous functions and discrete and
indiscrete topological spaces.
(a) If X is discrete, then every function f : X Y , where Y is any topological spaces, is continuous.
(b) If X is not discrete, then there exists a topological space Y and a function f : X Y that is not continuous.
(c) If Y is an indiscrete topological space, then every function f : X Y ,
where X is any topological space, is continuous.
(d) If Y is not indiscrete, then there exists a topological sapce X and a
function f : X Y that is not continuous.

7
Let X be infinite set and let T be a co-finite topology on X. Show that any
continuous function f : X R is constant. (R is equipped with the usual
metric topology).
82

8.
Let X and Y be topological spaces and let B be a base of open sets for Y .
Show that a function f : X Y is continuous if and only if f 1 (U ) is open
in X for every U B.
9.
(a) Show that (a, b) is homeomorphic to (c, d), (c, ) and R. (All spaces
are equipped with the usual topology).
(b) Show that R2 \ {(0, 0)} is homeomorphic to R2 \ B((0, 0), 1).

10.
A topological property is a property that, if possessed by a topological
space X, is also possessed by any topological space homeomorphic to X.
(a) Show that if f : X Y is a homeomorphism, then f (U ) is open in Y
for any open set U X.
(b) Show that Hausdorff is a topological property.
(c) Is completeness a topological property of metric spaces?

12.10

Problem Sheet 10

1.
Let (X, T ) be a compact topological space and let A, B are closed subsets
of (X, T ). Show that A B is compact.

2.
Let X = (0, 1) and let

T = {A R| A = or A = (0, 1) or A = (0, 1 1/n) for n 2}.


Show that every open set A different than X is compact. Is X compact?
3.
Let T be a co-countable topology on R, that is,
T = {A R| A = or R \ A is countable}.
Is [0, 1] compact in (R, T )? What are the compact sets in (R, T )?

4.
Consider (R, M), where M is the usual metric topology in R. Let
T = {A R| A = or R \ A is compact in (R, M)}.
(a) Show that T is a topology on R.
83

(b) Show that (R, T ) is compact but not Hausdorff.

5.
Let A be closed and let B be compact in (X, T ). Show that AB is compact.
6.
Let (X, T ) be compact and let f : X R be a continuous function. Show
that f is bounded, that is, there is M > 0 such that |f (x)| 6 M for all
x X. Show that f attains its maximum and its minimum value.
7.
Let (X, T ) be compact and (Y, S) Hausdorff. Show that if f : X Y is a
continuous bijection, then f is a homeomorphism.

8.
Let (X, T ) be a compact Hausdorff space and let T 0 be another topology on
X. Show that:
(a) if T T 0 but T 6= T 0 , then (X, T 0 ) is Hausdorff but not compact.
(b) if T 0 T but T 6= T 0 , then (X, T 0 ) is compact but not Hausdorff.
Hint: Use Problem 7.
9.
Let X be infinite set with the co-finite topology T . Show that (X, T ) is
connected.
10.
Is the topological space (R, T ) from Problem 4 connected?

12.11

Problem Sheet 11

1.
Show that if A is a connected subspace of a topological space (X, T ) and if
A B A, then B is connected.

2.
If A and B are connected subsets of a topological space (X, T ) such that
A B 6= , then A B is connected.

3.
Let {An } be a sequence of connected subsets of a topological
space (X, T )
S
such that An An+1 6= for all n N. Prove that nN An is connected.

4.
Let (X, d) be a metric space. Call a function f : X R locally constant if
for every x there exists r > 0 such that f | B(x,r) : B(x, r) R is constant.
84

Show that if (X, d) is connected, then every locally constant function is constant.
5.
A metric space (X, dX ) is called a chain connected if for every pair x, y
of points in X and every > 0, there are finitely many points x = x 0 , x1 ,
x2 , . . . xn = y such that dX (xi+1 , xi ) < for i = 0, 1, . . . , n 1. Prove that
a compact, chain connected metric space is connected.
6.
A point p X is called a cut point if X \ {p} is disconnected. Show that
the property of having a cut point is a topological property.
7.
Show that no two of the intervals (a, b), (a, b], and [a, b] are homeomorphic.
8.
Show that R and R2 are not homeomorphic (R and R2 are equipped with
the usual topologies) .
9.
Let A be countable set. Show that R2 \ A is path connected.

10.
Show that if A is an open connected subset of R n , then A is path connected.

References
[Mu] J. R. Munkres, Toplogy , Prentice Hall, 2000.
[M] B. Mendelson, Introduction to topology, Dover Publications.
[C] F. Croom, Introduction to topology , .

85

311 Home Page

The Department of Mathematics and Statistics

620-311 METRIC SPACES


Semester 1, 2004
Home

Welcome to 620-311. Please use the index on the left to find out more about this
course.

Course
Objectives

Swarup Gadde
Room 173 Richard Berry
Email: gadde at ms.unimelb.edu.au

Lecture Notes
Assessment

Second class test: Friday May 14th from 9.05am to 9.55am during class.

References
Class Times/
Consultation
Hours

Corrections/Other Handouts:
Solutions:

Student Rep.

Solutions 1

Solutions 2

Solutions 3

Solutions 4

Student Survey
Results

Solutions 5

Solutions 6

Solutions 7

Solutions 8

Solutions 9

Solutions 10

Solutions 11

Revision Questions:
Questions

Note: All email addresses on this Web page are disguised: For example, the email address j.blow@ms.unimelb.edu.au is given as j.blow at ms.unimelb.edu.au

Date created: 25 February, 2004


Last Modified: 24 May, 2004
Authorised: Department of Mathematics and Statistics
Maintainer: S.Gadde
Email:s.gadde at ms.unimelb.edu.au

http://www.ms.unimelb.edu.au/~s620311/index.html5/3/2005 21:19:46

Solutions to Problem Set 1.

1. Check if the following functions are metrics on X.


(a) d(x, y) = |x2 y 2 | for x, y X = R
(b) d(x, y) = |x2 y 2 | for x, y X = (, 0]
(c) d(x, y) = |arctan x arctan y| for x, y X = R
Solution: (a) d does not satisfy the first axiom. Indeed, d(1, 1) = |(1) 2
(1)2 | = 0. So d is not a metric.
(b) d defines a metric.
(c) d is a metric. If d(x, y) = |arctan x arctan y| = 0, then arctan x =
arctan y, so that x = y since x 7 arctan x is an one-to-one function. The
remaining axioms are evident.
3. Let X = R2 . For x = (x1 , x2 ) and y = (y1 , y2 ) define

if x1 = y1 and x2 6= y2 or if x1 6= y1 and x2 = y2 ;
1/2
d(x, y) = 1
if x1 6= y1 and x2 6= y2 ;

0
otherwise.
Verify that d is a metric and that the rectangles in the figure have different
area if d is used to measure the length of sides.

Solution: The first two axioms are trivially satisfied. To see that d satisfies
the triangle inequality we argue by contradiction and assume that there are
x, y, z R2 such that
d(x, z) > d(x, y) + d(y, z).

In view of the definition of d, this can happen if :


(1) d(x, z) > 0 and d(x, y) = d(y, z) = 0.
(2) d(x, z) = 1 and d(x, y) + d(y, z) = 1/2.
In the case (1), x = y = z since d(x, y) = d(y, z) = 0. But this contradicts,
d(x, z) > 0.
In the case (2), one of the numbers d(x, y), d(y, z) is equal to 0 and the
other is equal to 1/2. Say d(x, y) = 0 and d(y, z) = 1/2. Hence x = y and
so d(x, z) = 1/2 contradicting d(x, z) = 1.
The area of the rectangle I is equal to 1/2 1/2 = 1/4. The area of the
rectangle II is equal to 1 1 = 1.
4.
1

II
1
1/2

I
1/2

b
a
6
.
1+a
1+b
a
b
c
(b) If a, b, c 0 and a 6 b + c, show that
6
+
.
1+a
1+b 1+c
(c) Use (b) to show that if d is a metric on X, then
(a) Show that if 0 6 a 6 b, then

e y) =
d(x,

d(x, y)
1 + d(x, y)

for x, y X

is a metric on X.
Solution: (a) Consider f (x) =
to f 0 (x) =

1
(1 + x)2

x
for x 0. The derivaitve of f is equal
1+x
> 0 for all x 0. So f is increasing on [0, ) and this

a
b
6
as required.
1+a
1+b
(b) Since 0 6 a 6 b + c, it follows from (a) that

means that f (a) 6 f (b) for 0 6 a 6 b, that is,

a
b+c
b
c
b
c
6
=
+
6
+
1+a
1+b+c
1+b+c 1+b+c
1+b 1+c
b
b
c
c
since
6
and
6
.
1+b+c
1+b
1+b+c
1+c
(c) The first two axiom of a metric are evident. To see that de satisfies the
triangle inequality take x, y, z X. Then d(x, z) 6 d(x, y) + d(y, z), and in
view of (b),
d(x, y)
d(y, z)
d(x, z)
6
+
,
1 + d(x, z)
1 + d(x, y) 1 + d(y, z)
e z) 6 d(x,
e y) + d(y,
e z)
that is, d(x,
Q
5. Let (Xi , di ) be a metric space for 1 6 i 6 n and let X = ni=1 Xi . Define
X
1/2
n
2
d2 (x, y) =
di (xi , yi )
,
i=1

d (x, y) = max{di (xi , yi ) | 1 6 i 6 n},

where x = (x1 , . . . , xn ) and y = (y1 , . . . , yn ) X. Verify that d2 and d are


metrics on X.
Solution: If d2 (x, y) = 0, then di (xi , yi ) = 0, 1 6 i 6 n, so that xi = yi

for 1 6 i 6 n. So x = (x1 , . . . , xn ) = (y1 , . . . , yn ). Clearly, d(x,


Qny) = d(y, x)
since di (xi , yi ) = di (yi , xi ) for 1 6 i 6 n. Let x, y, z
i=1 Xi . Set
ai = di (xi , zi ), bi = di (xi , yi ), and ci = di (yi , zi ). We have ai 6 bi + ci for all
1 6 i 6 n. Hence
1/2
1/2 X
X
n
n
2
2
.
(bi + ci )
6
ai
i=1

i=1

Since

X
n

d2 (x, z) =

1/2

X
n

6
(bi + ci )
i=1
i=1
1/2
1/2 X
X
n
n
b2i
+
a2i
6
i=1
i=1

and the triangle inequality follows.


7.

b2i

1/2

X
n
i=1

c2i

X
n

i=1
1/2

c2i

1/2

= d2 (x, y) + d2 (y, z),

1
o

1
o

Figure 1. A ball B(0, 1) in a) (R2 , d1 ), b) (R2 , d2 ), c) (R2 , d ).


8. Show that diam B(x0 , r) 6 2r. Give an example showing that the strict
inequality is possible.
Solution: If x, y B(x0 , r), then
d(x, y) 6 d(x, x0 ) + d(x0 , y) < r + r = 2r.

So, diam B(x0 , r) 6 2r. Consider, X = Z with the usual metric d(x, y) =
|x y|. Then B(0, 1/2) = {0} so that diam B(0, 1/2) = 0.

Solutions to Problem Set 2.


1. Calculate d(A, B) for the following pairs of subsets in R 2 equipped with
the standard metric:
(a) A = {(x, 0)| x R} and B = {(x, 1)| x R}.
(b) A is the set of points on the x-axis whose x-coordinate satisfies 2n <
x < 2n + 1 for some n Z and B is the set of points on the line line
y = 1 for which 2m 1 < x < 2m for some m Z.
(c) A = B(x0 , r0 ) and B = B(x1 , r1 ) where x0 , x1 R2 .
Solution:
(a) d(A, B) = 1
q
p
2
2
(b) d(A, B) = 4(m n) + 1 if m 6 n, and d(A, B) = 4 (n m) + 1 + 1
if m > n.
(c) d(A, B) = 0 if A B = and d(A, B) = d(x 0 , y1 ) r0 r1 .
2. Let d and d0 be two metrics on X such that
d(x, y) 6 d0 (x, y) 6 d(x, y)
for all x, y X and positive constants and . Show that d and d 0 are
equivalent. Give an example of X and two equivalent metrics in X for which
the above inequality does not hold. Use the above fact to show that
d1 (x, y) =

n
X
i=1

d2 (x, y) =

"

|xi yi |

n
X
i=1

|xi yi |2

#1/2

d (x, y) = max{|xi yi | | i = 1, . . . , n}.

are equivalent on X = Rn .
Solution: If d(xn , x) 0, then 0 6 d0 (xn , x) 0 since 0 6 d0 (xn , x) 6
d(xn , x) 0. If d0 (xn , x) 0, then 0 6 d(xn , x) 0 since 0 6 d(xn , x) 6
(1/) d(xn , x) 0. So d and d0 are equivalent. Consider on R metrics
d(x, y) = |x y|

d0 (x, y) =

and

|x y|
.
1 + |x y|

Then d and d0 are equivalent but there is no constant > 0 such that
d(x, y) 6 d0 (x, y),

for all x, y R.

Indeed, if such exists, then n = d(n, 0) 6 d 0 (n, 0) 6 1 for all n N.


But n 0, contradiction. Finally, observe that
max{|xi yi || i = 1, . . . , n} 6

n
X
i=1

|xi yi | 6 n max{|xi yi || i = 1, . . . , n}.


1

i.e.,
d (x, y) 6 d1 (x, y) 6 n d (x, y),

and

d (x, y) 6 d2 (x, y) 6
Combining these inequality we get

n d (x, y).

1
d1 (x, y) 6 d2 (x, y) 6 n d1 (x, y).
n
So d1 , d2 and d are equivalent.
3 Consider the set X = [1, 1] as a subspace of R a metric subspace of
R with the standard metric. Which of the following sets are open in X?
Which are open in R? Which are closed in X and which are closed in R?
(a)
(b)
(c)
(d)
(e)

A = {x X | 1/2 < |x| < 2}


B = {x X | 1/2 < |x| 6 2}
C = {x R | 1/2 6 |x| < 1}
D = {x R | 1/2 6 |x| 6 1}
E = {x R | 0 < |x| 6 1 and 1/x 6 Z}

Solution:

(a) A is open in X but not in R, A is not closed in both spaces


(b) B is open in X, not open in R, not closed in X and not closed in R
(c) C is not open and not closed in X. It is not open and not closed in
R.
(d) D is not open in both spaces, and closed in both spaces.
(e) E open in both spaces, not closed in both spaces.
4. Sketch (where possible) the following sets, and decide whether it is an
open subset, or a closed subset, or neither of R 2 with the standard metric:
(a)
(b)
(c)
(d)
(e)

A = {(x, y)| 1 < x 6 1 and 1 < y < 1}


B = {(x, y)| xy = 0}
C = {(x, y)| x Q}
D = {(x, y)| 1 < 0 < 1 and y = 0}
S
E=
n=1 {(x, y)|x = 1/n and |y| 6 n}

Solution:
(a)
(b)
(c)
(d)
(e)

A is neither open nor closed


B is closed
C is neither open nor closed.
D is neither open nor closed
E neither open nor closed

1
1

-1
d

1
e

Figure 1
See Fig 1.
5. Find the interior, the closure and the boundary of each of the following
subsets of R2 with the standard metric:
(a)
(b)
(c)
(d)
(e)

A = {(x, y) R2 | x > 0 and y 6= 0}


B = {(x, y) R2 | x N}
C =AB
D = {(x, y) R2 | x is rational}
F = {(x, y) R2 | x 6= 0 and y 6 1/x}

Solution:

(a) A = A, A = {(x, y)|x 0}, A = {(x, y)|x = 0, y R}{(x, y)|x


0, y = 0}
(b) B = , B = B, B = B
(c) C = A, C = {(x, y) R2 | x 0}, C = {(x, y)|x = 0, y
R} {(x, y) R2 |x 0, y = 0}
(d) D = , D = R2 , D = R2 .
(e) F = {(x, y)|x 6= 0 and y < 1/x}, F = F {(x, y)| x = 0, y R},
F = {(x, y) | x 6= 0 and y = 1/x} {(x, y)| x = 0, y R}.
6. Let A be a subset of a metric space X. Is the interior of A equal to the
interior of the closure of A. Is the closure of the interior of A equal to the
closure of A itself?
Solution The answer to both questions is No!. For example consider A = Q
in R equipped with the usual metric. Then A = but A = R so that
(A) = R = R. Also A = .

7 Consider a collection Ai of subsets of a metric space X. Show that


!
[
[
\
\
Ai
Ai
Ai
Ai
iI

iI

Ai

iI

iI

iI

Ai

iI

iI

Ai

Ai

iI


S
S
S

Solution:
iI Ai
iI Ai : Let x S iI Ai . Then x Ai for
some i. Hence B(x,S
r) Ai and so, B(x, r) iI Ai . Consequently, x is
an interior point of iI Ai .
T
T
T
T

iI Ai
iI Ai : Let x
iI Ai . So for every B(x, r) iI Ai 6=
which means that B(x, r) Ai 6= for all i I. Hence x is an adherent
T
point of Ai for all i I, that is, x Ai for all i I. Therefore, x iI Ai .


T
T
T
T

iI Ai :
Let x
iI Ai
iI Ai . So B(x, r)
iI Ai .

Hence
T B(x, r) Ai for all i I, that is x Ai for all i I. Therefore,
x iI Ai .
S
S
S
x Aj for

iI Ai . Then
iI Ai : Let x
iI Ai
 some j I, and
S
so, B(x, r) Aj 6= . Consequently, B(x, r)
6= . This mean
iI Ai
S
S
that x is an adherent point of iI Ai , i.e., x iI Ai .

8. Let U be open in X and let A be closed in X. Show that U \ A is open


in X and A \ U is closed in X.
Solution: Observe that




U \ A = U X \ A , and A \ U = A X \ U .

Since A is closed, X \ A is open, and, in view of the first equality U \ A is


open as an intersection of two open sets. Similarly, X \ U is closed, and, in
view of the second equality, A \ U is closed as an intersection of two closed
sets.
9. Let X and Y be metric spaces and A, B are non-empty subsets of X
and Y , respectively.
(a) Prove that if A B is open in X Y , then A and B are open in X
and Y , respectively.
(b) Prove that if A B is closed in X Y , then A and B are closed in
X and Y , respectively.
Solution: (a) Let a A. We have to show that a is an interior point of A.
Since B is non-empty, there is b B. So (a, b) A B. Since A B is
open, (a, b) is an interior point of A B and there exists r > 0 such that
B((a, b), r) A B. For any x B(a, r), we have
d((a, b), (x, b)) = dX (a, x) + dY (b, b) = dX (a, x) < r,

so that (x, b) B((a, b), r) A B. Hence B(a, r) A and a is an interior


point of A. Consequently, any point in A is an interior point of A which
means that A is open.

(b) Let x X be an adherent point of A. We have to show that x A.


There exists a sequence {xn } such that xn A and xn x in X. Since Y
is non-empty, there is y Y . For the sequence {(x n , y) A B, we have
d((xn , y), (x, y)) = dX (xn , x) + dY (y, y) = dX (xn , x) 0,

showing that (x, y) is an adherent point of A B. Since A B is closed in


X Y , (x, y) A B, that is, x A, as required.

Solutions to Problem Set 3.


1. Show that A and A are disjoint, and A = A A. Conclude A is
open if and only if A = A \ A.
Solution: Let x A A. Then B(x, r) A for some r (since x is an
interior point of A) which implies B(x, r) [X \ A] = . On the other hand
B(x, r) [X \ A] 6= (since x A X \ A in view of definition of A),
contradiction. Since A = A X \ A A, A A A. Conversely, let
x A. Then B(x, r) A 6= for all r > 0. If B(x, r) A for some r, then
x A . Otherwise, B(x, r) [X \ A] 6= for all r > 0, which together with
B(x, r) A 6= for all r > 0 implies that x A. So A A A.
If A is open, then A = A so that A = A A = A A, and A = A \ A
since A A = . Conversely, if A = A\A, then A = A\A A\A = A
since A A. Hence A \ A = A \ A implying that A = A since A , A A.
So A is open as required.
2. Show that A is closed if and only if A A.
Solution: Assume that A is closed. Then A = A X \ A A = A.
Conversely, assume that A A. If x A but x 6 A, then B(x, r) A 6=
(since x is an adherent point of A) and B(x, r) [X \ A] 6= (since x
X \ A). Hence x A X \ A = A A, contradiction. Consequently, if
x A, then x A showing that A is closed. Alternatively, using Problem
1, A = A A A A A, so A is closed.

3. Let A, B X. Show that (A B) A B and give an example in


R in which these sets are different. Show that if A B = , then (A B) =
A B.
Solution: Observe that
(A B) = (A B) X \ (A B) = A B (X \ A X \ B)
[A B] [X \ A X \ B] [A X \ A] [B X \ B]
= A B.
Hence (A B) A B, as required. Strict inclusion is possible. For
example, take: X = R with the usual metric and A = (0, 1], B = [1, 2).
Then A = {0, 1}, B = {1, 2}, A B = {0, 1, 2} and (A B) = {0, 2}
so that (A B) 6= A B.
Finally, assume A B = . It suffices to show that A B (A B).
Let x A B. We assume x A = A X \ A. Hence x A and x 6 B
since A B = . Since x 6 B, x X \ B X \ B, and B(x, r) X \ B
X \ B since X \ B is open. Hence, if 0 < s 6 r, then B(x, s) B(x, r),
and so B(x, s) [(X \ A) (X \ B)] = B(x, s) (X \ A) 6= . If r < s,
then 6= B(x, r) [(X \ A) (X \ B)] B(x, s) [(X \ A) (X \ B)].
Combining these two cases, B(x, s) [(X \ A) (X \ B)] 6= for all s > 0,
i.e., x (X \ A) (X \ B) = X \ (A B). Since x A A A B, it
follows that x A B X \ (A B) = (A B), as required.
1

4. Let X and Y be metric spaces and A, B are dense subsets of X and Y ,


respectively. Show that A B is dense in X Y .
Solution:
We have to show that every point (x, y) X Y is an adherent point of
A B. Let (x, y) X Y . Then x is an adherent point of A and y is
an adherent point of B since A and B are dense in X and Y , respectively.
Hence there exist sequence {xn } and {yn } in A and B such that xn x in
X and yn y in Y . Consequently, (xn , yn ) (x, y) in X Y . Hence (x, y)
is an adherent point of A B, and A B = X Y .
5. Let (X, d) be a metric space and let a be a fixed point of X. Show that
|d(x, a) d(y, a)| 6 d(x, y)

(1)

for all x, y X.
Solution: By the triangle inequality,
d(x, a) 6 d(x, y) + d(y, a)
for all x, y X. So
But x, y are arbitrary,

d(x, a) d(y, a) 6 d(x, y).

d(y, a) d(x, a) 6 d(y, x) = d(x, y)


which combined with the above inequality gives
d(x, y) 6 d(x, a) d(y, a) 6 d(x, y) that is, |d(x, a) d(y, a)| 6 d(x, y).
6 Use (1) to show the following result. Let A be a dense subset of (X, d).
Show that a sequence {xn } of points on X converges to x if and only if
d(xn , a) d(x, a)
for all a A.
Solution: Assume first that xn x, that is, d(xn , x) 0. If a A, then,
by (1),
|d(xn , a) d(x, a)| 6 d(xn , x) 0
showing that d(xn , a) d(x, a). Conversely, assume that d(x n , a) d(x, a)
for all a A. Let > 0 and choose a A such that a B(x, /3) (such a
exists since A is dense). Moreover, there exists k N so that
/3 < d(xn , a) d(x, a) < /3

for all n k.

Then, for all n k,

0 6 d(xn , x 6 d(xn , a) + d(a, x) 6 d(xn , a) + /3 < d(x, a) + /3 + /3


< /3 + /3 + /3 = .

So d(xn , x) 0.

7. Use (1) to show that the function f : X R defined by f (x) = d(x, a),
x X, is continuous.

Solution:
Let x0 X. Take > 0 and set = . Then, in view of (1),
|d(x0 , a) d(x, a)| 6 d(x0 , x) < =

for all x such that d(x0 , x) < . Hence f is continuous at x0 and since x0
was an arbitrary point, f is continuous on X.
8. Define f : R R and g : R R by
(
(
x
1 if x 0
and g(x) =
f (x) =
1
1 if x < 0

if x 0
if x < 0.

Show that f and g are not continuous at x = 0.


Solution: Consider f . By the definition of f , f (0) = 1. Take B(f (0), 1/2) =
B(1, 1/2) = (3/2, 1/2). Then for every > 0, f (B(0, )) = {1, 1},
and since 1 6 B(f (0), 1/2), f is not continuous at 0.
Next consider g. By the definition of g, g(0) = 0. Take = 1/2 and consider B(g(0), 1/2) = B(0, 1/2) = (1/2.1/2). For every > 0, f (B(0, )) =
{1} [0, ) 6 B(g(0), 1/2) = (1/2, 1/2). Hence g is not continuous at 0.

9. Let (X, dX ) and (Y, dY ) be metric spaces. Show that f : X Y is


continuous if and only if
f (A) f (A)
for all subsets A of X.
Solution:
Assume that f is continuous. Let A X. Then f (A) is a closed subspace
of Y , and so f 1 (f (A)) is a closed subspace of X. Since A f 1 (f (A))
f 1 (f (A)), A f 1 (f (A)) since f 1 (f (A)) is closed. Hence f (A)
f (f 1 (f (A))) = f (A) as required. Conversely, assume that f (A) f (A)
for all A X. Let B Y be closed. We have to show that A := f 1 (B) is
a closed subset of X. Since f (A) f (A) = f (f 1 (B)) B = B since B is
closed in Y , we conclude A f 1 (B) = A. Hence A = f 1 (B) is closed as
required.
10. Let (X, dX ) and (Y, dY ) be metric spaces. Show that f : X Y is
continuous if and only if
f 1 (B) f 1 (B).

for all subsets B of Y .


Solution:
Assume that f is continuous and let B Y . Then f 1 (B) is closed since B
is closed and f is continuous. Since f 1 (B) f 1 (B), f 1 (B) f 1 (B) =
f 1 (B) and the result follows. Conversely, assume that f 1 (B) f 1 (B)
for all B Y . Let A Y be closed. Then f 1 (B) f 1 (B) = f 1 (B) so
that f 1 (B) is closed, and f is continuous.

Solutions to Problem Set 4.

1. Let Y be a subset of a metric space (X, d). Show that


Show that the function

Y = {x X|d(x, Y ) = 0}.

f (x) = d(x, Y )
is continuous on X. Conclude that
(a) B(Y, ) = {x X| d(x, Y ) < } is open in X, and
(b) B[Y, ] = {x X| d(x, Y ) < } is closed in X.
Solution: Let x Y . Then there exists {x n } Y such that d(x, xn ) 0.
By the definition of d(x, Y ), 0 6 d(x, Y ) 6 d(x, x n ) 0. So d(x, Y ) = 0.
Conversely, let d(x, Y ) = 0. Then for every n N, there exists x n Y so
that d(x, xn ) < 1/n. (Note that if there is no such x n , then d(x, Y ) 1/n
contradicting d(x, Y ) = 0). Hence d(x, x n ) 0 showing that x Y . To see
that f is continuous, take x, y X. Then by the triangle inequality,
d(x, Y ) 6 d(x, z) 6 d(x, y) + d(y, z)

for all z Y Hence d(x, Y ) d(x, y) 6 d(y, z) for all z Y , so that


d(x, Y )d(x, y) 6 d(y, Y ). Similarly, d(y, Y )d(y, x) 6 d(x, Y ). Combining
both inequalities,
d(x, y) 6 d(x, Y ) d(y, Y ) 6 d(x, y), i.e, |d(x, Y ) d(y, Y )| 6 d(x, y).

This mean that |f (x) f (y)| 6 d(x, y). Now take > 0 and set = . Then
if d(x, y) < , then |f (x) f (y)| 6 d(x, y) < = . So f is continuous. (in
fact, the above shows that f is uniformly continuous).
2. Let A and B be disjoint non-empty closed subsets of a metric space X.
Define
d(x, A)
f (x) =
for x X.
d(x, A) + d(x, B)
Show that f is a continuous function on X whose image is in [0, 1], and that
f (x) = 0 if and only if x A and f (x) = 1 if and only if x B. Next define
sets U = f 1 ([0, 1/2)) and V = f 1 ((1/2, 1]). Show that U and V are open
and disjoint, and that A U , B V .
Solution: By Problem 1, functions x 7 d(x, A) and x 7 d(x, B) are
continuous. Hence x 7 d(x, A) + d(x, B) is continuous, and since d(x, A) +
d(x, B) > 0, the function f (x) = d(x, A)/(d(x, A) + d(x, B)) is continuous
on X. If x A, then d(x, A) = 0, and so f (x) = 0. Conversely, if f (x) = 0,
then d(x, A) = 0 and so, in view of Problem 1, x A since A is closed.
If x B, then d(x, B) = 0 and d(x, A) > 0, again in view of Problem 1.
So f (x) = 1. Conversely, if f (x) = 1, then d(x, B) = 0, so that x B.
Summing up, f 1 {0}) = A and f 1 ({1}) = B. Since f : X [0, 1] and
[0, 1/2) and (1/2, 1] are open sets in [0, 1] equipped with the usual metric,
U = f 1 ([0, 1/2)) and V = f 1 ((1/2, 1]) are open in X. If x U V ,
1

then f (x) [0, 1/2) and (1/2, 1] which is impossible. So U V = . Since


{0} [0, 1/2) and {1} (1/2, 1], A = f 1 ({0}) f 1 ([0, 1/2)) = U and
B = f 1 ({1}) f 1 ((1/2, 1]) = V .

3. Which of the following functions are uniformly continuous?


(a) f (x) = sin x on [0, )
1
(b) g(x) =
on (0, 1)
1x

(c) h(x) = x on [0, )


(d) k(x) = sin(1/x), on (0, 1)
Solution: (a) By mean value theorem, for any x, y [0, ) such that
x < y there exists c (x, y) for which
f (x) f (y) = f 0 (c) (x y).

So taking > 0 and setting = ,

|f (x) f (y)| = |f 0 (c)| |x y| = |cos c| |x y| 6 |x y| < =

for all x, y [o, ) satsifying |x y| < . Hence f is uniformly continuous.


(b) Assume that g is uniformly continuous. Then for = 1/2, there is
> 0 so that if x, y (0, 1) and |x y| < , then |g(x) g(y)| < 1. Take
x = 1 1/n and y = 1 (2/n), with n 2 and so that 1/n < . Then
|x y| = 1/n < , and
|g(x) g(y)| = |n n/2| = n/2 1 > .

So g is not uniformly continuous.


(c) Let > 0. Since h is continuous at 0 and h(0) = 0, there exists 1 > 0
such that |h(x)| < /2 for all x [0, 1 ). Then for any x, y [0, 1 ],
|x y| < 1 and |h(x) h(y)| 6 |h(x)| + |h(y)| < /2 + /2 = . So h is
uniformly continuous on [0, 1 ]. Next observe that if 1 /2 6 x < y, then, by
mean value theorem, there exists t (x, y) such that
r
2
1
1
0
|xy|.
|xy| =
|h(x)h(y)| = |h (t)||xy| = |xy| 6 p
1
2 t
2 1 /2
r
1
So taking 2 =
we get
2
r
r
r
2
2
1
|x y| <

=
|h(x) h(y)| 6
1
1
2
for all x, y 1 /2 and |x y| < 2 . So h is uniformly continuous on
[1 /2, ). Now if = min{1 , 2 } and x, y [0, ) satisfy |x y| < , then
|h(x) h(y)| < so that h is uniformly continous on [0, ).
(d) Take = 1/2. If k is uniformly continuous, then there is > 0 so that
if x, y (0, 1) and |x y| < , then |k(x) k(y)| < 1/2. Set x n = 1/(n)
and yn = 1/((n + 1/2)). Since xn 0 and yn 0 as n , there exists
n such that xn , yn < /2 so that |xn yn | 6 xn + yn < /2 + /2 = . On
the other hand, k(xn ) = sin n = 0 and k(y) = sin((n + 1/2)) = (1) n . So
|k(x) k(y)| = 1 > . Hence k is not uniformly continuous.

4. Which of the following sequences converge uniformly on [0, 1].


x
(a) fn (x) =
1 +nnx
x
(b) gn (x) =
1 + xn
x
Solution: (a) For every x [0, 1], fn (x) =
0. So {fn } converges
1 + nx
1
pointwise to f , where f (x) = 0 for x [0, 1]. Note that f n0 (x) =
>
(1 + nx)2
0 for x [0, 1]. That is, fn is increasing on [0, 1] and as such, the maximum value of f occurs at x = 1. So, sup{|f n (x) f (x)|| x [0, 1]} =
sup{fn (x)| x [0, 1]} = fn (1) = 1/(n + 1) 0. Hence {fn } converges
uniformly to f .
(
1/2,
if x [0.1)
(b) The sequence {gn } converges pointwise to g(x) =
0,
if x = 1.
n1 (1 + xn ) nx2n1
n1
nx
nx
Also, gn0 (x) =
=
> 0 for all x [0, 1].
(1 + xn )2
(1 + xn )2
So, sup{|gn (x) g(x)|| x [0, 1]} = 1/2 6 0 as n . So {g n } does not
converge uniformly to g.
5. Let (X, d) and (Y, ) be metric spaces and let f : X Y . Show that f
is uniformly continuous if and only if for any two sequences {x n } and {yn }
such that d(xn , yn ) 0 it follows that (f (xn ), f (yn )) 0.
Solution: Assume that f is uniformly continuous. Let > 0. We have to
show that there exists k N such that
d(f (xn ), f (yn )) < ,

for all n k.

Since f is uniformly continuous, there exists > 0 such that


(1)

(f (x), f (y)) <

for all x, y satisfying d(x, y) < .

If d(xn , yn ) 0, then there exists k N such that d(x n , yn ) < for all
n k. So, in view of (1), we have that (f (x n ), f (yn )) < for all n k,
as required. Conversely, arguing by contradiction assume that f is not
uniformly continuous. Then there exists > 0 such that for all > 0 there
are points x, y X satisfying d(x, y) < but (f (x), f (y)) . Choose =
1
n . Then by the above there exist points x n , yn X such that d(xn , yn ) <
1/n but (f (xn ), f (yn )) . So, d(xn , yn ) 0 but (f (xn ), f (yn )) 6 0.
6. Suppose that {xn } is a sequence in a metric space (X, d) such that
d(xn , xn+1 ) 6 2n for all n N. Prove that {xn } is a Cauchy sequence.

Solution: Let m > n k. Then

d(xn , xm ) 6 d(xn , xn+1 ) + d(xn+1 , xn+2 ) + + d(xm1 , xm )

X 1
1
1
1
1
1
6 n + n+1 + + m1 6
= n1 6 k1 .
j
2
2
2
2
2
2
j=n

So if > 0 choose k N such that 1/2k1 < . Then, in view of the above
inequality,
1
d(xn , xm ) < k1 <
2
for all n, m k. Hence {xn } is Cauchy.

7. Suppose that {xn } and {yn } are Cauchy sequences in a metric space
(X, d). Prove that the sequence of real numbers {d(x n , yn )} converges.

Solution: Since R equipped with the usual metric is complete, it suffices


to show that the sequence {d(xn , yn )} is Cauchy. Let > 0. Since {xn } and
{yn } are Cauchy sequences, there exists k N such that
(2)

d(xn , xm ) < /2

and

d(yn , ym ) < /2

for all n, m k. Since by the triangle inequality


d(xn , yn ) 6 d(xn , xk ) + d(xk , yk ) + d(yk , yn ), and
d(xm , ym ) 6 d(xm , xn ) + d(xn , yn ) + d(yn , ym )
which implies
(3)

|d(xn , yn ) d(xm , ym )| 6 d(xn , xm ) + d(yn , ym ).

Combining (2) and (3) we get

|d(xn , yn ) d(xm , ym )| 6 d(xn , xm ) + d(yn , ym ) < /2 + /2 =

for all n, m k. So {d(xn , yn )} is Cauchy and since R is complete, {d(x n , yn )}


converges.
8. Decide if the following metric spaces are complete:
(a) ((0, ), d), where d(x, y) = |x2 y 2 | for x, y (0, ).
(b) ((/2, /2), d), where d(x, y) = |tan xtan y| for x, y (/2, /2).
Solution: (a) ((0, ), d) is not complete. To see this consider for example
{xn } with xn = 1/n for n 1. Then {xn } p
Cauchy in ((0, ), d). Indeed,
let > 0 and let k N be such that 1/k < /2. Then for all n, m k,


1
1
1
2
1

d(xn , xm ) = 2 2 6 2 + 2 < 2 < .
n
m
n
m
k

If ((0, ), d) is complete, there is x (0, ), such that d(x n , x) = |x2n


x2 | 0. But
1
0 < x2 = |x2 | 6 |x2n x2 | + |x2n | = d(xn , x) + 2 0,
n
contradiction. So ((0, ), d) is not complete.
(b) ((/2, /2), d) is complete. To see this let {x n } be Cauchy in ((/2, /2), d).
Set yn = tan xn . Then {yn } is Cauchy in R equipped with the usual metric
since |yn ym | = |tan xn tan xm | = d(xn, xm ). Since R with the usual
metric is complete, there is y R such that |y n y| 0. Let x = arctan y.
Then x (/2, /2), y = tan x, and
d(xn , x) = |tan xn tan x| = |yn y| 0.

Consequently, ((/2, /2), d) is complete.

Solutions to Problem Set 5.


1. Let X = (0, 1] be equipped with the usual
d(x, y) = |x y|. Show
metric

1
1
y) = for x, y X. Show that
that (X, d) is not complete. Let d(x,
x y

is complete.
d is a metric on X that is equivalent to d, and that (X, d)
Solution: The sequence {xn } with xn = 1/n is Cauchy in (X, d) but does
not converge. Indeed, for > 0 choose k N so that 1/k < /2. Then, for
n, m k,


1

1 6 1 + 1 6 2 < .
n m n m
k
If there is x X such that d(xn , x) 0, then


1
1
1

0 < |x| 6 x + = d(xn , x) + 0,
n
n
n

contradiction. So, (X, d) is not complete. That d and de are equivalent


follows from that fact that f (x) = 1/x is continuous. Finally, assume that
e Then 1/xn [1, ) (since xn (0, 1]), and the
{xn } is Cauchy in (X, d).
sequence {1/xn } is Cauchy in ([1, ), d). Since [1, ) is closed
in (R, d)


1
y 0 for some
and (R, d) is complete, ([1, ), d) is complete. Thus
x
n


1
1
e n , x) 0.

y [1, ). Set x = 1/y (0, 1]. Then


0, that is, d(x
xn x
e is complete.
Consequently, (X, d)
2. Consider the space X consisting of all continuous functions f : [a, b] R.
For f, g X, define
Z b
d(f, g) =
|f (x) g(x)| dx.
a

Show that d defines a metric on X. Is (X, d) complete?


Solution: Assume that d(f, g) = 0 for some f, g X. If f 6= g, then there
is x0 [a, b] such that |f (x0 )g(x0 )| > 0. By continuity of |f (x)g(x)| there
is interval I [a, b] containing x0 such that |f (x) g(x)| |f (x0 ) g(x0 )|/2
for all x I. Then
Z b
|f (x0 ) g(x0 )|
|f (x) g(x)|dx
length of I > 0,
2
a

contradiction. Consequently, f = g on [a, b]. The remaining properties of


the definition of the metric are obvious. The space (X, d) is not complete.
To see this, let

if x [a, (a + b)/2];
0
fn (x) = n(x (a + b)/2)
if x [(a + b)/2, (a + b)/2 + 1/n];

1
if x [(a + b)/2 + 1/n, 1].
1

Then, for > 0 there is k N such that 1/k < , and for n, m k,


Z b
1 1
1 1
d(fn , fm ) =
|fn (x) fm (x)|dx = 6 < .
2 n m
k
a

So {fn } is Cauchy with respect to the metric d. However, {f n } does not


Rb
converge. Indeed, if a |fn (x) f (x)|dx 0 for some f X. Then
Z b
Z (a+b)/2
Z (a+b)/2
|fn (x) f (x)|dx 0,
|fn (x) f (x)|dx 6
|f (x)|dx =
a

R (a+b)/2

so that a
|f (x)|dx = 0, and since |f (x)| is continuous and 0, we
get that f (x) = 0 for all x [a, (a + b)/2. If (a + b)/2 < s 6 b, then
(a + b)/2 + 1/n < s for n large, and
Z b
Z b
Z b
|fn (x) f (x)|dx 0,
|fn (x) f (x)|dx 6
|1 f (x)|dx =
s

Rb

so that s |1 f (x)|dx = 0 for all (a + b)/2 < s 6 b. Hence f (x) = 1


for (a + b)/2 < x 6 b. However, f is not continuous at x = (a + b)/2,
contradiction.

(a+b)/2

(a+b)/2

Figure 1. (a) Graphs of fn and f .

3. Cantors Intersection Theorem Let (X, d) be a complete metric space


and let {Fn } be a sequence of non-empty closed subsets of X such
T that
Fn+1 Fn for all n and diam(Fn ) 0 as n 0. Prove that nN Fn
consists of exactly one point.
Show that, if any of the conditions,
(i) (X, d) is complete,
(ii) Fn is closed,
(iii) diam(Fn ) 0
T
is omitted, then nN Fn may be empty.
Solution: For every n choose a point in F n , say xn . We claim that {xn } is
Cauchy. Indeed, take > 0 and choose k N such that diamF k < , This
is possible since diamFn 0. Since xn , xm Fk for n, m k,
d(xn , xm ) 6 sup{d(x, y)| x, y Fk } = diamFk < .

Thus {xn } is Cauchy and since (X, d) is complete,


there exists x X such
T
that d(xn , x) 0 . We claim that x nN Fn . Take any k N, then
xn TFk for all n k. Since xn x and Fk T
is closed, x Fk . Hence
x nN Fn . If there is another point, say y, in nN Fn , then
0 < d(x, y) 6 diamFk 0,

contradiction.
(i) Consider X = (0, 1] with the usual metric d. Then (X, d) is not complete.
Take Fn = (0, 1/n]. Then Fn is
T closed in X, Fn+1 Fn for all n 1, and
diamFn = 1/n 0. However, n1 Fn = .
(ii) In R with the usual metric d consider F n = (0, 1/n].
Then diamFn =
T
1/n 0 and Fn is not closed in R. The intersection n1 Fn = .
(iii) In R with the usual metric d consider
T F n = [n, ). Then Fn is closed
in R, Fn+1 Fn but diamFn = , and n1 Fn = .
e are metric spaces and that f : X Y is
4. Suppose that (X, d) and (Y, d)
a bijection such that both f and f 1 are uniformly continuous. Show that
e is complete.
(X, d) is complete if and only if (Y, d)
e
Solution: Suppose that (X, d) is complete and {y n } is Cauchy in (Y, d).
e
We have to show that there exists y Y such that d(yn , y) 0 as n .
For every n there exists xn X such that f (xn ) = yn . We claim that {xn }
is Cauchy. Indeed, let > 0. Then there exists > 0 such that
(1)

e b) <
d(f 1 (a), f 1 (b)) < for all a, b Y satisfying d(a,

e there is
since f 1 is uniformly continuous. Since {yn } is Cauchy in (Y, d),
N such that

(2)

e n , ym ) < for all n, m > N .


d(y

Combining, (1) and (2), we get

d(xn , xm ) = d(f 1 (yn ), f 1 (ym )) <


for n, m > N . So {xn } is Cauchy and since (X, d) is complete, there is x X
e n , f (x)) =
such that d(xn , x) 0, and since f is uniformly continuous, d(y
e
e
d(f (xn ), f (x)) 0. So (Y, d) is complete.
5. Let {fn } be a sequence of continuous functions f n : R R with the
property that {fn (x)} is unbounded for all x Q. Using Baires theorem
show that there is at least one x Qc such that {fn (x)} is unbounded.
Solution: Arguing by contradiction assume that {f n (x)} is bounded for
every irrational x. For every k N, let
Fk = {x R | |fn (x)| 6 k for all n N}.

Observe that fn1 ([k, k]) is closed since fn is continuous, and consequently
Fk is closed since
\
fn1 ([k, k]).
Fk =
nN

Moreover, Qc
irrational x. So

kN Fk

since, by assumption, {fn (x)} is bounded for every


[
[
R=
Fk
{r},
kN

rQ

that is, R is a countable union of closed sets since Q is countable. Every


singleton {r} is nowhere dense. Consequently, in view of Baire theorem,
some Fk has nonempty interior. Say Fk 6= . So (a, b) Fk , and {fn (x)}
is bounded for any x (a, b). In particular, {f n (x)} is bounded for any
rational number x (a, b), contradiction.
e be metric spaces such that (X, d) is complete. Let
6. Let (X, d) and (Y, d)
{fn } be a sequence of continuous functions from X to Y such that {f n (x)}
converges for every x X. Using Baires theorem show that for every
> 0 there exist k N and a non-empty open subset U of X such that
e n (x), fm (x)) < for all x U and all n, m k.
d(f
Solution: Fix > 0. For every k N define
e n (x), fm (x)) 6 /2 for all n, m k}.
Fk = {x X| d(f

Each of Fk is closed. Indeed, let x0 Fk , then there exists {xl } Fk such


that d(xl , x0 ) 0 as l . Since x 7 d(fn (x), fm (x)) is continuous,
d(fn (x0 ), fm (x0 )) = lim d(fn (xl ), fm (xl )) 6 /2.
l

S
for all n, m k. Hence x0 Fk and Fk is closed. Moreover, X = k1 Fk
since for every x X, {fn (x)} converges. In view of Baire theorem, F k 6=
for some k. Thus, there exists y and r > 0 such that B(y, r) F k . Put
U = B(y, r). If x U , then for all n, m k,
d(fn (x), fm (x)) 6 /2 < .

Solutions to Problem Set 6.

1.
(a) Let f (x) = x2 for x (0, a], and let X = (0, a] with the usual metric.
Find for what values of a is f a contraction. Show that f : X X
does not have a fixed point.
1
(b) Let f (x) = x+ for x 1, and let X = [1, ) with the usual metric
x
d. Show that f : X X, that d(f (x), f (y)) < d(x, y) for all x 6= y
but f does not have a fixed point.
Reconcile (a) and (b) with Banach fixed point theorem.
Solution:
(a) For x, y (0, a],
|x2 y 2 | = |x + y| |x y| 6 [|x| + |y|] |x y| 6 (2a) |x y|
so that f is a contraction if a < 1/2. Observe that if x (0, a] with a < 1/2,
then clearly 0 6 x2 6 a2 < a so that f : (0, a] (0, a]. If x2 = x, then
x = 0 and x = 1. Hence f : X X does not have a fixed point in X. Note
that (X, d) is not complete.
1
(b) Clearly, x + 1 for x 1 so that f : X X. Observe next that
x

 
 

1
1
1
x+
y+
= 1
(x y)
x
y
xy
so that
|f (x) f (y)| =

1
1
xy

|x y| < |x y|

since 1 1/(xy) < 1 for all x, y 1. Assume that f has a fixed point x.
1
1
Then x + = x, so = 0, contradiction. Note that f is not a contraction
x
x
since otherwise 1 1/(xy) 6 for some constant < 1 and all x, y 1.
Taking y = 1 and x large we get contradiction.
2. Consider f : R2 R2 given by f (x) = Ax, where


0.7
0.8
.
A=
0.2 0.05
Is f a contraction if R2 is equipped with the metric d1 , d2 , d ?
Solution: If x = (a1 , b1 ) and x = (a2 , b2 ), then
  
   

a
0.7
0.8
a1
0.7
0.8
2

f (x) f (y) =
b2
0.2 0.05
b1
0.2 0.05


= 0, 7(a1 a2 ) + 0.8(b1 b2 ), 0.2(a1 a2 ) 0.05(b1 b2 ) .
1

So
d1 (f (x), f (y)) = |0, 7(a1 a2 ) + 0.8(b1 b2 )| + |0.2(a1 a2 ) 0.05(b1 b2 )|
6 0.9 |a1 a2 | + 0.85 |b1 b2 | 6 0.9 [|a1 a2 | + |b1 b2 |]
= 0.9 d1 (x, y).

Hence f : (R2 , d1 ) (R2 , d1 ) is a contraction.


The map f : (R2 , d2 ) (R2 , d2 )
is not a contraction. Indeed, take x = (1, 1)
and y = (0, 0). Then d2 (x, y) = 2 but f (x)f (y) = (0.7+0.9, 0.20.05) =
(1, 5, 0.15). Hence
p

d2 (f (x), f (y)) = 1.52 + 0.152 > 1.52 = 2.25 > 2 = d2 (x, y)

Finally, f considered as a map from (R 2 , d ) to (R2 , d ) is not a contraction.


Indeed, take x = (1, 1) and y = (0, 0). Then d (x, y) = d ((1, 1), (0, 0)) =
1, and
d (f (x)f (y)) = d (f (1, 1), f (0, 0)) = d ((1, .5, 0.15), (0, 0))
= max{1, 5, 0.15} = 1.5.
Hence there is no constant (0, 1) such that d (f (x), f (y)) 6 d (x, y)
for all x, y R2 .
3. Consider the system of nonlinear equations

x1 = b1 + sin(a11 x1 ) + sin(a12 x2 ) + + sin(a1n xn ),

x2 = b2 + sin(a21 x1 ) + sin(a22 x2 ) + + sin(a2n xn ),


..
..
.
.

xn = bn + sin(an1 x1 ) + sin(an2 x2 ) + + sin(ann xn ),


where aik , 1 6 i, k 6 n, and bk , 1 6 k 6 n are given realPnumbers. Show
that the system has a unique solution x = (x 1 , . . . , xn ) if 16i,k6n a2ik < 1.
Solution: For x = (x1 , x2 , . . . , xn ) Rn , define
fk (x) = bk +

n
X

sin(akj xj ),

1 6 k 6 n.

j=1

Let b = (b1 , . . . , bn ) and f (x) = (f1 (x), . . . , fn (x)). Then f : Rn Rn , and


the system has a unique solution x = (x 1 , . . . , xn ) if and only if x = f (x).

P
2 1/2 .
Put :=
16i,k6n aik
Claim: d2 (f (x), f (y)) 6 d2 (x, y), where d2 is the Euclidean metric on
Rn .
By the mean value theorem,
|sin(akj xj ) sin(akj yj )| = |akj cos(akj t)| |xj yj | 6 |akj | |xj yj |.

Thus, applying Cauchy inequality,


n

n
X 
 X

|sin(akj xj ) sin(akj yj )|
|fk (x) fk (y)| =
sin(akj xj ) sin(akj yj ) 6
6

j=1
n
X

j=1

|akj | |xj yj | 6

j=1
X
n
j=1

|akj |2

1/2

X
n
j=1

1/2
1/2 X
n
2
2
|xj yj |

|akj |

d2 (x, y),

and
(d2 (f (x), f (y))2 =

n
X
k=1

j=1

|fk (x) fk (y)|2 6

= (d2 (x, y)) .

 X
n


|akj |2 (d2 (x, y))2

16j,k6n

Taking a positive square root,


d2 (f (x), f (y)) 6 d2 (x, y).

Hence f : (Rn , d2 ) (Rn , d2 ) is a contraction with a constant < 1, and


since (Rn , d2 ) is complete, f has a unique fixed point x = (x 1 , . . . , xn ) Rn .
4. For f C([0, 1], R), define
(T f )(x) = ex +

1
f (x)

,
2 1 + f (x)2

x [0, 1].

Show that (T f ) C([0, 1], R) and that T : C([0, 1], R) C([0, 1], R) is a
contraction. Use this fact to show that there exists exactly one function
f C([0, 1], R) such that
1
f (x)3 ex f (x)2 + f (x) = ex
2

for all x [0, 1].


Solution: By the mean value theorem applied to the function h(t) =
t
, there exists c (a, b) such that
1 + t2



a
b 1 c2

1 + a2 1 + b2 = (1 + c2 )2 |a b| 6 |a b|.

Thus, if f, g C([0, 1], R), then,




1 f (x)
g(x) 1
6 |f (x) g(x)| 6 d(f, g)
|(T f )(x) (T g)(x)| =

2 1 + f (x)2 1 + g(x)2 2
for all x [0, 1]. Consequently,

d((T f ), (T g)) 6

1
d(f, g)
2

so that T : C([0, 1], R) C([0, 1], R) is a contraction. Since C([0, 1], R) is


a complete metric space, there is exactly one function f C([0, 1], R) such
that
1
f (x)
f (x) = ex +
2 1 + f (x)2
for all x [0, 1]. Now observe that f satisfies he above equation if and only
1
if f (x)3 ex f (x)2 + f (x) = ex
2
5. Let (X, d) be a metric space and let X be the set of Cauchy sequences
x = {xn } in (X, d). Define a relation in X be declaring x = {xn } y =
{yn } to mean d(xn , yn ) 0.
(a) Show that is an equivalence relation.
e denote the
Denote by [x] the equivalence class of x X , and let X
set of these equivalence classes.
(b) Show that if x = {xn } and y = {yn } X , then limn d(xn , yn )
exists. Show that if x0 = {x0n } [x] and y0 = {yn0 } [y], then
lim d(xn , yn ) = lim d(x0n , yn0 ).

e define
For [x], [y] X,

D([x], [y]) = lim d(xn , yn ).


n

Note that the definition of D is unambiguous in view of the above


equality.
e D) is a complete metric space.
(c) Show that (X,
e D). Then xn = {xn , xn , xn , . . .} is
Hint: Let [xn ] is Cauchy in (X,
1
2
3
Cauchy in (X, d). So for every n N, there exists k n N such that
d(xnm , xnkn ) < 1/n

for all m kn .

Set x = {x1k1 , x2k2 , x3k3 , . . .}. Then show that x is Cauchy in (X, d)
and D([xn ], [x]) 0.
(d) If x X, let (x) be the equivalence class of the constant sequence
x = (x, x, x, . . .). That is, (x) = [x] = [x, x, x, . . .]. Show that
: X (X) is an isometry.
e D).
(e) Show that (X) is dense in (X,
e
Hint: Let [x] X with x = {x1 , x2 , x3 , . . .}. Denote by xn the
constant sequence {xn , xn , xn , . . .} and show that D([xn ], [x]) 0.
Solution:
(a) Let x = {xn }, y = {yn } and z = {zn } X . Since d(xn , xn ) = 0,
x x, and since d(xn , yn ) = d(yn , xn ) we conclude that x y implies
y x. Finally, if x y and y z, then d(x n , yn ) 0 and d(yn , zn ) 0 so
that d(xn , zn ) 0 since d(xn , zn ) 6 d(xn , yn ) + d(yn , zn ) 0. Hence x z,
and is indeed an equivalence relation.
(b) Let x = {xn } and y = {yn } X . Since
d(xn , yn ) 6 d(xn , xm ) + d(xm , ym ) + d(ym , yn )

we have d(xn , yn ) d(xm , ym ) 6 d(xn , xm ) + d(ym , yn ). Writing the same


inequality but with m and n interchanged, we get
|d(xn , yn ) d(xm , ym )| 6 d(xn , xm ) + d(yn , ym ).
It follows that {d(xn , yn )} is Cauchy and since R with the usual metric is
complete, the sequence {d(xn , yn )} exists. Next, if {x0n } [x] and {yn0 }
[y], then
(1)

lim d(xn , yn ) = lim d(x0n , yn0 ).

Since
d(xn , yn ) 6 d(xn , x0n ) + d(x0n , yn0 ) + d(yn0 , yn ),
we get
(2)

lim d(xn , yn ) 6 lim d(x0n , yn0 ).

Interchanging xn with

x0n

and yn with yn0 , we get

lim d(x0n , yn0 ) 6 lim d(xn , yn )

which together with (2) implies (1).


(c) D is a metric. Indeed, D([x], [y]) 0. If 0 = D([x], [y]) = lim n d(xn , yn ).
This means that x y, and so [x] = [y]. Symmetry of D follows from
the fact that d(xn , yn ) = d(yn , xn ). To prove the triangle inequality, take
e Since limits limn d(xn , zn ), limn d(xn , yn ) and
[x], [y] and [z] X.
limn d(yn , zn ) exist, and d(xn , zn ) 6 d(xn , yn ) + d(yn , zn ), we have
D([x], [z]) = lim d(xn , zn ) 6 lim d(xn , yn ) + lim d(yn , zn )
n

= D([x], [y]) + D([y], [z]).


e D) is complete. Take a Cauchy sequence {[x n ]}
Next we will show that X,
e D). Here xn = {xn } = {xn , xn , xn , . . .} denotes a Cauchy sequence in
in (X,
1
2
3
k
e such that D([xn ], [x]) 0.
(X, d). We have to show that there exists [x] X
n
n
Since x = {xk } is Cauchy in (X, d), for every n there exists k n N such
that
1
for all m kn .
(3)
d(xnm , xnkn ) <
n
Consider the sequence
x = {xn } = {x1k1 , x2k2 , x3k3 , . . .}.

We claim that x is Cauchy in (X, d). Indeed, denoting by x nkn the constant
sequence {xnkn , xnkn , xnkn , . . .}, we have
(4)

D([xn ], [xnkn ]) = lim d(xnj , xnkn ) 6


j

1
.
n

Let and choose N N such that 1/N < /3 and


D([xn ], [xm ]) < /3

for n, m N .

Then,
n
m
d(xnkn , xm
km ) = D([xkn ], [xkm ])

6 D([xnkn ], [xn ]) + D([xn ], [xm ]) + D([xm ], [xm


km ])
1
1

6 + D([xn ], [xm ]) +
< + + =
n
m
3 3 3
for all n, m N . Next we will show that D([x n ], [x]) 0. In view of (4),
1
(6) D([x], [xn ]) 6 D([x], [xnkn ]) + D([xnkn ], [xn ]) 6 D([x], [xnkn ]) + .
n
Take > 0. Since, in view of (5), the sequence {x nkn } is Cauchy, there exists
k N such that 1/k < /2 and
(5)

d(xnkn , xm
km ) < /2

(7)

for all n, m k.

Fixing n k and taking a limit m in (7), we get

D([xnkn ], [x]) = lim d(xnkn , xjkj ) 6 /2

(8)

Combining (6) with (8),


1
< /2 + /2 = .
n
e D) is complete.
for all n k. Consequently, D([xn ], [x]) 0, and X,
(d) If x = (x, x, x, . . .) and y = (y, y, y, . . .) are constant sequences in X,
then (x) = [x], (y) = [y] so that
D([x], [xn ]) 6 D([x], [xnkn ]) +

D([x], [y]) = lim d(x, y) = d(x, y).


n

Hence : X (X) is an isometry.


e there exists a sequence [xn ]
(e) It suffices to show that for any [x] X
n
e with x = (x1 , x2 , x3 . . .).
(X) such that D([x ], [x]) 0. Let [x] X
n
Denote by x the constant sequence {xn , xn , xn , . . .}. Then (xn ) = [xn ],
and we claim that D([xn ], [x]) 0. Since x is Cauchy, for given > 0, there
exists k N such that
d(xn , xm ) < /2

for n, m k.

Fixing n k and taking a limit m , we get

lim d(xn , xm ) 6 /2 < .

So, for n k,

D([xn ], [x] = lim d(xn , xm ) < .


m

Thus, D([xn ], [x]) 0 as required.

Solutions to Problem Set 7.


1. Let (X, d) be a complete metric space and f : X X be a function
such that
d(f (x), f (y)) 6 d(x, y)
for all x, y B(x0 , r0 ), where 0 < < 1 and d(x0 , f (x0 ))/(1 ) = r0 .
Show that f has a unique fixed point p B(x 0 , r0 ).

Solution:
Since B(x0 , r0 ) is closed in a complete metric space (X, d), (B(x 0 , r0 ), d) is
complete. Hence it suffices to show that f : (B(x 0 , r0 )) B(x0 , r0 ). Take
any x B(x0 , r0 ). We have to show that d(f (x), x0 ) 6 r0 . Indeed,
d(f (x), x0 ) 6 d(f (x), f (x0 )) + d(f (x0 ), x0 ) 6 d(x, x0 ) + r0 (1 )
6 r0 + (1 )r0 = r0 .

Hence f : B(x0 , r0 ) B(x0 , r0 ), and f has a unique fixed point in B(x 0 , r0 ).


2. (a) Let I = [x0 a, x0 + a] and let r > 0. If f, g : I R are continuous,
define
d(f, g) = sup{er(xx0 ) |f (x) g(x)||x I}.
Show that d defines a metric on C(I, R) which is equivalent to the supremum
metric (f, g) = sup{|f (x) g(x)|| x I}.
(b) Let K = I J where I = [x0 a, x0 + a], J = [y0 b, y0 + b], and let
f : K R be continuous function satisfying |f (t, y 1 ) f (t, y2 )| 6 |y1 y2 |
for all (t, y1 ), (t, y2 ) K. Let C = sup{|f (t, y)|| (t, y) K}, and let =
min{a, b/C}. For a continuous function y : [x 0 , x0 + ] J, set
Z x
f (t, y(t))dt, x [x0 , x0 + ].
T y(x) = y0 +
x0

Show that T : C([x0 , x0 + ], J) C([x0 , x0 + ], J) is a contraction


with respect to the metric d(y1 , y2 ) = sup{e2(xx0 ) |f (x) g(x)||x I}.
Solution:
If d(f, g) =, then er(xx0 ) |f (x) g(x)| = 0 for all x I, and since
e(xx0 ) > 0, f (x) = g(x) for all x I. So f = g. Symmetry of d is
obvious. Since |f (x) h(x)| 6 |f (x) g(x)| + |g(x) h(x)|,
er(xx0 ) |f (x) h(x)| 6 er(xx0 ) |f (x) g(x)| + er(xx0 ) |g(x) h(x)|
6 d(f, g) + d(g, h)

for all x I. So d(f, h) 6 d(f, g) + d(g, h). Hence d is a metric.


Since x 7 er(xx0 ) is decreasing, era 6 er(xx0 ) 6 era for all x I.
Hence
era |f (x g(x)| 6 er(xx0 ) |f (x g(x)| 6 era |f (x g(x)|,

and so,
(1)

era (f, g) 6 d(f, g) 6 leqera (f, g)


1

xI

which implies that d and are equivalent. In view of (1), (C(I, R), d) is
complete since (C)I, R), ) is.
(b) If y : [x0 , x0 + ] J and x0 6 x 6 x0 + , then
Z x
|T y(x) y0 | 6
|f (t, y(t))|dt 6 C 6 b,
x0

so that T y(x) J for all x0 6 x 6 x0 + . Similarly, T y(x) J for


all x0 6 x 6 x0 . If y1 , y2 : [x0 , x0 + ] J are continuous and
x0 6 x 6 x0 + , then
Z x
Z x
|T y1 (x)T y2 (x)| 6
|f (t, y1 (t)) f (t, y2 (t))|dt 6
|y1 (t) y2 (t))|dt
x0
x0
Z x
=
e2(tx0 ) e2(tx0 ) |y1 (t) y2 (t))|dt
x0
Z x


1
6
e2(tx0 ) dt d(y1 , y2 ) = e2(xx0 ) 1 d(y1 , y2 )
2
x0
1
6 e2(xx0 ) d(y1 , y2 ).
2
The same holds for x0 6 x 6 x0 . Multiplying both sides by e2(xx0 )
we get
1
e2(xx0 ) |T y1 (x) T y2 (x)| 6 d(y1 , y2 )
2
for all x [x0 , x0 + ]. So d(T y1 , T y2 ) 6 (1/2)d(y1 , y2 ), and T is a
contraction with respect to d.
3. Let I = [x0 a, x0 + a] and let U = I R. Let f : U R be continuous.
(a) Let y0 , y1 R. Show that a function y : I R is a solution of
y 00 (x) = f (x, y(x)),
y(x0 ) = y0

xI

y 0 (x0 ) = y1
if and only if
y(x) = y0 + (x x0 )y1 +

x
x0

(x s)f (s, y(s))ds,

x I.

(b) Assume, in addition, that f satisfies the Lipschitz condition with respect
to the second variable, |f (x, y1 )f (x, y2 )| 6 |y1 y2 | for all (x, y1 ), (x, y2 )
U , and some > 0. Prove that for given y 0 , y1 R there exists > 0 such
that the equation y 00 (x) = f (x, y(x)) has a unique solution y : [x 0 , x0 +
] R satisfying y(x0 ) = y0 and y 0 (x0 ) = y1 .
Solution:
(a) Integrating y 00 (t) = f (t, y(t)) from x0 to s I we get
Z s
Z s
0
0
y (s) = y (x0 ) +
f (t, y(t))dt = y1 +
f (t, y(t))dt.
x0

x0

This holds for all s I. Integrating by parts from x 0 to x we get



Z x Z s
y(x) = y(x0 ) + (x x0 )y1 +
f (t, y(t))dt ds
x0
x0

Z s
Z x
d 
f (t, y(t))dt ds
= y0 + (x x0 )y1 +
s
x0
x0 ds
 Z s
x
Z x
= y0 + (x x0 )y1 + s
f (t, y(t))dt

sf (s, y(s))dt
= y0 + (x x0 )y1 + x
= y0 + (x x0 )y1 +

x0
x

x0

f (t, y(t))dt

x0

x0

x0

sf (s, y(s))ds

x0

(x s)f (s, y(s))ds.

Rx
Conversely, differentiating y(x) = y 0 + (x x0 )y1 + x0 (x s)f (s, y(s))ds
R
x
we get y 0 (x) = y1 + x0 f (s, y(s))ds. Differentiating again y 00 (x) = f (x, y(x))
for x I.
(b) Fix y0 , y1 R and let J = [y0 b, y0 + b]. Let C = sup{|f (t, y)|| (t, y)
I J}. For y C([I, J) define
Z x
T y(x) = y0 + (x x0 )y1 +
(x s)f (s, y(s))ds.
x0

Claim 1: Ty is continuous. Since the function x 7 (x x 0 )y1 is continuous


and aR sum of continuous functions is continuous, it is enough to show that
x
x 7 x0 (x s)f (s, y(s))ds is continous on I. Let x 1 , x2 I and x1 > x2 .
Then

Z x1
Z x2




(x

s)f
(s,
y(s))ds
(x

s)f
(s,
y(s))ds

2
1


x0
x0
Z x1

Z x2
Z x2


=
(x1 s)f (s, y(s))ds
(x1 s)f (s, y(s))ds +
(x2 x1 )f (s, y(s))ds
x
x0
x0
Z x2
Z x01
|f (s, y(s))|ds
|x1 s| |f (s, y(s))|ds + |x2 x1 |
6
x0

x2

6 2a C |x1 x2 | + 2a C|x1 x2 | = 4a C |x1 x2 |

The same holds if x1 < x2 . So T y is continuous.


Claim 2: There is a > 0 so that if I = [x 0 , x0 + ], and y C(I, J),
then T y(x) J for all x I.


Z x



(x s)f (s, y(s))ds
|T y(x) y0 | = (x x0 )y1 +
x
Z 0x
6 |x x0 ||y1 | +
|x s||f (s, f (s))|ds
x0

6 |x x0 ||y1 | + |x x0 |2 C 6 |y1 | + 2 C

Hence choose (0, a] so that [|y1 | + C] < b.


Claim 3: There is > 0 such that T : C(I, J) C(I, J) is a contraction

with respect to d(f, g) = sup{|f (t) g(t)||t I}. Indeed, if z 1 , z2 C(I, J),
then
Z x

Z x



|T z1 (x) T z2 (x)| = (x s)f (s, z1 (s))ds
(x s)f (s, z2 (s))ds
x0
Zx0x





= (x s) f (s, z1 (s)) f (s, z2 (s)) ds
x
Z x0
|x s| |f (s, z1 (s)) f (s, z2 (s))|ds
6
x0
Z x
6
|x s| |y1 (s) z2 (s)|ds
x0
Z x
6 d(z1 , z2 )
|x s|ds
x0

6 |x x0 | d(z1 , z2 ) 6 2 d(z1 , z2 )

So taking smaller > 0 so that 2 < 1, T is a contraction. Since C(I, J) is a


contraction with respect to d, T has a unique fixed point y : [x 0 , x0 +]
R.
4. Let (X, d) be a metric space with the property that if Y is any nonempty closed subset of X and f : Y Y is any contraction, then f has a
fixed point. Show that (X, d) is complete.
Solution:
Arguing by contradiction assume that there exists a Cauchy sequence {x n }
which does not converge in X. We may assume that x n 6= xm for all n 6=
m. For x X, let F (x) = inf{d(x, xn )|n N}. Then F (x) > 0 since
otherwise there exists a subsequence {x nk } of {xn } such that d(x, xnk ) 0.
Hence {xnk } converges to x and since {xn } is Cauchy, {xn } converges to
x, contradiction. Take (0, 1) and let n 1 = 1. Since {xn } is Cauchy,
there exists n2 > n1 such that d(xi , xj ) 6 F (xn1 ) for all integers i, j n2 .
Again by Cauchy condition there exists n 3 such that d(xi , xj ) 6 F (xn2 ) for
all i, j n3 . Proceeding in this way we find a strictly increasing sequence
of positive integers {nk } such that d(xi , xj ) 6 F (xnk1 ) for all integers
i, j nk . Now define Y = {xn1 , xn2 , xn3 , . . .} and let f : Y Y be given
by f (xnk ) = xnk+1 for all k 1. To see that Y is closed assume that there
is x Y but x 6 Y . Then there exists a sequence {y m } Y such that
d(ym , x) 0. But this mean that F (x) = 0, contradiction. So Y = Y . To
see that f is a contraction, take arbitrary two points of Y , say x nk , xnl . We
may assume that l k. Then in view of definition of {n k },
d(f (xnk ), f (xnl )) = d(xnk+1 , xnl+1 6 F (xnk )

= inf inf{d(xnk , xn )|n N} 6 d(xnk , xnl )

Consequently , f : Y Y is a contraction, and in view of Banach fixed


point theorem, there exists xnk Y such that f (xnk ) = xnk . But this mean,
in view of definition of f , that xnk = xnk+1 . This is impossible since xn 6= xm
for all n 6= m. This contradiction shows that (X, d) is complete.

5. Let A be a dense subset of a metric space (X, d), and let (Y, ) be complete. Consider a uniformly continuous function f : A Y . Show that
there exists a unique uniformly continuous function F : X Y such that
F (x) = f (x) for all x A.

Solution:
Let x X. Then there exists {xn } A such that d(xn , x) 0. Hence
{xn } is Cauchy in (A, d). Since f : A Y is uniformly continuous, {f (x n )}
is Cauchy in (Y, ). Since (Y, ) is complete. there exists y Y such that
f (xn ) y. Set F (x) = y. We have to show that this definition is independent of the choice of a sequence {xn }. If {x0n } A is another sequence such
that d(xn , y) 0. Then d(xn , x0n ) 0 and since f is uniformly continuous
(f (xn ), f (x0n )) 0. Since (f (xn ), y) 0, (f (x0n ), y) 0. Observe that
if x A, then F (x) = f (x) since a constant sequence {x, x, x, . . .} converges
to x. Let > 0 Since f is uniformly continuous, there is > 0 such that
(2)

(f (a), f (b)) < /3

for all a, b A satisfying d(a, b) <

Take arbitrary x, y X such that d(x, y) < /2. There are sequence
{xn }, {yn } A such that d(xn , x) 0 and d(yn , y) 0. Hence there
is k N such that d(xn , x) < /4 and d(yn , y) < /4 for all n k. So
d(xn , yn ) 6 d(xn , x) + d(x, y) + d(y, yn ) < /4 + /2 + /4 =

for n k. In view of (2),


(3)

(f (xn ), f (yn )) < /3

for n N .

Since f (xn ) F (x) and f (yn ) F (y) in (Y, ), there exists m N such
that
(4)

(F (x), f (xn )) < /3

and(F (y), f (yn )) < /3

for n m. Take N = max{k, m}. Combining (3) and (4) we get

(F (x), F (y)) 6 (F (x), F (xN )) + (F (xN ), F (yN )) + (F (yN ), F (y))


= (F (x), f (xN )) + (f (xN ), f (yN )) + (f (yN ), F (y))
6 /3 + /3 + /3 = .

for all x, y satisfying d(x, y) < /2. To see that there is exactly one extension
of f , assume that F, G : X Y are uniformly continuous and F (x) =
f (x) = G(x) for all x A. Let x X. Then there exists {x n } A such that
d(xn , x) 0. Hence F (xn ) F (x) and G(xn ) G(x) in (Y, ) since F, G
are uniformly continuous (for this you only need that they are continuous).
But F (xn ) = f (xn ) = G(xn ). So f (xn ) F (x) and f (xn ) G(x) imply
that F (x) = G(x).

Solutions to Problem Set 8.

1.
Show that if (X, dX ) and (Y, dY ) are compact metric spaces, then the product metric space (X Y, d) is compact. (Here d is the product metric).

Solution:
Let {xn , yn } X be any sequence in X Y . Since (X, d X ) is compact,
there exists a subsequence {xnk } converging to some point x in (X, dX ).
Since (Y, dY ) is compact, then a sequence {ynk } has a subsequence {ynkl }
{ynk } converging to some point y Y . Then a subsequence {(x nkl , ynkl )}
of {(xn , yn )} converges to (x, y) in the product metric d. Hence (X Y, d)
is compact.
2.
Show that if A1 , . . . ,Ak are compact subsets of a metric space (X, d), then
Sk
i=1 Ai is compact.

Solution: S
Let {xn } ki=1 Ai . Then one of the sets contains xn for infinitely many n.
Say {xnk } Ai . Since Ai is compact, {xnk } has a converging subsequence,
S
S
say {xnkl } converges to x Ai ki=1 Ai . Consequently, ki=1 Ai is compact.

3.
Which of the following subsets of R and R 2 are compact? (R and R2 are
considered with the usual metrics).
(a) A = Q [0, 1]
(b) B = {(x, y) R2 |x2 + y 2 = 1}
(c) C = {(x, y) R2 |x2 + y 2 < 1}}
(d) D = {x||x| + |y| 6 1}
(e) E = {x|x 1 and 0 6 y 6 1/x}
Solution:
(a) Q [0, 1] is not closed, and hence not compact
(b) B is closed and bounded in R2 , therefore compact
(c) C is open so not compact
(d) D is closed and bounded, therefore compact
(e) E is not bounded, therefore not compact.

4.
Consider (Q, d) where d is the usual metric. Give an example of a set in this
metric space that is closed and bounded but is not compact.
Solution:
Take, for example, A = Q [0, 1]. Since [0, 1] is closed in R with the usual
metric, A is closed. It is also bounded. However, A is not compact. Indeed,
for any irrational number there is a sequence of rational numbers converging
to it. So take an irrational number x [0, 1] and let {x n } be a sequence of
1

rational numbers in [0, 1] such that d(x n , x) 0. However, {xn } does not
converge in (Q, d) and so A is not compact.
5.
Let A be a non-empty compact subset of a metric space (X, d).
(a) Let x X. Show that d(x, A) = d(x, a) for some a A.
(b) Let U X be open and A U .
Show that there exists > 0 such that S = {x X|d(x, A) < } U . Does
this hold if A is only closed but not compact?
Soluton:
Clearly, d(x, A) 6 d(x, a) for all a A since, by definition, d(x, A) =
inf{d(x, y)| y A}. Conversely, for every n N, there exists a n A
such that d(x, an ) 6 d(x, A) + 1/n since otherwise d(x, A) + 1/n is a lower
bound of {d(x, a)| a A}. Since A is compact, there exists a subsequence
{ank } and a A such that d(ank , a) 0. So
d(x, a) 6 d(x, ank ) + d(ank , a) 6 d(x, A) + 1/nk + d(ank , a).
Taking a limit k , we get d(x, a) 6 d(x, A) since 1/n k and d(ank , a) .
Arguing by contradiction assume that for every n there exists x n S such
d(xn , A) < 1/n but xn 6 U . In view of (a), there exists an A such that
d(xn , an ) = d(xn , A). Since A is compact, there exists a subsequence {a nk }
and a A such that d(ank , a) 0. Then
d(xnk , a) 6 d(xnk , ank ) + d(ank , a) < 1/nk + d(ank , a)
and since the left side converges to 0, the left side d(x nk , a) 0. Since
a A U and U is open, there exists > 0 such that B(a, r) U . Since
d(xnk , a) 0, there exists k such that xnk B(a, ) contradiction xn 6 U
for all n.
n
n
Take R with
S the usual metric and let A = N. Let U n = (n 1/2 , n + 1/2 )
and U = n1 Un . Then A is not compact in R since it is not bounded.
S
Then for any > 0, S = n1 (n , ). For given > 0 take k such that
1/2k < . Then k + 1/2k S since k + 1/2k (k , k + ). However,
k + 1/2k 6 Un for all n N , i.e., k + 1/2k 6 U .
6.
Show that if A is a totally bounded subset of a metric space (X, d), then
for every
> 0 there exists a finite subset {a 1 , . . . , an } of A such that
S
A ni=1 B(ai , ).

Solution:
Let > 0. Since A is totally bounded
S in X, there exists a finite number
of points x1 , . . . , xk such that A 16i6k B(xi , /2). For each 1 6 i 6
k, B(xi , /2) A is non-empty so let ai B(xi , ) A. We claim that
B(xi , /2) B(ai , ). Indeed, if y B(xi , /2), then
d(ai , y) 6 d(ai , xi ) + d(xi , y) < /2 + /2 = .

Hence
A

16i6n

as required.

B(xi , /2)

B(ai , )

16i6n

7.
Show that a metric space (X, d) is totally bounded if and only if every
sequence {xn } X contains a Cauchy subsequence.

Solution:
Assume that (X, d) is totally bounded. Let {x n } be any sequence in X.
Since (X, d) is totally bounded, X can be covered by finitely many balls One
of these balls, say B(y1 , 1), contains xn for infinitely many ns. . Choose
n1 N such that xn1 B(y1 , 1). Since X can be covered by finitely many
balls of radius 1/2, B(y1 , 1)B(y2 , 1/2) for some y2 contains xn for infinitely
many ns. Choose n2 > n1 such that xn2 B(y1 , 1)B(y2 , 1/2). Proceeding
in this way we find a sequence {nk } of positive integers such that nk+1 > nk
for all k, a sequence of balls B(yk , 1/2k ), and a subsequence {xnk } satisfying
xnk B(y1 , 1) B(yk , 1/2k ). We claim that {xnk } is Cauchy. First
note that d(xnk , xnk+1 ) 6 d(xnk+1 , yk ) + d(yk , xnk ) < 1/2k + 1/2k = 1/2k1
since xnk+1 B(y1 , 1) B(yk , 1/2k ) B(yk+1 , 1/2k+1 ) B(y1 , 1)
B(yk , 1/2k ). For l > k,
d(xnk , xnl ) 6 d(xnk , xnk+1 ) + d(xnk+1 , xnk+2 ) + + d(xnl1 , xnl )
<

2i = 1/2k2 .

i=k1

Take > 0 and choose N N such that 1/2 N 2 < . Then for l, k 6 N ,
d(xnk , xnl ) 6 1/2N 2 < .

Consequently, {xn } has a Cauchy subsequence. Conversely, arguing by contradiction assume that (X, d) is not totally bounded, there is such that X
is not equal to a finite union of balls of radius . Take any point x X and
call it x1 . Then there exists a point x2 6 B(x1 , ). So d(x2 , x1 ) .
Since X 6= B(x1 , ) B(x2 , ), there exists x3 6 B(x1 , ) B(x2 , ).
That is d(x3 , x1 ) and d(x3 , x2 ) . Continuing this way we find
a sequence {xn } such that xn+1 6 B(x1 , ) B(xn , ). That is,
d(xn+1 , x1 ) , . . . , d(xn+1 , xn ) . The sequence {xn } has the property that d(xm , xn ) for all n 6= m, so it does not contain a Cauchy
subsequence.
8.
Let X be a compact metric space and let U be an open cover of X. Show
that there exists a number r > 0 with the property: For every x X, there
exists U U such that B(x, r) U . The number r is called a Lebesgues
number of the cover U.
Solution:
Since U is an open cover of X, for every x X, there is U U such that

x U and since U is open, there exists r x > 0 such that B(x, 2rx ) U . The
collection of balls {B(x, rx )}xX is a cover of X. Since (X, d) is compact,
there are x1 , . . . , xk such that
Take

X = B(x1 , rx1 ) B(x2 , rx2 ) B(xk , rxk ).

0 < r = min{rx1 , rx2 , . . . , rxk }.


Given x X, choose 1 6 i 6 k such that x B(x i , rxi ). Then for each
y B(x, r),
d(y, xi ) 6 d(y, x) + d(x, xi ) < r + rxi 6 2rxi .

So B(x, r) B(xi , 2rxi ) U for some U U.


Here is a proof using only sequences. Arguing by contradiction assume that
for every n there exists xn such that B(xn , 1/n) 6 U for all U U. Since
(X, d) is compact, there is a subsequence {x nk } converging to a point x.
The point x U for some U U, and since U is open B(x, r) U for some
r > 0. If y B(xnk , 1/nnk ), then
d(y, x) 6 d(y, xnk ) + d(xnk , x) 6 1/nnk + d(xnk , x),

and since d(xnk , x) 0 and 1/nnk 0 as k 0, we have that y B(x, r)


if k is large. That is, B(xnk , 1/nnk ) B(x, r) U for k large. Hence contradiction.

Solutions to Problem Set 9.

1.
Show that (X, T ) is a topological space.
(a) Let X be infinite set. Let
T = {A X| A = or A = X or X \ A is finite}.

This is called co-finite topology or finite complement topology.


(b) Let X be uncountable set. Define T by
T = {A X| A = or A = X or X \ A is countable}.

This is called co-countable topology or countable complement topology..


(c) Let X = R and let
T = {A R| A = or A = R or A = (a, ) with a R}.

Solution:
S
(a) Let {Ai } T . We will show
S that TiI Ai T . We may assume that
Ai 6= RTand Ai 6= . Then X \ iI Ai = iI [X \Ai ] X \Ak for
S any k I.
Hence iI [X \ Ai ] is empty T
or finite sinceSX \ Ai is finite. So iI Ai T .
If A1 , . . . , An T . Then X \ 16i6n Ai = 1i6i6n [X \ Ai ], and since a finite
T
T
union of finte sets is finite, X \ 16i6n Ai is finte. So 16i6n Ai T .
S
T
(b) Let {Ai } TS. Then X \ iI Ai = iI [X \ Ai ] X \ Ak for any
k I.The set X \ iI Ai is countable
as a subset
S of a countable set X \ A k .
T
If A1 , . . . , An T . Then X \ 16i6n Ai = 1i6i6n [X \ Ai ], and since a
T
finite union of countable sets is countable, X \ 16i6n Ai is counatble and
T
so 16i6n Ai T .
(c) Let {Ai }iI where Ai =S
(ai , ) with ai R. If the set {ai |i I} is not
bounded
from
below,
then
iI Ai = R; if it is not bounded from below,
S
then iI Ai = (a, ),
where
a = inf{ai | i I}. If Ai = (ai , ) with ai R
T
for 1 6 i 6 n, then 16i6n Ai = (a, ) T , where a = max{ai | 1 6 i 6 n}.
Hence (R, T ) is a topological space.
2.
Let B = {[a, b)| a, b R}. Show that B is a basis for a topology on R. This
topology, denoted Tl , is called the lower-limit topology on R. Show that
the lower-limit topology is larger than the usual topology on R . Find the
closures of [a, b), (a, b), (a, b] and [a, b] in (R, T l ).

Solution:
(a) If x R, then x [x, x + 1) B. Let x [a 1 , b1 ) [a2 , b2 ). Then
x [a, b), where a = max{a1 , a1 } and b = min{b1 , b2 }. So B is a basis
for a topology on R. Next we will show that T T l but T 6= Tl , where
T is the usual topology on R.
S Consider (a, b). Then there is N such that
a + 1/N < b, and (a, b) = nN [a + 1/n, b) Tl . Consider [a, b) Tl .
The point a is not an interior point of [a, b) since (c, d) [a, b) c 6= for any
(c, d) T containing a. So [a, b) 6 T , and T l 6= T .
1

S
[a, b) = [a, b) since [a, b) is closed. Indeed, (, a) = n1 [n, a) is open
since it is a union open sets, and similarly, [b, ) is open since [b, ) =
S
c
n1 [b, n). So, [a, b) closed since [a, b) = (, a) [b, ) is open.
(a, b) = [a, b) since a is a boundary point of (a, b), and [a, b) is closed since the
complement [a, b)c is open as a union of open sets, [a, b) c = (a) [b, ).
(a, b] = [a, b]. Indeed, (a, b) is the interior of (a, b] and a, b are the only
boundary points of (a, b] so that ov(a, b] = (a, b] (a, b] = {a, b} (a, b) =
[a, b].
[a, b] = [a, b], since [a, b] is closed in view of (a, b] = [a, b] and the general
fact that the closure of a set is closed.
3.
Let T = {A R| 0 6 A or A = R}. Show that T is a topology on R. What
are the closed sets in (R, T )? What is {1}? Is this topology Hausdorff?

Solution:
Clearly, and R T . Let {Ai }iI T . If one of the sets Ai = R, then
the union is S
equal to R. If not of them
S is equal to R, then 0 6 A i for all i
so that 0 6 iI Ai . In either
case,
iI Ai T . If A1 , . . . , An T , and
T
0 6 Ai forTsome k, then 0 6 iI Ai . If 0 Ai for all i, then Ai = R for all
i, and so iI Ai = R. Hence T is a topology on R. Closed sets in (R, T )
are complements of open sets. So and all sets C R such that 0 C are
closed. The closure of {1} is the smallest closed set containing {1}. Since
any non-empty closed set has to contain 0, {1} = {0, 1}.
The topology T is not Hausdorff since if x = 0, then the only open set
containing x is R, and for any open set A containing y 6= 0, R A = A 6= .
4.
Let A, B be a subsets of a topological space (X.T ). Show that A B AB
and A B = A B.

Solution: Since A B A and A B B, A B A and A B B.


So A B A B.
Since A A and B B, we have A B A B. The set on the right
side is closed, so A B A B. Conversely, A A B and B A B.
So A A B and B A B. Hence A B A B A B = A B.
Consequently, A B = A B.
5.
Let A X where (X, T ) is a topological space. Show that X \ A = X \ A
and (X \ A) = X \ A.

Solution:
Since A A, X \ A X \ A . Hence, X \ A X \ A = X \ A since
X \ A is closed (A is open). Conversely, let B be any closed set containing
X \ A. Then X \ B is open and X \ B A. Hence X \ B A since A is
the largest open set contained in A. Then, taking compliments, X \ A B.
Since this holds for any closed set containing X \A, we have X \A X \ A.

The proof of (X \ A) = X \ A is similar.

6.
Prove the following statements about continuous functions and discrete and
indiscrete topological spaces.
(a) If X is discrete, then every function f : X Y , where Y is any topological spaces, is continuous.
(b) If X is not discrete, then there exists a topological space Y and a function f : X Y that is not continuous.
(c) If Y is an indiscrete topological space, then every function f : X Y ,
where X is any topological space, is continuous.
(d) If Y is not indiscrete, then there exists a topological sapce X and a
function f : X Y that is not continuous. mbox

Solution:
(a) For any set A Y , f 1 (A) is open in discrete topology. So f is continuous.
(b) Take Y to be the set X with the discrete topology. Then any subset A
of Y is open. Take f : X Y to be the identity map, that is, f (x) = x for
all x X. Since X is not discrete, there is a set A X which is not open.
However, A is open in Y and f 1 (A) = A. So f is not continuous.
(c) There are only two open sets in Y , namely, and Y . Clearly, f 1 () = ,
and f 1 (Y ) = X. So f is continous.
(d) Since Y is indiscrete, there is a subset A different than and Y which
is open in Y . Take X to be the set Y with the indiscrete topology, and let
f : X Y be the identity map. Than f 1 (A) = A is not open so that f is
not continuous.
7
Let X be infinite set and let T be a co-finite topology on X. Show that any
continuous function f : X R is constant. (R is equipped with the usual
metric topology).
Solution:
Let f : X R be continuous. Arguing by contradiction assume that f
is not constant. So there are a, b f (X) so that a 6= b. There is also
r > 0 such that open intervals I = (a r, a + r) and J = (b r, b + r).
Since f is continuous and I, J are open then, f 1 (I), f 1 (J) are open sets
in (X, T ). The set f 1 (J) is open in (X, T ). So R \ f 1 (J) is finite and
f 1 (J) is infinite since X is infinite. Similarly, f 1 (I) is open which means
that X \ f 1 (I) is finite. However, since J I = , J R \ I so that
f 1 (J) f 1 (R \ I) = R \ f 1 (I) which says that an infinite set is contained in a finite set, contradiction.
8.
Let X and Y be topological spaces and let B be a base of open sets for Y .
Show that a function f : X Y is continuous if and only if f 1 (U ) is open
in X for every U B.

Solution:
Assume that f : X Y is continuous. Let U B. Then U is open in Y
and so, f 1 (U ) is open in X. Conversely, suppose that f 1
S (U ) is open in
X for every U B. Take an open set W Y . Then W = xW Ux , where
S
S
Ux B. Then f 1 (W ) = f 1 ( xW Ux ) = xW f 1 (Ux ). By assumption,
S
f 1 (Ux ) is open in X, so xW f 1 (Ux ) is open in X, and f 1 (W ) is open
in X. Hence f is continuous.
9.
(a) Show that (a, b) is homeomorphic to (c, d), (c, ) and R. (All spaces
are equipped with the usual topology).
(b) Show that R2 \ {(0, 0)} is homeomorphic to R2 \ B((0, 0), 1).

Solution:
dc
(x a) + c. Then f : (a, b) (c, d) is a bijection. f
(a) Take f (x) =
ba
ba
and its inverse f 1 (x) =
(x c) + a are continuous.
dc
Note that g defined by g(x) = 1/x 1 is a homeomorphism from (1, 0)
onto (0, ). By previous part there is a homeomorphism from (a, b) to
1
(x a). Then g f : (a, b) (0, ) is
(1, 0), for example take f (x) =
ba
a homeomorphism. Finally, Take h(x) = gf (x)+c. Then h : (a, b) (c, )
is a homeomorphism.
The map g(x) = tan x is a homeomorphism from (/2, /2) onto R.

The map f (x) =


(x a) /2 is a homeomorphism from (a, b) onto
ba



(x a) /2 is a home(/2, /2), and so h(x) = g f (x) = tan


ba
omorphism from (a, b) onto R.


(1 + a )
a, where a is the norm
(b) For a = (x, y) 6= (0, 0), let f (a) =
a
p
2
2


of a, a = x + y . Then




(1 + a ) (1 + a )

f (a) = a =
a = 1 + a > 1.

a

a

Hence f
: R 2 \ {(0, 0)} R2 \ B((0, 1), 1). For b = (x, y) R2 \ B((0, 0), 1),
b 1

let a =
b b. Then a 6= (0, 0), and f (a) = b. Hence f is onto. If
f (a) = f (b), then





f (a) = 1 + a = 1 + b = f (b) .


Hence a = b , and since f (a) = f (b), we get that a = b. So f is
one-one. Consequently,
f is a bijection. The inverse f 1 of f is given by

b 1
1 are continuous, f is a homeomor
f 1 (b) =
b b. Since f and f
phism.

10.
A topological property is a property that, if possessed by a topological
space X, is also possessed by any topological space homeomorphic to X.
(a) Show that if f : X Y is a homeomorphism, then f (U ) is open in Y
for any open set U X.
(a) Show that Hausdorff is a topological property.
(b) Is completeness a topological property of metric spaces?
Solution:
(a) Since f : X Y is a homeomorphism, f 1 : Y X is continuous. Hence for any open set U X, (f 1 )1 (U ) is open in Y . But
(f 1 )1 (U ) = f (U ), so f (U ) is open.
(b) Let f : X Y be a homeomorphism and let X be Hausdorff. Take
any two points a, b Y . Since f is a bijection, there are x, y X such that
x 6= y and f (x) = a and f (y) = b. Since X is Hausdorff, there are open sets
U and V X such that x U and y V and U V = . By (a), f (U ) and
f (V ) are open in Y . Moreover, a f (U ), b f (V ), and f (U ) f (V ) =
since if c f (U ) f (V ), then, in view that f is one-one, there is x U V
such that c = f (x). But this contradicts the fact that U V = .
(c) Let d be the standard metric d(x, y) = |x y| , x, y R. The map
f (x) = arctan x, x R is a bijection of R onto (/2, /2). Moreover,
f ; (R, d) ((/2, /2), d) and the inverse map f 1 : ((/2, /2) d)
(R, d), f 1 (x) = tan x, are continuous. So (/2, /2) d) and (R, d) are
homeomorphic. However, (R, d) is complete and ((/2, /2), d) is not complete since (/2, /2) is not closed in (R, d).

Solutions to Problem Set 10.

1.
Let (X, T ) be a compact topological space and let A, B are closed subsets
of (X, T ). Show that A B is compact.

Solution:
S
S
Let {Ui }iI be an open cover of A B. Then A iI Ui and B iI Ui .
Since
A and B are
S
S compact, there are finite sets J, K J such
S that A
iJ Ui and B
iI Ui . Then J K is finite and A B
iJK Ui . So
A B is compact.

2.
Let X = (0, 1) and let

T = {A R| A = or A = (0, 1) or A = (0, 1 1/n) for n 2}.

Show that every open set A different than X is compact. Is X compact?


Solution:
(a) Let A T and A 6= (0, 1). Hence A = (0, 1 1/k) for some k
1. Suppose that
S {Ui }i I is an open cover of A. Here I N. Since
(0, 1 1/k) iI Ui , there is i I such that (0, 1 1/k) U i . Hence A is
compact S
in this topology. Consider now the whole space. X = (0, 1). Then
(0, 1) = n2 (0, 1 1/n). However, (0, 1) cannot be covered by finitely
S
many of such sets since for any N N, 16n6N (0, 1 1/n) = (0, 1 1/N ).
So (X, T ) is not compact.
3.
Let T be a co-countable topology on R, that is,

T = {A R| A = or R \ A is countable}.

Is [0, 1] compact in (R, T )? What are the compact sets in (R, T )?

Solution:
The set [0, 1] is not compact in this topology. For example take the following
open cover. Let UnS= R \ {1/k|
S k n}. Then R \ Un = {1/k| k n} so
that Un T , and n1 Un = n1 [R \ {1/k| k n}] = R. So {Un }nN
is an open cover of [0, 1]. If [0, 1] were compact in (R, T , then [0, 1]
U1 U2 UN for some N > 1. However, U1 UN = R\{1/k|k N }
and [0, 1] 6 R \ {1/k| k N }, contradiction.
We claim that the only compact subsets of (R, T ) are finite sets. Clearly, if
F R is finite, then it is compact. Conversely, let A be any inifite subset
of R. Hence it contains a sequence {a n } such that an 6= am for all n 6= m.
Let xn = a2n , and let Un = R \ {xk | : k n}. Then R \ UnS= {xk | : k n}
is countable and so Un is open in this topology. Moreover, n1 Un = R, so
S
A n1 Un . If A were compact, then A U1 UN for some N > 1.
Since U1 UN = R \ {xk |k N }, x1 , x2 , . . . , xN 1 A but not in
U1 UN . Hence A is not compact in this topology. Consequently, A is
compact in (R, T ) if and only if A is finite.
1

4.
Consider (R, M), where M is the usual metric topology in R. Let
T = {A R| A = or R \ A is compact in (R, M)}.
(a) Show that T is a topology on R.
(b) Show that (R, T ) is compact but not Hausdorff.

Solution
(a) Clearly, and R T . Let {Ui }iI be any subcollection
of T
T . We may
S
assume that Ui 6= and Ui 6= R for all T
i. Then R \ iI Ui = iI [R \ Ui ]
with R \ Ui compact for all i. The set iI [R \ Ui ] is closed since it is the
intersection
of closed sets and it is bounded. T
So it is compact,
implying that
S
Tn
n
iI Ui T . If U1 , . . . Un are in T , then R \ i=1 Ui =
i=1 [R \ Ui ] is closed
and bounded since the
finite
union
of
closed
and
bounded
T sets is closed and
T
bounded. Thus, R \ ni=1 Ui is compact implying that ni=1 Ui T .
(b) Suppose not. Then there is an open cover {U i }iI of R without a finite
subcover. We may assume that Ui s are non-empty and not equal to R. Set
C
Tn = R \ Ui . Then each Ci is compact in
T R and forSevery finite set F I,
iF Ci 6= since otherewise R = R \
iF Ci =
iF Ui , contradiction.
Fix k I and consider collection of non-empty sets C k Ci , i I. Each
of this sets is closed in Ck and by above the family
{Ck Ci }iI has a fiT
nite
intersection
property.
Since
C
is
closed,
(C
k
iIS k Ci ) = .SHence
T
T
iI Ui ,
iI [R \ Ci ] =
iI Ci 6= . But this mean that R 6= R \ iI Ci =
contradicting the fact that {Ui }iI covers R.
Let x, y be distinct points of R. Suppose that U, V T are such that x U ,
y V and U V = . Then R = R \ (V V ) = [R \ U ] [R \ V ]. The sets
R \ U , R \ V are compact, so they are bounded, and their union is bounded
contradicting that R is unbounded.
5.
Let A be closed and let B be compact in (X, T ). Show that AB is compact.

Solution:
Let {Ui }iI be an open cover of A B. The set U = X \ A is open and the
sets {U } {Ui }iI form an open cover of B since B (X \ A) (A B).
Since B is compact, there are Ui1 , . . . , Uin such that B U Ui1 Uin .
Since U = X \ A, it does not contain points of A B and since A B B,
A B Ui1 Uin . Hence A B is compact.

6
Let (X, T ) be compact and let f : X R be a continuous function. Show
that f is bounded, that is, there is M > 0 such that |f (x)| 6 M for all
x X. Show that f attains its maximum and its minimum value.

Solution:
Since f is continuous, for every x X there exists an open set U x X
such that |f (y) f (x)| < 1 for all y U x . Clearly, {Ux }xX forms an
open cover of X. Hence X = Ux1 Uxn since (X, T ) is compact. Let
M = max{|f (x1 )|, . . . , |f (xn )|} + 1. If x X, then x Uxi for some xi , and

so |f (x)| 6 |(xi | + 1 6 M . Hence f is bounded, and C := sup{f (x)| x


X} < }.
S Assume that f (x) < C for all x X. In view of the definition of
C, X = nN Un , where Un = {x| f (x) < C 1/n} = f 1 ((, C 1/n)).
By compactness, X = U1 UN for some N N. Since Ui UN for
1 6 i 6 N , X = U1 UN UN . Hence for any x X, f (x) < C 1/N ,
but this mean that C = sup{f (x)|x X} 6 C 1/N , contradiction. This
contradiction shows that there is x 0 X such that f (x0 ) = C, that is, there
is a point where f attains its maximum.
7.
Let (X, T ) be compact and (Y, S) Hausdorff. Show that if f : X Y is a
continuous bijection, then f is a homeomorphism.
Solution:
Since f (X) = Y and f is one-to-one, we have to show that f 1 : Y X is
continuous. Hence we have to show that the preimage of any closed set in
X under f 1 is closed in Y . Let A X be closed. Hence it is compact in
(X, T ). Observe, that (f 1 )1 (A) = f (A) since f is a bijection. Hence f (A)
is compact in (Y, S). Since Y is Hausdorff, f (A) = (f 1 )1 (A) is closed, as
required.
8
Let (X, T ) be a compact Hausdorff space and let T 0 be another topology on
X. Show that:
(a) if T T 0 but T 6= T 0 , then (X, T 0 ) is Hausdorff but not compact.
(b) if T 0 T but T 6= T 0 , then (X, T 0 ) is compact but not Hausdorff.

Solution:
(a) Let x, y X. Since (X, T ) is Hausdorff, then there are U, V T
such that x U, y V and U, V are disjoint. Since T T 0 , U, V T 0
and so (X, T 0 ) is Hausdorff. Let f be the identity map, that is, f (x) = x
for all x. Then f : (X, T 0 ) (X, T ) is a bijection and continuous since
T T 0 . If (X, T 0 ) is compact, then f is a homeomorphism. Hence the map
f 1 : (X, T (X, T 0 ) is continuous. The inverse f 1 is the identity map,
f 1 (x) = x, x X. But the continuity of the identity map from (X, T to
(X, T 0 ) means that T 0 T . This together with the assumption T T 0
gives T = T 0 , contradiction with T 6= T 0 . So (X, T 0 ) is not compact.
(b) Let f : (X, T ) (X, T 0 ) be the identity map. Since T 0 T , f is
continuous. By the assumption (X, T ) is compact, so f (X) is compact in
(X, T 0 ). But f (X) = X. So (X, T 0 ) is compact. If (X, T 0 ) is Hausdorff,
then f is a homeomorphism since (X, T ) is compact and f is a bijection
and is continuous. So the inverse f 1 : (X, T 0 ) (X, T ) is continuous.
The inverse f 1 is the identity map, f 1 (x) = x for all x. The continuity
of the identity map from (X, T 0 ) to (X, T ) means that (X, T ) (X, T 0 ).
This together with (X, T 0 ) (X, T ) gives (X, T 0 ) = (X, T ), contradiction.
Hence (X, T 0 ) is not Hausdorff.
9.
Let X be infinite set with the co-finite topology T . Show that (X, T ) is

connected.
Solution:
Suppose that X = U V for some disjoint open sets U, V T . If U 6= and
U 6= X, then X \ U is finite. So V = X \ U 6= is finite, and so, V 6 T . So
either U = (and then V = X) or U = (and then V = ). Hence (X, T )
is connected.
10.
Is the topological space (R, T ) from Problem 4 connected?

Solution:
Suppose that R = U V for some non-empty disjoint open sets U, V T .
Then R \ U and R \ V are compact in (R, M). In particular, R \ U and R \ V
are bounded. Since R = U V and U V = , U = R \ V and V = R \ U .
Thus, U and V are bounded, and their union is also bounded, contradicting
that R is unbounded.

Solutions to Problem Set 11.

1.
Show that if A is a connected subspace of a topological space (X, T ) and if
A B A, then B is connected.

Solution:
Let f : B {0, 1} be any continuous function. We will show that f is constant. Assume not. Then f (B) = {0, 1}. Let g = f | A : A {0, 1}. Then g
is continuous and since g is constant, say g(A) = {0}, i.e., f (A) = {0}. Since
f (B) = {0, 1}, there is x B such that f (x) = 1. Since f is continuous and
{1} is open in {0, 1} (with the discrete topology). f 1 ({1}) = U is open in
B. Since x U and x B A, U A 6= . Take any y U A, then
on one hand f (y) = 0 (since y A) and on the other hand f (y) = 1 (since
y U ), contradiction.
2.
If A and B are connected subsets of a topological space (X, T ) such that
A B 6= , then A B is connected.

Solution:
Let x A B. Say Y = A B is disconnected. Then there exists a continuous surjection, f : Y {0, 1}, where {0, 1} is considered with discrete
topology. Since A is connected, so f (A) ( {0, 1}. Say f (A) = {0}. Further, f (B) ( {0, 1} since B is connected and so f (B) = {1} since otherwise
f (A B) = {0} contradictiong that f is a surjection. But, f is continuous;
in particular, f 1 ({1} is open in A B since {1} is open in {0, 1} ({0, 1} is
equipped with the discrete topology). Set U = f 1 ({1}. Then x U since
x B. Since also x A, then U A 6= . If y U A, then f (y) = 0 since
f (A) = {0}. But this contradicts that f is equal to 1 on U . Hence f cannot
be surjection, and so, Y is connected.
3.
Let {An } be a sequence of connected subsets of a topological
space (X, T )
S
such that An An+1 6= for all n N. Prove that nN An is connected.

Solution:
S
Let Y = nN An be disconnected. Then there exists f : Y {0, 1} where f
is a continuous surjection and {0, 1} is considered with the discrete topology.
Hence there are x, y Y such that f (x) = 0 and f (y) = 1. Since each A k
is connected and f |Ak : Ak {0, 1} is continuous, f (Ak ) is equal either {0}
or {1}. Since f is a surjection, there are n 6= m such that f ({A n ) = {0} and
f (Am ) = {1}. We may assume that n < m. We have f (A n ) = {0} and since
An An+1 6= , f (An+1 ) = {0}, then f (An+2 ) = {0} since An+1 An+2 6= ,
and so forth. So f (Am ) = {0}, contradicting f (Am ) = {1}.
4.
Let (X, d) be a metric space. Call a function f : X R locally constant if
for every x there exists r > 0 such that f | B(x,r) : B(x, r) R is constant.
1

Show that if (X, d) is connected, then every locally constant function is constant.
Solution: Fix x0 X and let f (x0 ) = a. Consider U = {x X | f (x) = a}.
Then U is nonempty since x0 U , and is open and closed since f is locally
constant. Indeed, if x U , then there is a ball B(x, r) on which f is constant, that is, for any y B(x, r), f (y) = f (x) = a. So B(x, r) U and
U is open. To see that U is closed, let x X \ U . Then f (x) 6= a and f
is constant on some ball B(x, r). So for any y B(x, r), f (y) 6= a since
f (y) = f (x) 6= a. This means that B(x, r) X \ U , Thus X \ U is open and
U is closed. Since X is connected, U = X.
5.
A metric space (X, dX ) is called a chain connected if for every pair x, y
of points in X and every > 0, there are finitely many points x = x 0 , x1 ,
x2 , . . . xn = y such that dX (xi+1 , xi ) < for i = 0, 1, . . . , n 1. Prove that
a compact, chain connected metric space is connected.
Solution:
Assume not. So, there is a continuous surjective function f : X {0, 1}.
We consider {0, 1} with the discrete metric d. Since (X, d X ) is compact,
then f is uniformly continuous. Let 0 < < 1, there is > 0 such that if
dX (x, y) < , then d(f (x), f (y)) < . Since f is a surjection, there are x and
y such that f (x) = 0 and f (x) = 1. Since X is chain connected, there is a
finite set of points x0 = x, x1 , . . . , xn1 , xn = y such that dX (xi , xi+1 ) <
for all 0 6 i 6 n 1. Thus, d(f (xi , xi+1 ) < for 0 6 i 6 n 1. Since d
is the discrete metric, f (xi ) = f (xi+1 ) for all 0 6 i 6 n 1. This mean
that f (x) = f (x0 ) = f (x1 ) = = f (xn1 ) = f (xn ) = f (y), contradicting
f (x) = 0 and f (y) = 1. So, X is connected.
6.
A point p X is called a cut point if X \ {p} is disconnected. Show that
the property of having a cut point is a topological property.
Solution:
Let X be a topological space with a cut point, say p X, and let f : X Y
be a homeomorphism. We claim that f (p) is a cut point of Y . Since X \ {p}
is disconnected, there are two disjoint sets U and V such that U and V are
open in X \ {p}, U V = X \ {p}. Then g = f | X\{p} : X \ {p} Y \ {f (p)}
is a homeomorphism, and so g(U ) and g(V ) are open in Y \ {f (p)}. Since
g(U ) g(U ) = , and Y \ {f (p)} = g(U ) g(V ). Hence Y \ {f (p)} is disconnected, and f (p) is a cut point of Y .
7.
Show that no two of the intervals (a, b), (a, b], and [a, b] are homeomorphic.
Solution:
Consider, (a, b) and (a, b]. Assume that f : (a, b] (a, b) is a homeomorphism. Let c = f (b). Then c is a cut-point of (a, b). So by the previous
problem, f 1 (c) = b is a cut point of (a, b], that is (a, b] \ {b} = (a, b) is

disconnected, contradiction.
The proofs for (a, b) and [a, b] is similar.
Consider (a, b] and [a, b]. Assume that f : [a, b] (a, b] is a homeomorphism. Since f is one-one, one of the points f (a), f (b) belongs to (a, b).
Say f (a) (a, b). Then f (a) is a cut point of (a, b) and so f 1 (f (a)) is a
cut point of [a, b]. But [a, b] \ {f 1 (f (a))} = [a, b] \ {a} = (a, b] which is
connected, contradiction. Hence (a, b], and [a, b] are not homeomorphic.
8.
Show that R and R2 are not homeomorphic (R and R2 are equipped with
the usual topologies) .
Solution:
First notice that R2 \{a} is connected since any two points x, y R 2 \{a} can
be joint by a continuous path. So R2 \ {a} is connected. Assume now that
R and R2 are homeomorphic. Then there is a homeomorphism f : R 2 R.
Denote x = f (0). The map g = f |R2 \{0} : R2 \ {0} R \ {x} is a homeomorphism. Since, by above, R2 \ {0} is connected, f (R2 \ {0}) = R \ {x} is
connected, contradiction.
9.
Let A be countable set. Show that R2 \ A is path connected.

Solution:
Let x, y R2 so that x 6= y. Since A is countable and there uncountably
many lines passing through x, there are uncountably many lines passing
hrough x and containing no point of A. Take one of such lines, say L.
Then L X \ A. If y L, then (t) = (1 t)x + ty L for t [0, 1],
(0) = x, (1) = y, so that x and y get be joint by a continuous path in
R2 \ A. Assume now that y 6 A. There are uncountably many lines through
y which dont intersect A and are not parallel to L. Pick one of them, say
K. Then y K, K R2 \ A, and K intersects L at a point z. Now let
(t) = (1t)x+tz, (t) = (1t)z+ty for t [0, 1]. Then (t), (t) R 2 \A,
(0) = x, (1) = z, (0) = z, and (1) = y. Finally, define
(
(2t)
0 6 t 6 1/2,
(t) =
(2t 1) 1/2 6 t 6 1.
Then is a continuous path in R2 \ A joining x with y.

10.
Show that if A is an open connected subset of R n , then A is path connected.

Solution:
Fix a point x0 A and let U be the collection of all x A such that there
is a path : [0, 1] A from x0 to x, that is (0) = x0 and (1) = x. The
set U is not empty since x0 U , and we claim that U is open. Indeed, let
x U . Since A is open, there is a ball B(x, r) such that B(x, r) A. We
will show that B(x, r) U . Take y B(x, r). Then there is a continuous
path (t) = (1 t)x + ty B(x, r), 0 6 t 6 1, joining x with y. Since there

is a continuous path : [0, 1] A such that (0) = x 0 and (1) = x, the


path defined by
(
(2t)
for 0 6 t 6 1/2
(t) =
(2t 1) for 1/2 6 t 6 1
satisfies (0) = x0 , (1) = y, (t) A for all t [0, 1]. Further, is continuous. So y U , and B(x, r) U , and U is open. Let V be the collection
of points x A which cannot be joint by a continuous path. Then V is
open. To see this assume that x V . Then B(x, r) A for some r. Any
point y B(x, r) can be connected by a continuous path with x, just take
(t) = (1 t)y + tx. If y can be connected to x 0 by a continuous path, then
x0 can be connected by a continuous path with x, contradiction. Hence
B(x, r) V . Since A = U V and U V = , and A is connected, one of
this sets has to be empty. Since x0 U , then V = . So A = U , and the
result follows.

S-ar putea să vă placă și